You are on page 1of 300
An Instructor's Solutions Manual to Accompany pT Tele] ile oltre) Vibrations Balakumar Balachandran Edward B. Magrab IM maoussluLL2n0! CENGAGE Learning” : o's CENGAGE © Learning” (© 2008, 2004 Cangage Learning ALL RIGHTS RESERVED. No part ofthis work covered by th ‘copyright herein may be reproduced, transmitted, stored, or ‘used in any form or by any means graphic, electronic, or ‘mechanical, incusing But not ited fo photocopying, recording, scanning, digzing, taping, Web distribution, information networks, oF information storage and retrieval systems, except as permitted under Section 107 or 108 of the 41978 United States Copyright Act, without the por written permission ofthe pubsher except as may be permitted by the fcense toms below. | For rod rfomation and tctnlegy seltnce contact engage Lanming Academie Resource Carter, | 0425-0563 For permission to use material rom this txt or product submit ‘requests chine at www.congage.compermissions. Furor pemissions quostons can be maliod to ermissionrequest@cengage.com. ISBN-13: 978-0-495-41 126.0 ISBN-10; 0.495-41126-4 Cengage Learning 200 First Stamford Place, Suite 400 Stamford, CT 06902 Usa ‘Cengage Learning isa leading provider of customized learning solutions wth ofc locators around the globe, Including Singapore, the United Kingdom, Australi, ‘Mexico, Brazil and Japan. Locate your local office at: International.cengage.comv/region ‘Congage Learning products are represented in | Canada by Nelson Education, Ld | roryou couse and ering sotto, vi Sealemiacenpaga conensoetn Purchase any of our products at your local college store or at ou” prefered online store ‘wor ichaptors.com ta ee an aan ed eet CRO ean iC oe SL READ IMPORTANT LICENSE INFORMATION Dear Professor or Other Supplement Recipient Cengage Losing has provided you with ths product che “Supplement for your review and, fo the extent that you adopt the associated textbook fr use Im connection wih your course (the. “Course, you and your students who purchase the fextbook ‘may’ use the Supplement as “described below. Cengage Learning. has estabished these use. Imatons In response to concems raed by author, professors, and other Users. regarding the pedagogical problems. stemming from Unlimited detrbuton of Supplements Ccongage Leaming herby grants you a nonzansferableleanse to.use the Supplomentin connection with the Course, subject to ‘the folowing conditions. The Supplement Is for your personal commercial use only and may not be reproduced, posted flewroniealy of dstibuted, excapt that portons of the Supplement may be provided to your students IN PRINT FORM ONLY in conection with your nstucton of the Course, so lng ‘as such studens are advised that they mey not copy of distrbute any porn of the Supplement to any tid party. Test Danks and oor testing matorals may be mage availabe inthe classroom and colected atthe end of each clas session, of Printed in {the United States of America} 1234567 1211100908 Balachandran 26 posted electronically 2s described hersin. Any material posted Sioctrenicaly must be though a password protected sto, with all Copy and download functonlty dsabled, and accessible solely by Your students who have purchased the associated textbook forthe Course. You may not sel, cense, suction, or ofervise redistribute the Supplement in any form. We ask that you take reasonable stops ‘fo protect. the. Supplenent ftom unauthorzed use, Feproduction or sistrbtlon, Your ae of tho Supplement indicates your acceptance ofthe condtions set forth n this Agreement. you Go nat accept these condone, you must retum the. Supplement ‘Gnused within 30 days of recelpt Al hts (including without tation, copyght,patants, and trade Secrets) inthe Supplement are and wil remain the sole. and exclusive property of Cengage Lesming andor is loensers. The Supplement is furnished by Cengege Leaming on an “as Is" basis without any warantes, express orimpled. This Agreement wil bs ‘governed by and construed pusvant to the laws of fo State of Few York, without regard to such State's confit of aw ules “Thank you fr your asitance in helping to safeguard the Integy ofthe content contained in his Supplement. Wo ust you fa the Supplement a ust teaching oo. Instructor’s Solutions Manual to Accompany Vibrations, Second Edition Balakumar Balachandran, University of Maryland Edward B. Magrab, University of Maryland Table of Contents Chapter 1 — Introduction . rb ibratory Systems Chapter 2 ~ Modeling of Chapter 3 — Single Degree-of-Freedom Systems: Governing Equations... 35 Chapter 4 ~ Single Degree-of-Freedom Systems: Solution for Response and Free Response Characteristics . ses 81 Chapter 5 — Single Degree-of-Freedom Systems Subjected to Periodic Excitations 113 147 Chapter 6 — Single Degree-of-Freedom Systems Subjected to Transient Excitations . Chapter 7 — Multi Degree-of Freedom Systems: Governing Equations and Natural Frequencies and Mode Shapes ......... a 165 Chapter 8 — Multi Degree-of-Freedom Systems: General Solution for Response and Forced Oscillations 243 Solutions te Exercises ~ Chapter 1 Solutions to Exerci: Chapter 1 Section 1.1 1.1 Choose any two contributors from Table 1.1, study their contributions, and write a paragraph about each of them. Section 1.2.1 1.2 Consider the planar pendulum kinematics discussed in Example 1.1, start with position vector r”/° resolved in terms the unit vectors i and j, and verify the expressions obtained for the acceleration and velocity given by Eq. (f) of Example 1.1 Solution 1.2 ‘The relationships among the unit vectors are €, = cosOi+sindj e, =—sindi+cosOj Then, the position vector is ((h=Lcos0)j) d 01) +— OE 6 (cosOi + Lsindj) dt = Leos00i+ Lsin06j = = Lbe, The acceleration vector is, ro ad ia, ~Lsin06* + Lcos66)i+(Leos06? + Lsin08) j (Leos06i) +Lbsinodj) 6? (-sin0i + cos0j) + LO(cosOi + sinOj) Lb e, + 10’ e, 1.3 Consider the kinematics of the rolling disc considered in Example 1.2, and verify that the instantaneous acceleration of the point of contact is not zero, 1 (©2009 Cengage Learning. ALL RIGHTS RESERVED Solutions to Exercises ~ Chapter 1 Sol jon 1. Making use of Eq. (1.7) that relates accelerations of two arbitrary points on a rigid body, we have 0° =” +0x(oxe") +x From Example 1.2, we have and, hence, +10 j-rbi =i Thus, it is verified that the instantaneous acceleration of the point of contact is not zero. 1.4 Show that the acceleration of the particle in the rotating frame of Example 1.3 is a=(%, -20y,-a'x, ~ ay, )e, + (jj, + 20%, —@"y, +X, )e, where @ is the magnitude of the angular acceleration of the rotating frame about the z axis, Solution 1.4 From Eq. (b) of Example 1.3 dr FG, -0y,)¢, +5, +08,)¢, ‘Then the acceleration is dy a dt [G,-0y,e]+2[0, +04, 4] =a ts,-0y)+64,-07) +e, £6, 105)46, 405) =(%, - oy, —ay,)e, +(x, — OY, (Oxe,) + (jj, + Ox, +ax,)e, +(V, +Ox,(Mxe,) where a = da/dt and we have used Eq. (1.8). We note that @ = wk; therefore, xe, =okxe, =0e, xe, =0kxe, =-08, Then 2 (©2009 Cengage Learning. ALL RIGHTS RESERVED. Solutions to Exercises ~ Chapter 1 a=(%, ~ 20}, -a°x,-ay,)e, + (33, + 20%, —@y, + ax, )e, ip — 2WYp O°, — AY, )e, + (Fp + 2M, — Oy, + Oye, 15 In Figure E1.5, a slider of mass M, is located on a bar whose angular displacement in the plane is described by the coordinate @ The motion of the slider from the pivot point is measured by the coordinate ry. The acceleration due to gravity acts in a direction normal to the plane of motion. Assume that the point O is fixed in an inertial reference frame and determine the absolute velocity and absolute acceleration of the slider. Se 15 Choose unit vectors e,’ and ey’ fixed to the slider as shown in the figure. Then, the position vector from point O to the slider is ie) +5 (xe}) Noting that @ = Ok, where k is the unit vector that points out of the plane, we find that v, = fel + Ok xe, = el + nde, The absolute acceleration is found from a, =“ (ie + nbet) = et +4 Hh + bes + bes +d a dt at = ie, +i, (wxet) +i bes + res +46 (wxes) = (I-48 ef +(n0-+ 278) es 1.6 A pendulum of mass m is attached to a moving pivot of mass M as shown in Figure E16. Assume that the pivot point cannot translate in the vertical direction, If the horizontal translation of the pivot point from the fixed point O is measured by the coordinate x and the angle @ is used to describe the angular displacement of the pendulum from the vertical, determine the absolute velocity of the pendulum. 3 (©2009 Cengage Learning. ALL RIGHTS RESERVED Solutions to Exercises ~ Chapter 1 Solution 1.6 The position vector of the location of the mass with respect to the fixed-point 0 is given by 1207 4 1,=xi-le, ‘Then the velocity is v, % ie, = 81—I(« x e,) = 31-I(-OK) xe, = Xi-Ipe, = xi—19(cosgi-sing/) =(4-Ipcos)i+lpsingj We could have also obtained the solution as follows: xi-Isingi-lcospj x-Ising)i—leosgj ‘Then the velocity is =(%-Ipcose)i+losingj Section 1.2.2 1.7 Determine the number of degrees of freedom for the systems shown in Figure E1.7. ‘Assume that the length Z of the pendulum shown in Figure E1.7a is constant and that the Tength between each pair of particles in Figure E1.7b is constant. Hint: For Figure E1.7c, the rigid body can be thought of as a system of particles where the length between each pair of particles is constant. Solution, a) Three coordinates ( = 3) and one inextensible constraint (m = 1): number of degrees of freedom is 3 — 1 = 2. b) Three particles, each with 3 coordinates and three inextensible constraints: number of degrees of freedom is 9-3 = 6. c) Rigid body: number of degrees of freedom is 6; 3 translation and 3 rotational. The system of three particles shown in Figure E1.7b is an example of a rigid body, since the particles are a constant distance apart from each other. In three-dimensional space, for a system of N particles representing a rigid body, the number of degrees of freedom is 3’ — GN ~ 6), where 3N is the total number of coordinates associated with tae N particles and 3N ~ 6 is the total number of inextensional constraints that ensures thet the particles are always at a constant distance apart. By extension to the case shown in Figure E1.7e, the number of degrees of freedom is 6. 4 (©2009 Cengage Learning. ALL RIGHTS RESERVED. Solutions to Exercises ~ Chapter | Section 1.2.3 1.8 Draw free-body diagrams for each of the masses shown in Figure E1.6 and obtain the equations of motion along the horizontal direction by using Eq. (1.15). Solution 1.8 ‘The free-body diagrams for each of the masses in an figure E1.14 are as shown to the right. The force N is shown in the free-body diagram of mass M to account, for the constraint that the pivot mass cannot move in, the j-direction. Summing forces along the édirection, for each of the two masses, we find for the pivot mass from Eqs. (1.15) that -Tsing = Mé and for the pendulum mass that Tsing = m(¥ +19" sing -Ipcos) me 1.9 Draw the free-body diagram for the whole system shown in Figure E1.6, obtain the system equation of motion by using Eq. (1.14) along the horizontal direction, and verify that this equation can be obtained from Eq. (1.15). Solution 1.9 ‘The free-body diagram is shown to the right. Note that the internal force T does not appear in this diagram, Making use of Eqs. (1.14) for the motion along the horizontal direction, we find that Me 0 = Mi+m(i#-+19" sing -Igcosg) which can be obtained by adding the two equations obtained to the solution to Exercise 1.8. ™ 1.10 Determine the linear momentum for the system shown in Figure E1.5 and discuss if itis conserved. Assume that the mass of the bar is Mar and the distance from the point O to the center of the bar is Liar. Solution 1.10 ‘The linear momentum of the system is given by P= Poor + Pair where Poor = Micelra, 9 5 (©2009 Cengage Learning. ALL RIGHTS RESERVED Solutions to Exercises ~ Chapter 1 Making use of the slider velocity V» determined in the solution to Exercise 1.5, we arrive at Paser = M, (fe, +7 6e') Thus, P= M, fel +(MygLsa +7) Oe) Since there are no external forces acting on the system, by virtue of Eq. (1.14), the system’s linear momentum is conserved. 1.11 Determine the angular momentum of the system shown in Figure 1.6 about the point O and discuss if it is conserved. Solution 1.11 ‘The angular momentum of the system about the point 0 is given by HE =H yetitan + Has [Ok +1, xp where A is the unit vector normal to the plane, the position vector r» runs from point O to the pendulum, and p is the linear momentum of the pendulum. Making use of the solution to Example 1.4, we have 1, =(R+reosp)el +rsingel p=nV, =m(-r(0+4)singe; +(R0+r(p+6)cosp)es) Joi-+(R+rc0sp)(R6-+r($+4)cosg) +r? (+8) Jk which can be written as H=[Jo0+ R047 (946) +R (26+ @)coso |k If no external moments act on the system, since O is a fixed point, from Eq. (1.17) it is clear that the angular momentum of the system is conserved. 1.12 A rigid body is suspended from the ceiling by two elastic cables that are attached to the body at the points 0" and O”, as shown in Figure E1.12. Point G is the center of mass of the body. Which of these points would you choose to carry out an angular-momentum balance based on Eq, (1.17)? Solution 1.12 Since point G is the center of mass of the body, this point would be used to carry out an angular momentum balance based on Eq. (1.17). One cannot use Eq. (1.17) with points 0’ and O” since neither of them is a fixed point. 6 (©2009 Cengage Learning, ALL RIGHTS RESERVED. Solutions te Exercises ~ Chapter | 1.13 Consider the rigid body shown in Figure E1.13. This body has a mass m and rotary inertia Jg about the center of mass G. It is suspended from a point O on the ceiling by using an elastic suspension. The point of attachment O' is at a distance / from the center of mass G of this body. (0) is an external moment applied to the system along an axis normal to the plane of the body. Use the generalized coordinates x, which describes the up and down motions of point O” from point O, and 0 which describes the angular oscillations about an axis normal to the plane of the rigid body. For the system shown in Figure E1.13, use the principle of angular-momentum balance given by Eq. (1.17) and obtain an equation of motion for the system. Assume that gravity loading is present. Solution 1.13 The principle of angular-momentum balance given by Eq. (1.17) is applied with respect to the center of mass of the system. Thus, d Mk = Jk) where Aris the unit vector normal to the plane of the system. ‘Then we have -M()=J¢8 If, instead, the principle of angular-momentum balance given by Eq. (1.17) is applied with respect to the fixed point O of the system, the result is -M()k+[(x-Isin0)i-1cos6j]x(mgi) {sob +[(x-/sind)i-1c0s0j]x(m¥,,)) = 4 (Yooh +[(e- Isind)i—1e0s0j]x[ m((i-16c0s0) i+ 16sin04))) where, we have substituted the velocity of the center of mass from the solution to Exercise 1.13. Carrying out the different cross-product operations, we obtain (-M(W) + mglcos0)k = JO +m4(i(x-Isin0)dsind +1(i—I6c0s0)cos0) k which leads to —M(1) + mglcos6 = — 16.0030) cos0) Sand ae + ml-S ((x—1sind) dsind +( Section 1.2.4 1.14 For the system shown in Figure E1.13, construct the system kinetic energy. 7 ©2009 Cengage Learning. ALL RIGHTS RESERVED Solutions to Exercises ~ Chapter 1 Solution 1.14 We make use of Eq. (1.23) to determine the system kinetic energy. Note that the velocity of the center of mass G can be obtained as in the solution to Exercise 1.6; that is, V,, =(%-10c0s0)i+i6sin0j Then, from Eq. (1.23), the kinetic energy is 1 1 2 T= Job Yuu) +5 Je 4 m( i? +276? -2:16 cos) at IP 2 2 1.15 Determine the kinetic energy of the planar pendulum of Example 1.1. 4 Solu 1.15 From Eqs. (b) and (¢) of Example 1.1, we find that v — Le, — LO(cos0i+sin0j) Then, from Eq, (1.22), the kinetic energy is 1 T= tm») 20 (cos 0+ sin? 0) = 1 m0? 1.16 Consider the disc rolling along a line in Figure E1.16. The disc kas a mass m and a rotary inertia Jc about the center of mass G. Answer the following: (a) How many degrees of freedom does this system have? and (b) Determine the kinetic energy for this system. Solution 1.16 a) One degree of freedom, since one independent coordinate (x or 0) is needed to describe the motion. Due to the non-slip constraint, x =r, and x or @ can always be expressed in terms of the other coordinate. b) Making use of Eq, (1.24), we have Noting from Example 1.2 that we arrive at 8 (©2009 Cengage Learning. ALL RIGHTS RESERVED. Solutions to Exercises ~ Chapter 1 1.17 In the system shown in Figure 1.6, if the mass of the pendulum is m, the length of the pendulum is r, and the rotary inertia of the disc about the point O is Jo, determine the system kinetic energy. Solution 1.17 ‘The kinetic energy of the system can be written as T= Tenn +a where 1 Tyna = 5" Vn Ve rss = Vy Ve) 1 52 Trac = 5 Job 2 Noting from the solution to Example 1.4 that -r(9+68)singe; + (26+7(9+6)cosp)es we determine that T jedton = 1.18 Referring to Figure E1.6 and assuming that the bar to which the pendulum mass m is connected is massless, determine the kinetic energy for the system. Solution 1.18 ‘The kinetic energy of the system is given by T=T y+ Tyenton where tr, pend =hm(¥-¥,) $M (it: 2 Making use of the solution for the velocity in Exercise 1.6, we find that = ; m((i-Igcose)i+lpsings)-((x-Ipeosp)i+losingj) ©2009 Cengage Learning. ALL RIGHTS RESERVED Solutions to Exercises — Chapter | Thus, 1 22, 1 2 zm[(e-Weose) 40 @ sin’ 9 +5 Me =F (m+ Me +hm(r@? —2ilpcose) 1.19 Determine the kinetic energy of the system shown in Figure E1.5. Solution 1.19 ‘The kinetic energy of the system is given by T = Tye + Totter where 1 cae = My Wy Va) Try =I? 2 Making use of the velocity determined in the solution of Exercise 1.5, we find that Tae = 5 My (fel rb) (ie; nds) = M, (8 +04) and, therefore, Ii +hM, (@ +76") = (Jo+M yp) O she 10 (©2009 Cengage Learning. ALL RIGHTS RESERVED. Solutions to Exercises - Chapter 2 Solutions to Exercises Chapter 2 Section 2.1 2.1 Examine Eqs. (2.1) and (2.5) and verify that the units (dimensions) of the different terms in the respective equations are consistent. Solution 2.1 In Eqs. (2.1), the units of Jg are kg-m? and the units of md” are kg-m. In Eq. (2.5), the units of energy are N-m, and the units of nv are kg(m/s)* = kg(m/s’)m = N-m. Section 2.2 2.2 Consider the slider mechanism of Example 2.2 and show that the rotary inertia Jor about the pivot point O’ is also a function of the angular displacement 9. Solution 2.2 ‘The rotary inertia is Jo =Jy(B)+ Iq, +Iq, +5, where Pr a Poly. , In (B) = mia tma -mlE-(8) +a -atent] To determine how f depends on g, we use geometry. First P(—)= 4? +b*—2abcosp a =r'(g) +b? -2r(g)bcos(4—¢ - f) Combining the above equations, we obtain 7'(—) =1'(g) +b -2r(g)bcos(a — g - B) +b? -2abcosp which leads to b=acosg +r(g)cos(z — 9 - B) and, hence, costa — p= PASS? MW ©2009 Cengage Learning, ALL RIGHTS RESERVED. Solutions to Exercises ~ Chapter 2 thus, resulting in =n—9-cos'i( $= a0088 pan-o-ca( 9552) ‘Thus, we have expressed the angle f in terms of the angle g. Thus, since J,, is a function of g, Jo is also a function of g. 2.3 Consider the crank-mechanism system shown in Figure E2.3. Determine the rotary inertia of this system about the point O and express it as a function of the angular displacement @. The disc has a rotary inertia J about the point O. The crank has a mass ‘mg and rotary inertia Jg about the point G at the center of mass of the erank. The mass of the slider is mp. Solution 2.3 The total inertia of the system is J ly =JytJ,+Iq where J, is the rotary inertia of the slider about the point O, J; is the rotary inertia of the crank about the point O and and ra is the distance from O to the point G and rp is the distance from O to the point p. From geometry, we see that (reos@ +Ieosy)’ +d? 1% =(rcos0+acosy) +(rsind—asiny)” In addition, rsind =d+lsiny = sin! (180 = 2 1 Thus, the total inertia of the system Jo can be expressed as a function of 8, therefore, Section 2.3.2 2.4 Find the equivalent length ZL. of a spring of constant cross section of diameter d that has the same spring constant as the tapered spring shown in Case 2 of Table 2.3. Both springs have the same Young’s modulus E, 12 (©2009 Cengage Learning. ALL RIGHTS RESERVED. Solutions to Exercises — Chapter 2 Solution 2.4 From Cases 1 and 2 of Table 2.3, we have that AE _ dE L, 4L, Therefore, the equivalent length is 2.5 Extend the spring combinations shown in Figures 2.6b and 2.6c ‘o cases with three springs. Verify that the equivalent stiffness of these spring combinations is consistent with Eqs. (2.14) and (2.16), respectively. Solution 2.5 In Figure 2.5b, we extend Eq. (2.13) to obtain F(x) -AG)+h@)+ BE) Shr thx thr =(k +k +h)x= which is consistent with Eq. (2.14). In Figure 2.6c, we extend Eq. (2.15) to obtain, Xex ta tH, p+EE.(2 which is consistent with Eq. (2.16). 2.6 Consider the mechanical spring system shown in Figure E2.6. Assume that the bars and determine the equivalent spring constant k., which we can use in the relation Solution 2.6 If the left-hand portion of the spring moves by an amount 5 when the hinged point moves down by an amount x, then VE hx} =5- Ph x -2hx eG 1B (©2009 Cengage Learning, ALL RIGHTS RESERVED. where This leads to Solutions to Exercises ~ Chapter 2 Gal 12h ||_a he _ he 2(a/2y J} 2(@/2yr Since the right-hand portion of the spring also moves by an amount 6, the potential energy in the spring is 4 = V = 5426) ‘Therefore, the equivalent spring constant is ke (2) @ 2.7 Consider the three beams connected as shown in Figure E2.7, The beam that is attached to the ends of the two cantilever beams is pinned so that its ends can rotate unimpeded. Determine the system’s equivalent spring constant for the transverse loading shown. Solution 2.7 ‘The equivalent system is shown at the right. ‘The cantilever beams act as two springs in parallel, with the combination spring 4 having an equivalent stiffness of k. This parallel spring combination is in series with the pinned-pinned beam. ‘Thus, se ae (ea) where, from Table 2.3, Note that as k. > 00, ke —> kas. (©2009 Cengage Learning. ALL RIGHTS RESER’ Solutions te Exercises — Chapter 2 2.8 For the weightless pulley system shown in Figure E2.8, determine the equivalent spring constant. Recall that when the center of the pulley moves by an amount x, the rope moves an amount 2x. Solution 2.8 If the pulley attached to spring &, moves down an amount x; and the pulley attached to spring k, moves up an amount x, then the displacement of each spring is The total displacement x of the springs is seay tage te ‘Therefore, the equivalent spring constant is pele Ly “alk, 2.9 Determine the equivalent stiffness for each of the systems shown in Figure E2.9. Each system consists of three linear springs with stiffness k1, kz, and ks. Solution 2.9 (a) The two springs in parallel have an equivalent spring constant kg =k +hy and this equivalent spring is in series with the third spring; therefore, the equivalent spring constant for the system is 1h peftat “ : i) (b) The two springs in series have an equivalent spring constant w-(iez) and this equivalent spring is in parallel with the third spring; therefore, the equivalent spring constant for the system is Kea hy thy 15 ©2009 Cengage Learning. ALL RIGHTS RESERVED. Solutions to Exercises ~ Chapter 2 2.10 For the two cantilever beams whose free ends are connected to springs shown in Figure E2.10, give the expressions for the spring constants 1 and A and determine the equivalent spring constant k, for the system. Solution 2.10 From Table 2.3 be ky > ky=htks > : ode 2.11 For the system of translation and torsion springs shown in Figure E2.11, determine the equivalent spring constant for torsional oscillations. ‘The disc has z radius b, and the translation springs are tangential to the disc at the point of attachment Solution 2.11 From Table 2.3, Case 8, we have for the two torsion in series that 16 (©2009 Cengage Learning. ALL RIGHTS RESERVED. Solutions to Exercises — Chapter 2 ‘The torque required to rotate the disc is TH1,, 40,40 4% =h,0+k,0 + B°OK, + BOk,, = (ky, +h, +07, +B%,,)O Rewriting this result as rHko ‘we find that the equivalent stiffness is Kah +h, +0? + By 2.12 For the system of translation springs shown in Figure E2.12, determine the ‘equivalent spring constant for motion in the horizontal (x) direction only. Assume that |Ax| << so that the remain constant. Solution 2.12 ‘The equivalent spring constants for the configurations shown are -(—_ kth kg=[tet ye ‘Then the equivalent system is Upon moving the ends of the equivalent springs an amount x in the horizontal direction, the displacement of each spring is 8x3 = xC080, (compression) 6, =xcos(7—0,) (extension) 5, =xc0s(0,—7) (extension) The corresponding force in each spring is 17 (©2009 Cengage Learning. ALL RIGHTS RESERVED. Solutions to Exercises ~ Chapter 2 Fin= hb Fakd, Foo = ks so Then the sum of the forces in the x-direction is F = F,;,00s0, + F,cos( - 0,)+F,, cos(, —) 120 ps 6088, + k,5, COS — O,)+k Sigg C08(0, — 7) = [kins 008" 8, +k, 008" (7 —0,)+k, c05°(0,— 7) |x Rewriting this result as Fokx wwe find that the equivalent stiffness of the spring system is k, 2s C08" 8, + k, €05"( — 0, )+ky, COS*(0,— 2) If the intersection point of the springs is not constrained; that is, confined to a slot to move in the x direction only, then we have an additional constraint on the spring system. This constraint is obtained by setting the sum of the forces in the y-direction to zero. Thus, we have —F,8in8, + F,sin(x ~0,)— F,,sin(@, — x) =0 Upon using the force displacement relations gives, ~hinsBy Sin, + kB, sin( 0.) ~ ky Sin(O, ~ 2) = After substituting for the deflections in terms x, we obtain ky, 6080, sind, + k, cos( st ~0,) sin(zt —0,) — ky, cos(O, ~)sin(@,— 7) =0 2.13 Consider the system shown Figure £2.13, which lies in the X-Y plane. This system, which is called a crab-leg flexure, is used in microelectromechanical sensors. A load along the X direction is applied to the mass to which all of the four crab-leg flexures are attached. Each flexure has a shin of length Z along the X direction, width b along the ¥ direction, and thickness / along the Z direction. ‘The thigh of each flexure has a length L, along the ¥ direction, a width 6, along the X direction, and a thickness along the Z direction, ‘The equivalent stiffness of each of these flexures in the be direction can be shown to 2 (L+1,(6/6)) where the Young's modulus of elasticity is E, which has a value of 160 GPa for the polysilicon material from which the flexure is fabricated. The dimensions of each crab 18 (©2009 Cengage Learning. ALL RIGHTS RESERVED. Solutions to Exercises ~ Chapter 2 leg are as follows: L = 100 um and b = 6, =h=2 ym. For the thigh length Z, spanning the range from 10 um to 75 ym, plot the graph of the equivalent stiffness of the system versus 1, Solution 2.13 ‘The above equation is plotted in the figure below. 10; 95} | 3 2 88) <8 3 75| 7 65) a a a a a er) tm Figure £2.13 ‘MATLAB program used to generate figure for Exercise 2.13. Lt-linspace(10e-6,75e-6,100); Kflex=B*h*b3*(4*L+L1*(b/b0)°3) (L+LA*(bPY 3YL/; plot(10°6*L flex.) set(gca,fontsize'14) label(L_t(\mum,fontsiz’14) ylabel(k_{flexure} (N/m)'*fontsize’,14) axis({10 75 6 10)) 2.14 Based on the expression for knee provided in Exercise 2.13, the sensitivity of the equivalent stiffiness of each flexure with respect to the flexure width b and the thigh width », can be assessed by determining the derivatives dinexedb and dknccyd/db, respectively. Carry out these operations and discuss the expressions obtained, Solution 2.14 We rewrite the expression in Exercise 2.13 as 19. ©2009 Cengage Learning. ALL RIGHTS RESERVED. Solutions to Exercises — Chapter 2 ee renee) pom) where derivative dly Differentiating this equation with respect to b and simplifying, we obtain Hk pae ay, (44215 (8/6) +B (6/0)') yr (non = = We see that when L,(b/b) 01 the sensitivity becomes Kram, 3920, db ‘Thus, this sensitivity in this region is most affected by Z, followed by b. When L,(b/b) 01 sensitivity becomes eae 126°C, eer which is four times more sensitive as the previous limiting case. Sensitivity derivative dkigeureldby Differentiating the top equation with respect to b, and simplifying, we obtain Acre £: (o/b) db, 11, (l4L(6/0)) We see that when 1, (6/6) 0.1 the sensitivity becomes A penne, 907°C, _ 9Eh( b, db, L UAL 20 ©2009 Cengage Learning. ALL RIGHTS RESERVED. Solutions to Exercises ~ Chapter 2 which is independent of b. The sensitivity in this region is most affected by the ratio by. 2.15 Find the torsion-spring constant ke of the stepped shaft shown in Figure E2.15, where each shaft has the same shear modulus G. Determine the equivalent spring length of a shaft of constant diameter D, and length L that has the same spring constant as the torsion spring shown in Figure E2.15. lution 2.15, Making use of Eq. (2.18) for three torsion springs in series and Table 2.3 to compute the torsion stiffness, we obtain where ‘Thus, (224, 32h, 32h, aGD; GD! xGDi @) G/L ,h 5 32\ Di” Di” DF Fora shaft of uniform stiffness, we have 4, = Ge 2D! ey eamea2 Ts Upon equating Eqs. (a) and (b), we obtain #G/ L, 32 (DF which leads to Section 2.3.3 2.16 Consider the two nonlinear springs in parallel shown in Figure E2.16. The force- displacement relations for each spring are, respectively, F,G@)= yetkax f=1,2 21 (©2009 Cengage Learning. ALL RIGHTS pSERVED, Solutions to Exercises ~ Chapter 2 a) Obtain the expressions from which the equivalent spring constant can be determined. b) If F = 1000 N, ki = k = 50,000 N/m, and @ = 2 m, determine the equivalent spring constant. Solution 2.16 (a) The linear equivalent stiffness of each spring around an operating point x = x9 is ak, Sy a (k,+3kax? =1,2 al, (k,+3k,ax4 Des i =(k, +3kax2) f=h2 Since these are springs in parallel, the equivalent stiffness coefficient k, is given by = (hy +h) +3(k, +k Jaxd b) When ky = ky = k= SOkN/m, @= 2 m'', and = 1000 N, the total force acting on the system is Foi +P, =2ke+2kax’ where F and are the forces acting on springs I and 2, respectively. For the given value of the force, the static deflection in each spring x» is found by solving the equation 1000 = 2x 50x10" x x +2 50x10? x2x.x3 or 200x3 +100x, Using the MATLAB function root's yields the real root xp = 0.01 m. Making use of the results from Part a), the linear equivalent stifiiness is k= (ky +h) + Hk, + ax; = 2k + boxy =2x50x10° +6%50%10° x2x 0.01? = 100060 N/m 2.17 Consider the two nonlinear springs in series shown in Figure E2.17. The force- displacement relations for each spring are, respectively, E,Q)=kxtkax 7 =1,2 a) Obiain the expressions from which the equivalent spring constant can be determined. b) If F = 1000 N, &, = 50,000 N/m, kz = 25,000 Nim, and @ = 2 m°, determine the equivalent spring constant. Solution 2.17 22 (©2009 Cengage Learning. ALL RIGHTS RESERVED. Solutions to Exercises — Chapter 2 (a) At the operating point, let the static deflection in the nonlinear spring 1 be xi, and the static deflection in the nonlinear spring 2 be xz. Then, the respective equivalent stiffness of the springs 1 and 2 are given by 4, | yaw =k, +3kax'| =k, +3kax,, j=L2 Since the two springs are in series, the equivalent stiffness coefficient & is given by )' hake Kathe 1 K, (b) For , k; = 50,000 Nim, ko = 25,000 Nim, @=2 m?, we first determine the static deflections in the nonlinear springs. To this end, we note that F(x)=F=k,x,+k ax, j=12 ‘Thus, we obtain the equilibrium equations 100x}, +50x,, -1=0 50x}, +25x,,-1=0 By using the MATLAB function root's on each of these equations, we find that i= 0.02 m and x,,=0.04 m Making use of Part (a), we arrive at ik, = 50x 10° +3x50x10° x2 x 0,02? = $0.12 kN/m ky, =2510! +3x25%10? x2x0.04? = 25.24 kNim and the equivalent stiffiness is 50,12 25.24 = 50.12+25.24 107865 Nim 2.18 Consider the data in Table E2.18 in which the experimentally determined tire loads versus tire deflections have been recorded. ‘These data are for a set of dual tires and a single wide-base tire, The inflation pressure for all tires is 724 KN/m?. Examine the stiffiness characteristics of the two different tire systems and discuss them. Solution 2.18 For the data in the table, we use MATLAB to plot these results, which are shown below. We see that the wide-base tires are less stiff compared to that of the dual tires and their stiffness is almost linear over a wide deflection range. The dual tire is stiffer than the wide-base tire and its stiffness characteristic is discernibly nonlinear. Furthermore, if we fit a second-order polynomial to each of the curves using the MATLAB function Lsqcurvefit, we find that the dual tire relation is 23 ©2009 Cengage Learning. ALL RIGHTS RESERVED. Solutions to Exercises ~ Chapter 2 Fgt-are =81.85-+968.725 421.86" N and that for the wide-base tire is Fete = 149.84 + 784.185 +6325? N where dis the deflection and the coefficient multiplying &* represents the strength of the nonlinearity. We see from the coefficients of these fitted equations that the dual-tire nonlinear coefficient is 3.4 times larger than that for the wide-base tire. asp S 0 S(O CS 080M Deflection (mm) Figure £2.18 MATLAB program used to generate figure for Exercise 2.18 fac=4.44822; % N/Ib_f mm=25.4; % mm/in Fefac*[0 2000 4000 6000 8000 10000]; %Table data in N DDT=mm*{0 0.30 0.55 0.75 0.95 1.10]; % Table data in mm, DWB=mm*[0 0.40 0.75 1.10 1.40 1.65]; % Table data in mm plot(DDT,F ke set(b(1), fonts xlabel(Defleetion (mm), fontsize’,14) ylabel(Force (N);fontsize’14) set(gca,fontsize' 14) ‘opt=optimset(‘display'fof?); fit-inline(x(1)#x(2)*q+x3)*q.°2%') xdt=Isqourveit(it[.1 1000 100), DDT.F,(),0,0p0; dlisp(CF =" num2strxdt(1))* +" num2strxd(2)) d+ num2str(xat(3)) 8-2") xwb-Isqourvefit(fit.1 1000 100, DWB,F,[]{.op0); disp((F =" num2str(xwb(1)) +" num2str(xwb(2)) 'd + num2ste(xwb(3)) "42" 24 (©2009 Cengage Leaming. ALL RIGHTS RESERVED. Solutions to Exercises ~ Chapter 2 Section 2.3.4 2.19 Consider the manometer shown in Figure 2.16 and seal the ends. Assume that the initial gas pressure of the sealed system is P, and that L, is the initial length of the cavity. Determine the equivalent spring constant of the system when the column of liquid undergoes “small” displacements, Solution 2.19 When the top is sealed, it becomes a piston. Since the energy storage devices undergo equal displacements, the springs are in parallel, and from Eqs. (2.27) and (2.34b), we obtain 284, + a 2.20 Consider “small” amplitude angular oscillations of the pendulum shown in Figure £2.20. Considering the gravitational loading and the torsion spring &; at the pivot point, determine the expression for the system’s equivalent spring constant. The masses are held with rigid, weightless rods for the loading shown, Solution 2.20 In this case, we generate the system’s potential energy and identify the equivalent spring constant ke. V(O)=Vay +Vng +V5, eo Flip =—m,gb—+ mga +260 mg + mga + =F Lh,+mga—m,g6]0° Therefore, k, =k, +mga—m,gb Section 2.4.1 2.21 Determine the equivalent damping of the system shown in Figure £2.21 Solution 2.21 The dampers ¢} and c2 are in parallel; therefore, the equivalent damper is c12 = ¢ + €2. Noting the force location, equivalent damper c12 and damper c3 are in parallel. Thus, the equivalent damper is ¢¢ = ¢12 +63 = ¢1 + ¢2 + ¢5, 2.22 Determine the equivalent damping for the system shown in Figure E2.22. Solution 25 (©2009 Cengage Learning. ALL RIGHTS RESERVED. Solutions to Exercises ~ Chapter 2 Since cy and cp are in series, the equivalent damping of this series combination is and, therefore, the equivalent damping of the system is cg = e12 + ¢3 2.23 Representative damping-force magnitudes versus speed data are given in Table 2.23 for a racecar damper in compression. Examine these data and discuss the type of ‘damping model that can be used to represent them. Solution 2.23 ‘The data are plotted below. It is seen that the damping model will require a nonlinear representation of a viscous damper consisting of three linear pieces. ‘The three linear ranges run from v= 0 to 7.6 mm/s; v= 7.6 to 15.2 mmv/s; and v= 15.2 to 40.6 mm/s. ee a ee) Velocity (mms) Figure £2.23 MATLAB program used to generate ewe for Exeeie 223 fuc4.44822; 96 Nib_f mm-25.4; % main fd-fact[0 1326 40 60 80 100 110 120 130 140 150 160 170 180 190 200}; vd-mm"(00.1 0.20.3 0405 0607080910 1.11213 1415 1.6; plot(vd,fd, ‘ks ‘set(gea, ‘fontsize’,14) abel( Velocity (mv, ontsize'14) | ylubel(Force CN)", "Yontsiz' 14) 26 (©2009 Cengage Learning. ALL RIGHTS RESERVED. Solutions to Exercises ~ Chapter 2 2.24 Determine the viscosity of the fluid that one needs to use to realize a parallel-plate damper when the top plate has an area of 0.02 m’, the gap between the parallel plates / = 0.2 mm, and the required damping coefficient is 20 N-s/m. Solution 2.24 From Eq. (2.50), we find that ch _ 20. Ns'm)(2x10* m) 4 0.02 m? 2 Ns/in® 2.25 Determine the equivalent damping coefficient for the following nonlinear damper FR) =¢,¢+0,%" where c; = 5 N-s/m and c; = 0.6 N-s'/m’. Note: the damper is to be operated around a speed of 5 mvs. Solution 2.25 Using Eq. (2.47), we find that ae =, +3eyi2 =543x0.6x5?=50 Nem 2.26 Represent the vibratory system given in Figure E2.26 as an equivalent vibratory system with mass m, equivalent stiffness k., and equivalent damping coefficient c. Solution 2.26 Since the springs and dampers are in parallel, the equivalent damping coefficient is = c1 +2 and the equivalent spring stiffiness is ke ~ ki + ho. 2.27 Represent the vibratory system given in Figure E2.27 as an equivalent vibratory system with mass m, equivalent stiffness k., and equivalent damping coefficient c. Solution 2.27 ‘The dampers are in parallel; therefore, the equivalent damping coefficient is ce = ci + 2 +¢3. The springs kp and 4; are in series and the equivalent stiffness of this series combination is, 7 1 1 iy=| D4 (e+) ‘The springs k; and fo3 are in parallel; therefore, the equivalent spring stiffiness is ke = ki + has. 2.28 The vibrations of a system with stiffness & and damping coefficient ¢ is of the form x(0) = X,sinex. This type of response, which is called a harmonic response, is possible 27 (©2009 Cengage Leaming. ALL RIGHTS RESERVED. Solutions to Exercises ~ Chapter 2 when a vibratory system is excited by a harmonic force. Evaluate the work done by the spring force and the work done by the damper force, which are given by the integrals s0x{0) alo E,= | kede= | bot de 20 a safe fore 1) a Solution 2.28 The work done by the spring force is E,= | keidt= [ k(X, sin(on)(X,ocos(wt))ar a 3 ale =koX? f sin(wt)cos(or)dt = KX? fsingcosede = ax:|Lsin'o] 0 ‘The work done by the damper force is E,= f cvtar= "T (X.ecos(on))'dt cx? “f c0s"(ony 2 Ecos’ aol. ie =coX: = coX2| 2 41 sin(2 cox? Jeos (pxd9 = coX? [g +f sin ~| =conX? 2.29 For the system of Exercise 2.28, assume that k = 1000 N/m, c = 2500 N/(m/s), Xo = 0.1 m, and @= 9 rad/s. Plot the graph of the sum of the spring force and damper force versus the displacement; that is, kx-+o% versus x. Solution 2.29 ‘The sum of the spring and damper force is ec + c¥ = 1000 0.1sin(91) +2500%9 x 0.1e0s(91) =100sin(91) +2250cos(91) The result is plotted below as a function of x = 0.1sin(91). It can be seen that the graph has the shape of an ellipse. 28 ©2009 Cengage Learning. ALL RIGHTS RESERVED. Solutions te Exercises ~ Chapter 2 ‘linspace(0.2*pi9, 100); y=100*sin(9*t)+2250*e0s(9*1); x-0.1%sin(9*); plox(syk-) set(gca fontsize'14) ylabel(kxrev; fontsize’14) xlabel(x'fontsize14) 2.30 For the system of Exercise 2.28, assume that k > 0, ¢ > 0, and «> 0. Show that the graph of kx+c% versus x will have the form of an ellipse. Solution 2.30 Let F(x,3)= kx tek Then, if'x = X,sinor, F(x,8) =k + c@X, cost which can be written as F(x,3) =k tcoX,Vi—sin? of and rearranged to obtain (F(,%)-kx' =e? (X2-x*) This leads to the equation 29 (©2009 Cengage Learning. ALL RIGHTS RESERVED. Solutions to Exercises ~ Chapter 2 F(x,3)+( +00”)? -2keF(x,i)— C0, which is the equation of an ellipse in the F(x, #)-r plane. Section 2.4.2 2.31 Consider the viscous-damping model given by Eq. (2.46) and the dry-friction model given by Eq. (2.52). Sketch the force versus velocity graphs in each case for the following parameter values: ¢ = 100 Nimm/s), m= 100 kg, and = 0.1 Solution 2.31 Equation (2.46) gives and Eq, (2.52) yields FQ) = amg sen() = ).1 100 x 9.8sgn(#) = 98sgn(%) ‘These are plotted below. Fo) 200 Foo 100 on 1 ee 12 Fm) Fw) -98 #3. (246) F052) Figure £2.31 2.32 Normalize the linear viscous-damper force given by Eq. (2.46) using the damping coefficient c, the dry-friction force given by Eq. (2.52) using pg, the fluid-damping force given by Eq. (2.54) using the damping coefficient cg, and the hysteretic force given by Eq. (2.57) using Arp. Plot the time histories of the normalized damper forces versus time for harmonic displacement of the form x(/) = 0.4sin (2nt) m. Discuss the characteristics of these plots. Solution 2.32 From Eq. (2.46), we have the viscous foree $= 087 cos(2zt) From Eq. (2.52), we have the dry friction fore 30 ©2009 Cengage Learning. ALL RIGHTS RESERVED. Solutions to Exercises ~ Chapter 2 Fe) sgn(%) =sgn (cos(2m)) ung From Eq. (2.54), we have the fluid force FC) 4 5) = (1.62) |ecos(21) |eos(221) oe From Eq, (2.57), we have the hysteretic force FE) _ son(i)|.x|=0.4sen (cos(221))|sin(220)] kf, These results are plotted below. ‘The normalized viscous damping force versus time plot shows that this force varies harmonically at the frequency @ = 2 rad/s. In all the other cases, the resp. normalized damping forces do not vary harmonically and they contain additional frequency components. In these cases, the damping forces are nonlinear. cous ory ton ‘ 7 oe ea B os) 2 3 go 2 a bos L 4 49 2 4 oO 2 4 ru ttre oa Fecxie) ‘MATLAB program used to generate figure for Exercise 2.32 ‘inspace(0,4,100); subplor@2,2,1) plot(,0.8*pitcos(2*pi*t), ylabel(F(dx/dtV/c',"fontsize’,14) itlecViscous''fontsiz’, 14) ‘set(gea,'fontsize',14) subplot(2,2,2) ot(,sizn(cos(2*pi" 31 (©2009 Cengage Learning. ALL RIGHTS RESERVED. Solutions to Exercises ~ Chapter 2 ‘ylabel( F(dx/di) (mama) fontsize 14) ve@)=L.1*V(4); axis(v) title(Dry friction’ fontsize’,14) set(gca, fontsize’ 14) subplot(2,2,3) plot(,(1.6*piy’2*abs(cos(2*pi*t).*eos(2*pittk-) yylabel(F(dwatye_d);'fontsize’,14) label(t(5);'fontsize’ 14) title( Fluid’, Yontsize’ 14) set(gca, fontsize’,14) subplot(2,2,4) plot(,0.4*sign(cos(2*pi*)).*abs(sin(2*pi*),k-) ‘ylabeiCF(dw/at)(Kipilbeta_), fontsize’14) title(Hysteetic’fontsize'4) xlabel(t (5) fontsize 14) set(gea,fontsize’14) _ 2.33 Construct vibratory models for each of the three systems shown in the Figure E2.33. Discuss the number of degrees of freedom and the associated generalized coordinates in each case. Solution 2.33 ) One possible car model: If we ignore the mass of the tires and the elasticity of the car, then we have a ‘rigid’ platform that can ‘* rotate about the long axis of the car (left to right) ‘© rotate about the short axis of the car (front to back) © translate vertically Therefore, this model of the car has 3 degrees of freedom with the three generalized coordinates corresponding to the two rotations and the vertical translation, b) One possible motorcycle model: If we ignore the mass of the tires and the elasticity of the motorcycle, then we have a “rigid” frame that can * rotate about the short axis of the motorcycle (front to back) © translate vertically ‘Therefore, this model of the motorcycle has 2 degrees of freedom with the generalized coordinates being the rotation coordinate and the translation coordinate. ©) One possible model of the cable car: ‘translate vertically * rotate (swing) in the vertical plane ‘Therefore, this model of the car has 2 degrees of freedom with the generalized coordinates being a rotation coordinate and a vertical translation coordinate. 2.34 A vibratory system has a mass m = 10 kg, k = 1500 N/m, and c= 2500 Ni(m/s). Given that the displacement response has the form x(t) = 0.2sin(9/) m, plot the graphs of é, the spring force kx, and the damper force ct versus time and discuss them. 32 ©2009 Cengage Learning. ALL RIGHTS RESERVED. Solutions te Exercises ~ Chapter 2 Solution 2.34 The three forces are, respectively, mi = —10 9 x0.2sin(91) =-162sin(91)_N ci =2500%9x0.2sin(91) = 4500c0s(91) N ex = 1500 0.2sin(92) = 300sin(9") N Each force component varies sinusoidally at the same frequency, but the damping force is 90° out of phase with the spring force and the inertial force is 180° out of phase with the spring force. The results are shown below. Force (N) 1 @) Figure £2.34 [LAB used fo generate figure for Exereise 2.34 ‘-linspace(0,2,100); sesin(9*); plot(300*5,k-t4500*c0s(9*1)k~ 3-162" xlabel(tfontsize’14) ylabel(Force (N)'fontsize’4) {abj-legend¢Stifiness' Damping’ Inertia’ set(b(1),fontsize’,14) set(gca,fontsize’ 14) 0 2},{0 0k 33 (©2009 Cengage Learning, ALL RIGHTS RESERVED. Solutions to Exercises — Chapter 2 34 ye Learning. ALL RIGHTS RESERVED. Solutions to Exercises ~ Chapter 3 Solutions to Exercises Chapter 3 Section 3.2.1 3.1 Rewrite the second-order system given by Eq. (3.8) as a system of two first-order differential equations by introducing the new variables x, =x and x, =. The resulting system of equations is said to be in state-space form, a useful form for numerically determining the solutions of vibrating systems. Solution 3.1 We note that and, therefore, a dt Upon substituting these equations into the governing equation ax de m—>te—+ke= f(t ae eg BLO we find that dx, m ex, +h, = SO dt Thus, the two first order equations are de, 3.2 A vibratory system with a hardening nonlinear spring is governed by the following equation mitck+k(xtax’)=0 Determine the static-equilibrium position of this system for a = 1 and linearize the system for “small” oscillations about the system static-equilibrium position, 35 (©2009 Cengage Learning. ALL RIGHTS RESERVED. Solutions to Exercises ~ Chapter 3 Solution 3.2 To determine the static equilibrium positions, we set the velocity and acceleration terms to zero. Thus, the static equilibrium positions are determined by the roots of the polynomial K(x, +x3)=0 or +x,=0 Xn tty This cubic polynomial has one real-valued root, which is xy = 0. ‘The linearized system describing small oscillations about the equilibrium position x = has the form mi+ck+k(143x2)x=0 For the equilibrium position x= 0, the linearized system is mi +ck+ke=0 3.3 A vibratory system with a softening nonlinear spring is governed by the following equation mitt cit Kx-ax” Determine the static-equilibrium positions of this system for a = 1 and linearize the system for “small” oscillations about each of the system static-equilibrium positions. Solution 3.3 To determine the static equilibrium positions, we set the velocity and acceleration terms to zero. ‘Thus, the static equilibrium positions are determined by the roots of the polynomial or whose roots are ‘The linearized system describing small oscillations about an equilibrium x = x has the form mi + ct-+k(1—3x),)x=0 ion x= 0, the linearized system is mi+ci+ke=0 36 ©2009 Cengage Learning. ALL RIGHTS RESERVED. Solutions to Exercises ~ Chapter 3 For the equilibrium position x4,= +1, the linearized system takes the form mit+ci—2ke =0 3.4 Determine the equation governing the system studied in Example 3.13 by carrying outa force balance, Solution 3.4 From the free-body diagram shown at the right, we carry out a force balance in the vertical direction to obtain m (6, + AL) Le mit+kx+k,(5, +AL)eosO=0 5, Noting that k VE+8-L and coso=—— a wwe find the equation of motion to be ik kyx +h, (5,/L+y ise] - x Vee /E For x/L| << 1, Ji+x°/E ~1 and this equation of motion becomes mi+(k,+6,k,/L)x=0 Since 1d, = 7, we can rewrite the equation of motion as mi+(k,+7/L)x=0 3.5 A mass m is attached to the free end of a thin cantilever beam of length L, as shown in Figure E3.5. The fixed end of this beam is attached to a shaft of radius r that is rotating about its axis at a speed of Q rad/s. Assume that the mass of the bear is negligible and its equivalent stiffness is ks, Derive the governing equation of motion for transverse vibrations of the beam in terms of the variable x and obtain an expression for the system’s natural frequency. Solution 3.5 ‘The centrifugal force in the radial direction is F=(L+ayme Let @be the angle formed by the cantilever beam when the mass m displaces an amount x in a transverse direction to the beam from the static equilibrium position. For small x displacement, P= Fsin0~ FO 37 (©2009 Cengage Learning. ALL RIGHTS RESERVED. Solutions to Exercises ~ Chapter 3 where F; is the component of the force acting in the x-direction. In addition, since 0 is small, x = Ltan@ and therefore, 9 ~ x/L. Thus, the component of force in the x-direction due to the rotation is =(L+a)mE (2) =(I+a/L)mQ%x A summation of forces in the x-direction gives m+ k,x+(14+a/L)mQ?x=0 m+ (kyx-+(I+a/L)mQ?)x=0 where, from Table 2.3, the stiffness of the cantilever beam is where E is the Young’s modulus of the beam material and is the area moment of inertia about the bending axis, Section 3.2.2 3.6 Derive the governing equation of motion for the rocker-arm valve assembly shown in Figure E3.6. Assume “small” motions. The quantity JJ, is the mass moment of inertia about point 0 of the rocker arm of length (a + b), kis the stiffness of the linear spring that is fixed at one end, and M is the external moment imposed by the cam on the system. ‘This moment is produced by the contact force generated by the cam at the end of the rocker arm. Sol a. For ‘small’ rotation @ of the rocker arm, the displacement of the mass m is x= 60, If we ignore the effects of gravity, then the system kinetic energy, system potential energy, dissipation function, and generalized force are, respectively, where k is the unit vector normal to the plane of the system. By using the Lagrange equation fai (eta | eta Neb aa at\36) 00° 36° 36 ~~? 38 (©2009 Cengage Learning. ALL RIGHTS RESERVED. Solutions to Exercises ~ Chapter 3 we arrive at “u. +mb*)6)-0+0-+kb°0 =M (J, +b?) 6 + kb°0 = M 3.7 Derive the governing equation of motion for the system shown in Figure E3.7. The ‘mass moment of inertia of the bar about the point O is Jo, and the torsion stiffiness of the spring attached to the pivot point is k,. Assume that there is gravity loading. Solution 3.7 The kinetic energy, potential energy, respectively, pation function, and generalized force are, SF 2 1 2 where the datum is chosen to be at the bottom of the bar. The Lagrange equation d(ar) av £(5|45—=0 a6)” 00 v ke +g Zc0s0 is used to obtain 454 Le G9) kO—mg sind =0 or 1.0 +k0-mgsind=0 For “small” oscillations about the equilibrium position 0 Joh ~me}0=0 3.8 Determine the equation governing the system studied in Example 3.15 by carrying out a force balan Solution 3.8 39 (©2009 Cengage Learning. ALL RIGHTS RESERVED. Solutions to Exercises ~ Chapter 3 From the free-body diagram shown at the right, we carry out a moment balance about the point O to obtain Lng b IG+ KP + Cykyl, COS ing+ +l, 2 Limg Ly L+h, 2 Considering ‘small’ angular oscillations about the vertical position, we note that ing +[mi+ k(x, -x,)]L,cosp=0 cosp=l, sing=g, x,~Lp, and x,~Lp Then the equation of motion can be written as [Z+mei ore tion] hr kts + ®8(L,-2) Johar Section 3.3.1 3.9 A cylindrical buoy with a radius of 1.5 m and a mass of 1000 kg floats in salt water (p = 1026 kg/m’). Determine the natural frequency of this system. Solution 3.9 From Eq, (2.27), we see that the equivalent stiffness due to the buoyancy force is k, = pgd=1026x98x1x1.5' =71,073 Nim Then, the natural frequency is 2, -[&- 71073 _ 3.43 rad/s mY 1000 3.10 A 10 kg instrument is to be mounted at the end of a cantilever arm of annular cross section. The arm has a Young’s modulus of elasticity E = 72 x 10° N/m? and a mass density = 2800 kg/m*. If this arm is 500 mm long, determine the cross-section dimensions of the arm so that the first natural frequency of the system is above 50 Hz. Solution 3.10 From Table 2.3, the stiffiness of cantilever subjected to a transverse force at its free end is Solutions to Exercises — Chapter 3 Noting that the natural frequency is 1(3E a) 4 foal alm Dae) we obtain from this equation 3nE For fy = 50 Hz and the given parameter values, we find that the cross-section dimensions should be such that 4x10x0.5' x(2a x50)" Bax 72x10” In order for f, to be greater than 50 Hz, the cross-section dimensions should be such that ‘ ‘ % 7.272%107 m’ "i riar'>7272x107 m* 3.11 The static displacement of a system with a motor weight of 385.6 kg is found to be 0.0254 mm. Determine the natural frequency of vertical vibrations of this system. Solution 3.11 o,= 2 = | 98-2115 dis "V5, V254x107 a, From Eq. (3.15) Sy=SE= 98.86 Hz 2a 3.12 A rotor is attached to one end of a shaft that is fixed at the other end. Let the rotary inertia of the rotor be Jc, and assume that the rotary inertia of the shaft is negligible compared to that of the rotor. ‘The shaft has a diameter d, a length L, and it is made from material with a shear modulus G. Determine an expression for the netural frequency of torsional oscillations. Solution 3.12 From Table 2.3 Gd 32L Therefore, 41 ©2009 Cengage Learning, ALL RIGHTS RESERVED. Solutions to Exercises ~ Chapter 3 o,= [= [2 -£ | aais Jo V32JgL 4 V2 an expression for the natural frequency for the system shown in Figure E3.5. 3.13 Obtail Solution 3.13 From the equation of motion determined in the solution for Exercise 3.5, the natural frequency is foe | m 1 o where ky @, = + m 3.14 Consider the hand motion discussed in Example 3.3 and let the hand move in the horizontal plane; that is, the gravity force acts normal to this plane. Assume that the length of the forearm / is 25 cm, the mass of the fore arm m is 1.5 kg, the object being carried in the hand has a mass M = 5 kg, the constant ky associated with the restoring force of the biceps is 2 x 10° N/rad, the constant K, associated with the triceps is 2 x 10° N/rad, and the spacing a= 4 cm. Determine the equation of motion of this system, and from this governing equation, find the natural frequency and damping factor of the system, Solution 3.14 Since the gravity loading acts normal to the plane, Eq. (¢) of Example 3.3 reduces to (™ + Bros K,a’6+ kad =0 ‘Then the natural frequency can be determined as o- fe ha _ [2x10 008 is o6 sadly "Vim, \(M +mj3)P — \(5+1.5/3)025 and the damping factor can be determined as Ka _ 2x10" x0.04 2@,(M +m/3)P — 2x15.26x(5+1.5/3)x0.25° 305 3.15 A spring clongates 2.5 mm when stretched by a force of 5 N. Determine the static deflection and the period of vibration if a mass of 8 kg is attached to the spring. 42 ©2009 Cengage Learning. ALL RIGHTS RESERVED. Solutions to Exercises ~ Chapter 3 Solution 3.15 The spring constant can be determined as -F-_5_. 5 0.0025 From Eq. (3.6), the static displacement for an 8 kg load is N/m _ mg _ 8x98 we 2000 =n, ft 22 k 2000 3.16 Determine the natural frequency of the steel disk with torsion spring shown in Figure £3.16 when k= 0.488 N-m/rad, d= 50 mm, puss = 7850 kg/m’, and = 2 mm. Solution 3.16 ‘The mass of the disk is m= Fd’ hPan = 4£(7580)(0.05)" (0.002) = 0.02977 kg =39.2 mm 397 s and it mass moment inertia is 1 Jo = gma? = 5(0 02977)(0.05)’ =9.3x10~ kg-m? Then, the natural frequency is 1 fk 1 [0.488 I ae Jo iE + 3.17 Consider a nonlinear spring that is governed by the force-displacement relationship vol) where a = 3000 N, b = 0.015 m, and c= 2.80. If this spring is to be used as a mounting for different machinery systems, obtain a graph similar to that shown in Figure 3.5b and discuss how the natural frequency of this system changes with the weight of the machinery. Solution 3.17 A plot of the equation with the values indicated is given below. The horizontal dashed lines represent a deviation of + 8.7 % about the solid horizontal line. We see that the range of natural frequencies is lower than that shown in Figure 3.5b and the range of Wis slightly wider. 4B ©2009 Cengage Learning. ALL RIGHTS RESERVED. Solutions to Exercises ~ Chapter 3 eee arene een eee eee eee 820 1a00 1800 2000 2509, 3000 3600 40004500 ‘son wo Figure £3.17 ‘MATLAB program that generated figure for Exercise 3.17 firinline'sqr(9.8%e/b)*(W/a).%-1/(24e) (2*pi) W201; 33000; b=0.015; e-2.8; ‘Welinspace(500,5000); plot(W, fngWa,b.0)k-) Avg tn(1500,3,b,0)*4n(4000,a,b,0))2; sdel=(En(1500,3,6,0)-£1(4000,3,b,0))2; vatdel; ‘Avg-del; disp(deV/Avg*100) hold on plot([500 5000},[thfh~,[500 S000}, 1] k~:{500 5000},(Avg vel) set(gca,'fontsize’14) ylabel('n (H)"fontsize’14) label¢W (N),'fontsize’ 14) vax ¥(1)=500; axis(v) 3.18 The static deflection in the tibia bone of a 120 kg person standing upright is found to be 25 jum. Determine the associated natural frequency of axial vibrations. Solution ‘The relation for the natural frequency is given by Eq. (3.15). Thus, 44 (©2009 Cengage Learning. ALL RIGHTS RESERVED. Solutions to Exercises ~ Chapter 3 [x — | 98 _ 696.1 raais 3, V25x10% 3.19 A solid wooden cylinder of radius r, height h, and specific gravity sis placed in a container of tap water such that the axis of the cylinder is perpendicular to the surface of the water. Assume that the density of the water is py... It is assumed the wooden cylinder stays upright under small oscillations. a) If the cylinder is displaced a small amount, then determine an expression for its natural frequency. b) If the tap water is replaced by salt water with specific gravity of 1.2, then determine whether the natural frequency of the wooden cylinder increases or decreases and by what percentage. Solution 3.19 From Eq, (2.27), we see that the equivalent stiffness due to the buoyaney force is k= BP yom? ‘The mass of the eylinder is Mm, =S,PyptPh = fen [Benet _ [a Nm. Y8PnpAh Vs Ji 12gpyo7? 7 Ong = he = ft 28P no - [28 «0950, m, \SPnotPh \ s,h ‘Therefore, the natural frequency in salt water inereases by 9.5 %. a) For tap water, b) For salt water, ‘Then 3.20 Consider the pulley system shown in Figure E3.20, The mass of each pulley is small compared with the mass m and, therefore, can be ignored. Furthermore, the cord holding the mass is inextensible and has negligible mass, Obtain an expression for the natural frequency of the system. Solution 3. The two massless pulleys and the inextensible cord assumptions permit us to determine an equivalent spring constant k. If the force in the cord is F, then the 45 (©2009 Cengage Learning, ALL RIGHTS RESERVED. Solutions to Exercises ~ Chapter 3 displacements in the left and right springs with spring constan's k, and kp are, respectively, 2F 2F Ee ae If the total displacement of the cord is x, then, because we have a pulley system, x=2x, +2, Therefore, upon substitution, Then, the natural frequency is 3.21 A rectangular block of mass m rests on a stationary half-cylinder, as shown in Figure 3.21. Find the natural frequency of the block when it undergoes small oscillations about the point of contact with the cylinder. Solution 3.21 From the figure, we see that the ce length ¢ + b = 0, ¢ = (h/2)tan8, and a=beos@. Then, a=(r0—(h/2)tan8)cos0 =rOcos6 —(h/2)sind For “small” @ about @ = 0, this equation reduces to ax(r-h/2)0 dee Let O be the pivot point as shown in the figure. Then, carrying out 2 moment balance about the pivot point, we obtain JoG+mga=0 or Job+me(r—h/2)0=0 where for “small” angular oscillations about @=0, we have 46 ©2009 Cengage Learning, ALL RIGHTS RESERVED. Solutions to Exercises ~ Chapter 3 mi? 2 (hy EB (RY me | (ro) +{4) ]~m| £4(# 12 onl ) (3) of E(2)] ‘Then the natural frequency is mg(r-h/2) g(r=h/2) 2/12+(W/2)') (EA2+(H/ )’) Section 3.3.2 3.22 Formulate a design guideline for Example 3.8 that would enable a vibratory system designer to decrease the static deflection by a factor n while holding the damping ratio and damping coefficient constant. Solution 3.22 ‘The approach is based on what is illustrated in Example 3.7. From Eq. (3.6), it is known that There is only one choice that would allow us to keep the damping ratio and the damping coefficient unchanged since 2k that is, the product mk must remain constant. Thus, m and k are changed by the same factor; that is, m m>* and kak vn For this selection, the static deflection is decreased by a factor of n. 3.23 Formulate a design guideline for Example 3.8 that would enable a vibratory system designer to decrease the static deflection by a factor » while keeping the damping ratio and mass m constant, Solution 3.23 Since and m is a constant, then k must be increased by k-onk 47 (©2009 Cengage Learning, ALL RIGHTS RESERVED. Solutions to Exercises ~ Chapter 3 to decrease the static deflection by a factor of n. Since the damping ratio must be held constant, then pee (Ogee cet ee cee (ne Img” 2m\nkg 2m Vink jent must be increased by a factor of Vn ; that is, coevn Therefore, the damping coe! 3.24 An instrument’s needle indicator has a rotary inertia of 1.4 x 10° kg-m?. It is attached to a torsion spring whose stiffness is 1.1 x 10° N-m/rad and a viscous damper of coefficient c. What is the value of c needed so that the needle is critically damped? Solution 3.24 From Eq. (3.21), For critical damping, ¢~= 1; therefore, 6, =2ykJ = 201-4 x10" x1.1x10 =7.85 Nm/(rad/s) 3.25 Determine the natural frequency and damping factor for the system shown in Figure £2.26. 1S Solution, The kinetic energy, the potential energy, and the di respectively, pation function are, We use Eq. (3.44) with q; =x and Q; = 0 to obtain Gi) +(6,+6,)+(h+h)x=0 mi +(c,+e,)¥+(k, +k) x=0 We see that the equivalent mass m, =m, the equivalent damping coefficient c. = c1 + c2, and the equivalent stiffness k= ki + ko. Therefore, 48 (©2009 Cengage Learning. ALL RIGHTS RESERVED. Solutions to Exercises — Chapter 3 3.26 Determine the natural frequency and damping factor for the system shown in Figure E2.27. Solution 3.26 First we determine the equivalent spring constant for the two springs in series ky and ks. Thus, from Eq. (2.16), ky=[2+4 kok Then, the kinetic energy, the potential energy, and the dissipation function are, respectively, 2 Lod 5 Vax + phat = (hs) 2 Datei terrae ae +re+e)# 2 2 2 We use Eq. (3.44) with qi =x and Q: = 0 to obtain a at imi) +(o,+e, +6)+ (ky + hy)x=0 mit+ (6,40, +6,)¥+(k +h,)x=0 We see that the equivalent mass m, = m, equivalent damping coefficient c. = ¢1 + ¢2 +c, and equivalent stiffness ke = ki + hos. ‘Therefore, [k_ [tks m. m +q+e 2mo, Section 3.6 3.27 For the base-excitation prototype shown in Figure 3.6, assume that the base displacement y(f) is known, choose x(f) as the generalized coordinate, and derive the equation of motion by using Lagrange’s equation. 49 ©2009 Cengage Learning. ALL RIGHTS RESERVED. Solutions to Exercises ~ Chapter 3 ‘The kinetic energy, the potential energy, and the dissipation function respectively, mi’ k(x-y) 1 2 D==c(#-3 3ce-5) We use Eq. (3.44) with g1 =x and Q; = 0 to obtain d A (mi) els) +k(0-y) =0 mi + ck + he = cp thy 3.28 Obtain the equation of motion for the system with rotating unbalance shown in Figure 3.7 by using Lagrange’s equations. Solution 3.28 The position vector from point O to mass ma is h-ccosat)i+(x+esinon)j where /r is the horizontal offset of point O’ from point O and x is measured in the transverse direction from the static-equilibrium position. Therefore, the velocity vector is (cosinon)i-+(%+ ewcost) j Then the total kinetic energy is T Mi? +(e) Mi? +m (cosinor) +(%+20 cosor} | 1 2 a 2 ee ee = Me tom, [e%o? sin? or +3? +2*0* cos? ot + 2iwcos.ar | 4 (+m,)2 +m, [6%0? +2e%0c0s 01] 2 2° Note that the potential energy associated with the gravity loading is not considered here, because motions about the static equilibrium position are considered. ‘The total potential energy and the dissipation function are, respectively, 50 ©2009 Cengage Learning, ALL RIGHTS RESERVED. Solutions to Exercises — Chapter 3 Application of the Lagrange equation (ar) ar aD, ov dt a me & results in S ((We+m,) + 0m, c0s01)—0+ cir kx =0 (M +m,)¥—em,0” sinet + ct + ke =0 (M +m, )¥+ ct + ke =em,0* sinot 3.29 Obtain the equation of motion for the system shown in Figure 3.10 by using moment balance and compare it to the results obtained by using Lagrange’s equation. Solution 3.29 Referring to the figure, for ‘small’ angular oscillations, we see that taking the sum of the moments about the pivot point O gives (cL) 1, + (KO) L, +5 o,8+ 2, 2 Jo,O-—mgL, sin 0 -m,g—sind (Jo, +o.) O + eL{0+ (1 =mgl, -mgo-0 where Jo: and Jon are given by Eq, (b) of Example 3.11. We see that this result agrees with that obtained in Example 3.11. 3.30 Derive the governing equation for the single-degree-of-freedom system shown in Figure £3.30 in terms of @ when @ is small, and obtain an expression for its natural frequency. ‘The top mass of the pendulum is a sphere, and the mass m, of the horizontal rod and the mass my of the rod that is supporting m, are each uniformly distributed. The cylinder rolls without slipping. ‘The rotational inertia J, of the cylinder is about the point O and Jip is the total rotational inertia of the rod about the point s. Assume that these rotational inertias are known. Solution 3.30 51 (©2009 Cengage Learning. ALL RIGHTS RESERVED. Solutions to Exercises — Chapter 3 Assuming that the cantilever beam material has a Young’s modulus of elasticity E and the inertia about the bending axis is J, the two springs at the left-end of the system have an equivalent spring constant ot Ok 3EL } where, The geometric constraint relations are x.=70, x, =Lp x=hp, 4=Lp Therefore, ahem tmp ads (2) ata (2) @ 2 2 2 2°UL 27, See 2 2 Lh If we write the kinetic energy as T=h me 3m we see that If we take the datum plane for the lower half of the pendulum to be a distance L2 away from point s and the datum plane for the upper half of the pendulum and the center of the sphere to be a distance Zs away from point s, then the total potential energy is 52 (©2009 Cengage Learning. ALL RIGHTS RESERVED. Solutions to Exercises — Chapter 3 eel eee ee L eee — Bg phasl tghal th times (tease) 1-cosg)~m,gl,(1~cos) For “small” g, we have cos <1 and the potential energy becomes [at thal thal tmes aren alee] spomlghalee If we write the potential energy as elton] we find that corral tf lif te) o-son-na() ‘The dissipation function is which can be written as where (z) Then, based on the last of Eqs. (3.47), we obtain the governing equation 33 ©2009 Cengage Learning. ALL RIGHTS RESERVED. Solutions to Exercises ~ Chapter 3 mO+c,0+k,0=M() from which the natural frequency can be determined as 3.31 For the fluid-float system shown in Figure E3.31, J, is the mass moment of inertia about point O and ms is the mass of the bar. Answer the following: a) For “small” angular oscillations, derive the governing equation of motion for the fluid float system 'b) What is the value of the damping coefficient ¢ for which the system is critically damped? Solution 3.31; (a) We represent the rotation of the bar rT i by the generalized coordinate @,the vertical ©» © M_, displacement of the mass by the coordinate = Rigid, x, and the vertical displacement of the ies ae % a damper by the coordinate x, For small 2 | limes rotations, we have that ° LV i ‘Then the system kinetic energy, system potential energy, dissipation function, and generalized force are, foe 7(6) 1) =F IP +3 omit (J, + mL FP v(a)=0 DO)= Fei) =4. Q, =mgi 2(bat) amgi-2-( £074 j)+mgi- Meal 2 Ft ~(Jmae+me)- p22 exo where we have used Eqs. (3.45a). Making use of Eq. (3.44), recognizing that qi = @ and carrying out the different operations, we obtain (©2009 Cengage Learning. ALL RIGHTS Solutions to Exercises ~ Chapter 3 ad 2) 2 1 xa” on Gl (Jo +m? )8) -0+ ea 0--0-(}mg-+me).- p20 (i, +mt?) + ca°6-+ 4a? pel?0 =(m-+m,/2) The term on the right-hand side is a constant term, which would not appear in the governing equation if oscillations about the static-equilibrium position were considered. (b) Comparing the above equation with the standard form given by Eqs. (3.47), we identify the equivalent inertia, the equivalent stiffness, and the equivalent damping as m,=J,+mE ad? pg? ‘Then, making use of Eq. (3.19), the damping coefficient ¢ that will make the system critically damped corresponds to which leads to 2y(u, +ml?)(pxa°gh /4) ‘Therefore, the damping coefficient c that will make the system critically damped is 4 [ucrme\(or8) 3.32 Determine the nonlinear governing equation of motion for the kinematically constrained system shown in Figure 3.32. Consider only vertical motions of my. Solution 3.32 Consider the motion of the linkages and the nomenclature given in the figure. From trigonometry, the displacements of the masses are 55 ©2009 Cengage Learning, ALL RIGHTS RESERVED. Solutions to Exercises ~ Chapter 3 x, =(a+b)[sind-sin(@—y)] x, =b[sind-sin(0-y)] x, = b[cos(@ -y) -cos0] ‘Then the velocities are 4, = (a+ byycos(O—y) byrcos(O-y) by sin -y) ‘The total kinetic energy is 1 Ailes eee ima + mks + ms 2 , 7 tm(a +b)*ycos*(0-y) + FMB? 0s'(0-y/) +hmpy sin’(0-y) = F[tmcas by +m,b"}cos*(0 —w) + mp sin? (O-y) |W? ‘an be written as Myr where My) ={m(a+by' +m,b"}cos*(0—y) + mB sin’ (0 -y) Assuming that gravity loading is acting normal to the plane, the total potential energy is v fh +hbx = S[h {sind —sin(@-y)}? +, fc0s(0-y)—cos0} | which can be written as v=kw) where Bre npc 2 2 Kw)= Sle {sind —sin(o~y)}’ + & feos(0 -y) —c0s0}"] Given that the system is not damped and there are no extemal forces acting on the system, the Lagrange equation is d(or) ar av al oy 56 ©2009 Cengage Learning, ALL RIGHTS RESERVED. Solutions to Exercises ~ Chapter 3 which evaluates to a 2-2 (Lager?) ?_xtyy— $(uen)-Z(fuow + 2 xw-o My + My? -harww +Kiy)=0 MW + SM WW? +KWY)=0 where the prime denoted the derivative with respect to y. 3.33 Determine the natural frequency of the angle bracket shown in Figure E3.33. Each leg of the bracket has a uniformly distributed mass m and a length L. Solution 3.33, We can derive the expression for the system natural frequency either one of the following two ways: i) by using the energy expressions and ii) by using the equation of motion derived from the moment balance method. Let @ be the angular displacement of the bracket, with @= 0 corresponding to the initial position. Fach leg of the bracket is at an angle y from the vertical. i) Energy expressions and natural frequency: i on ro t ‘The configuration of the system in the displaced in position is as shown in the figure on the right. Based iwn/ } on the illustration , the potential energy is : Ve ma-£(cos —cos(y +8) = mg % (cost -0)-cosy) =mg £[reosy—cosly +8)—cos(y-8)] om £[aeosy ~200s((y+0-+7-0)/2)e0s((y+0-7+0)/2)] =mgL[cosy —cosy cos] =mgLcosy[1-cos0] For ‘small’ oscillations about @= 0, we have Ve H(mgheos7)o* from which we identify the equivalent stiffness as k= mgLcos@. The kinetic energy is 37 ©2009 Cengage Learning. ALL RIGHTS RESERVED. Solutions to Exercises ~ Chapter 3 1 1 T= LIP + IP =>(2,)0° qih + IF => (24.) where J, = ml7/3 is the mass moment of inertia of each bar about point O. From this expression, we identify the equivalent inertia as m, =2J,. Then the natural frequency is [K _ [mgLeosy ™ ii) Moment balance method and natural frequency: ° 6 Referring to the figure at the right, we have that ro | a . in zsin(y +8) vf i iS Le sin -8) | a le melt Then the sum of the moments about point O gives 2,6 + mgl,—mgl, =0 2J,6 + mg (Ly-L,)=0 26+ mma Sfsiny +0)-sin(y -0)]=0 24,0 + mgLsin ((y +@-y +0)/2)cos((y +0 +y-8)/2)=0 24,6 +mgL.cosy sind = 0 For “small” oscillations about = 0, the above equation reduces to 2,6 + mgL.cos yO =0 mglcosy 2. 3.34 Determine the natural frequency for the vertical oscillations of the system shown in Figure E3.34. Let L be the static equilibrium length of the spring and let |x/L| << 1. The angle vis arbitrary. Solution 3.34 The length of the extended string Z, is related to the amount of downward extension x and the angle yby ‘Therefore, the natural frequency is 2 =D +x? -2xLeos(7-y) =P +x +2xLcos(y) 58 (©2009 Cengage Learning. ALL RIGHTS RESERVED. Solutions te Exercises — Chapter 3 ‘Then, the amount 5 that each spring stretches is given by 5=1,-L= JP +¥ +2xLeos(y)-L = { 1+ (ey + 2eostr) 7 ] For the assumption that |x/L| << 1, we have am t{ie12eosy)-1] =s0087 Since dis the extension of each spring, then the potential energy of the system is 1 v —1ks? Lie? = 2 ty > 3 (2keos" y)s* (2k) 6" ewe cos? y from which we identify the equivalent stiffness as k, =2keos* y The total kinetic energy is, ‘Therefore, the natural frequency is 2, a fe rkeos'y ayy m m 3.35 A planar pendulum of mass m and constant length / is shown in Figure E3.35. This pendulum is described by the following nonlinear equation ml°6 +mglsind =0 where @ is the angle measured from the vertical. Determine the static-equilibrium positions of this system and linearize the system for “small” oscillations about each of the system static-equilibrium positions. Solution 3.35 We rewrite the equation as and let @be of the form 0=0,+8, 59 (©2009 Cengage Learning, ALL RIGHTS RESERVED. Solutions to Exercises ~ Chapter 3 where the subscript d indicates the dynamic portion of the solution and the subscript 0 indicates the static (time-independent) portion of the solution. Substituting this assumed solution into the governing equation gives 6, +F sin(0, +8, ‘Thus, the static equilibrium position is given by S sing, =0 1 whose roots are 8, = 0, and @, = 7. Afier using the expansion [see Eq. (f) of Example 3.12)] sin(d, +0.) =sin0, +0, cos0, +... in the nonlinear equation, we obtain the following linear equation for ‘small’ oscillations about an equilibrium position 0= @, 6, +£0,c0s0, + Hsing, =0 For ‘small’ oscillations about 6, = 0, the linearized system is 6, +4, 0 and for ‘small’ oscillations about @, = 7, the linearized system is 6, > =0 3.36 For the translating and rotating disc system of Figure 3.9, choose the coordinate x measured from the unstretched length of the spring to describe the motion of the system. What are the equivalent inertia, equivalent stiffness, and equivalent damping properties for this system? Solution 3.36 In terms of the coordinate x for the system shown in Figure 3.9, the potential energy, energy, and the dissipation function are given by, respectively, 60 ©2009 Cengage Learning, ALL RIGHTS RESERVED. Solutions to Exercises — Chapter 3 Upon comparing the above expressions to the standard forms given by Eqs. (3.46b), we determine the equivalent system properties to be Jo m,=m+—% 3.37 For the inverted pendulum system of Figure 3.10, choose the coordinate x; measured from the unstretched length of the spring to describe the motion of the system, What are the equivalent inertia, equivalent stiffness, and equivalent damping properties for this system? Solution 3.37 In terms of the coordinate x, the kinetic energy, the potential energy, and the dissipation function are given by, respectively, Upon comparing the above expressions to the standard forms given by Eqs. (3.46b), we determine the equivalent system properties to be 2my" Lm,D, SE em +2 3 L 38 kak _ msl . Lh 2 ce 3.38 Consider a pendulum with an oscillating support as shown in Figure E3.38. The support is oscillating harmonically at a frequency a that is, u(t) =Ueosat At the point about which the pendulum rotates, there is a viscous damping moment cl” ) Determine expressions for the kinetic energy and the potential energy of the system. 'b) Show that the governing equation of motion can be written as 61 ©2009 Cengage Learning. ALL RIGHTS RESERVED. Solutions to Exercises — Chapter 3 dod 51S dr 7 -U,Q? cosQz]sind =0 ae where and u,-2 7 ) Approximate the governing equation in (b) for ‘small’ angular oscillations about 8, = 0 using a two-term Taylor expansion for sind, and show that the nonlineac stiffness is of the softening type. Solution 3.38 The position vector and the velocity of the mass with respect to the fixed point O are given by 1, =[u-LeosO]i+ Isindj (Wésind +i)i+16cos0j ‘The kinetic energy of the mass is Tr F(a) = i ml (idsind +i) +((dc0s0) | pole sin? 0+ 16iisind +1 +°O* cos* 6 | : m[ PO +17 + 216tisind | 2 The potential energy of the mass is y mg[-lcos0+u] ‘The Lagrange equation (3.44) takes the form 4(a) mr Vo a\ a6) 00° 08 which evaluates to PO 4 (m1°6 + mliisin®) ~ mibitcos 0 + mgl sind = m6 + mliisinO + ml6icos6 — mlbivcosO + mgl sin 8 + cl’6 =0 mil°6 + cl’6 + mliisinO + mgl sind =0 mP°6 +cl°6+ ml(ii+ g)sind =0 For the given applied harmonic displacement u, we see that 62 ©2009 Cengage Learning. ALL RIGHTS RESERVED. Solutions to Exercises — Chapter 3 ii=-Uo cosat Thus, the governing equation becomes ml°6 + cl°6-+ ml(g—Ua* cost) sind =0 Introducing @, and the non-dimensional quantities given in the problem statement, wwe obtain 2 2 d'O dO mle; Tate Po, woe Uo" cosot)sind =0 £o, 2 2 (1-U,0% cosQr)sind =0 If we expand sindabout 0= 0 as 7 sing 9-2 6 the governing equation reduces to ae Since the @ term is preceded by a negative sign, the nonlinearity is of the softening type. £0 +2 Bs s(-ustconn(o-Z)-0 3.39 Use Lagrange’s equation to derive the equation describing the vibratory system shown in Figure E3.39, which consists of two gears, each of radius r and rotary inertia J. ‘They drive an elastically constrained rack of mass m. ‘The elasticity of the constraint is k. From the equation of motion, determine an expression for the natural fiequency. Solution 3.39 ‘The kinetic energy and the potential energy are, respectively, 2d? ~([4+ far Jo 2 2 ‘Then, from Eq. (3.44), with q: = 0, D = 0, and Q; = 0, we obtain aie +mr)6|+ k*9=0 (20 +mr?) 6+ k9=0 Therefore, the natural frequency is given by 63 ©2009 Cengage Learning, ALL RIGHTS RESERVED. Solutions to Exercises ~ Chapter 3 3.40 Obtain the governing equation of motion in terms of the generalized coordinate 0 for torsional oscillations of the wind turbine shown in Figure £3.40, Assume that the turbine blades spin at « rad/s and that the total mass unbalance is represented by mass m, located ata distance e from the axis of rotation. ‘The support for the turbine is a solid circular rod of diameter d, length Z, and it is made from a material with a shear modulus G. The turbine body and blades have a rotary inertia J;, Assume that the damping coefficient for torsional oscillations is ¢, Solution 3.40 The kinetic energy, the potential energy, and the dissipation function are, respectively, where, from Table 2.3, Gad* 32L ‘The mass unbalance is accounted for through the generalized force. Then, from Eq. G.44), with qi = @and from Eq. (3.45) with a 1, Lew coswt we obtain J6+60+k,0 =m,Lew cosat 3.41 The uniform concentric cylinder of radius R rolls without slipping on the inclined surface as shown in Figure E3.41. The cylinder has another cylinder of radius r < R concentrically attached to it. The smaller cylinder has a cable wrapped around it. The other end of the cable is fixed. The cable is parallel to the inclined surface. If the stiffness of the cable is k and the mass and rotary inertia of the two attached cylinders are ‘m and Jo, respectively, then determine an expression for the natural frequency of the system in Hz. The length of the unstretched spring is L. Solution 3.41 We let x be the displacement of the center of the cylinder in the direction parallel to the incline. ‘Then, the kinetic energy and the potential energy are, respectively, lot pp lye 2 zm +5 Job slime +Jo)6 64 (©2009 Cengage Learning. ALL RIGHTS RESERVED. Solutions to Exercises - Chapter 3 Taking the static equilibrium position as the reference point, the potential energy of the cylinder can be determined from the vertical position of the disk as Vi =mg(y,-%) where Lsina y= (L+AL)sin or =(L-+ RO)sinor ‘Therefore, V, = mg(Lsina -(L+ R0)sin@) =—mgROsina ‘Then, the total potential energy is Saray +h ky*8 —mgROsina ‘The Lagrange equation is Carrying out the indicated operations, we find that (mk? + J,)6 +4770 = mgRsina Then, recognizing that m, =mR? + Jy k= kr? we obtain the natural frequeney as Lf iw 2x \mR?+Jo 3.42 For the pulley shown in Figure E3.42, determine an expression for natural frequeney when, for oscillations about the static equilibrium position. The springs are stretched by an amount x, at the static equilibrium. ‘The rotary inertia of the pulley about its center is Jo, the radius of the pulley is r, and the stiffness of each translation spring is k. Solution 3.42 When one sprin is extended, the other is compressed. Therefore, for displacements about the static equilibrium position, the potential energy is 1 v be het ke? = k(r0)° = kr’? =—(2kr? ) 0° Note that gravity loading is already accounted for, since we are considering oscillations about the static equilibrium position. The kinetic energy is 65 ©2009 Cengage Leaming. ALL RIGHTS RESERVED. Solutions to Exercises ~ Chapter 3 T=15,0° 2 We identify the equivalent inertia and the equivalent stiffness on the system as m,=Jo Kk, = 2kr? Hence, the natural frequency is 3.43 The pendulum shown in Figure £3.43 oscillates about the pivot at O. If the mass of the rigid bar of length Zs can be neglected, then determine an expression for the damped natural frequency of the system for “small” angular oscillations. olution 3.43 For “small” angular oscillations about the upright position, we determine that the kinetic energy, the potential energy, and the dissipation function are, respectively, 1 270 Sly sind)’ —mg(L,—1,)cos0 T=256° (40) matt ~1)(1- 50") = S(t +e ~h)) 0° ~ mel —h) (7) be £((4.-1)sin0)] o(,—h)' (00508) = he(,-L) where Jo=m(L,-L)° Making use of the Lagrange equation given by Eq. (3.44), we determine that the equation of motion as Job +e(Ly- LY 6+ (kL +mg(L,-h))O= ‘Then, the natural frequency is AL; +mg(L,~L,) _ [kl +mg(Ls-L) Jo (Lh) 66 ©2009 Cengage Learning. ALL RIGHTS RESERVED. ‘The damping factor is Solutions to Exercises ~ Chapter 3 where 3.44 Consider the pendulum shown in Figure £3.44. If the bar is rigid and weightless, then determine the natural frequency of the system and compare it to the natural frequency of the pendulum shown in Figure 2.18b. What conclusions ean you draw? lution 3.44 If gravity acts normal to the plane of the system and the oscillations are small, the inetic energy and the potential energy are, respectively, Tah mee 2 1 2 V ==k(b0) £i(00) Therefore, the governing equation of motion is, m6 + kb? =0 io ao fe mE LNm ‘The natural frequency of the system shown in Figure 2.17b is fi In the current system, the natural frequency is independent of the acceleration due to gravity. However, it is dependent on the mass of the system, the spring stiffness, and the location of the spring. and the natural frequency is 3.45 Consider the weightless rigid rod shown in Figure E3.45. At one end of the rod is a mass m, and from the other end of the rod another mass m is suspended from a taut string. The system is undamped and it is in equilibrium as shown in the figure. For “small” angles of rotation @ about the static equilibrium position @,, obtain an expression for the period. 67 (©2009 Cengage Learning, ALL RIGHTS RESERVED. Solutions to Exercises ~ Chapter 3 Solution 3.45 At equilibrium, we assume that the rigid rod makes an angle 0, with respect to the vertical. ‘The system oscillates about this equilibrium angle with an angular displacement 4. Then, the potential energy of this system is V =mgL (cos(0,)~cos(0, +0))—mgRsin(9) The kinetic energy is 7 = 1 mb6 ++ mR°6? cos? 0 2 2 For this undamped system, the system equation of motion will be obtained from Lagrange’s equation, which is fai((@ta | Mota sos at\ 06 00 Evaluation of the different terms leads to 4 (3) = m126+mRbcos (0) -2mR*0? sin(0)cos(0) ar 262 oa 0) 59 7 kG sin(0)cos(0) g =mgLsin(0, +0) —mgRcos(0) Noting that the generalized force Qy= 0, the equation of motion is m[ (+R? cos' (0))5— R°6* sin(0)cos(8)+ gLsin(@, +8)— gkcos(4)]=0 For “small” motions about the equilibrium position, we have sin(0, +0) ~sin(0,)+@cos(8,)+ cos(0) sin(@)=0 Retaining only the linear terms in @, the governing equation reduces to (2 +R°)6 + ghcos(0,)0 = gk -gLsin(0,) Noting that at static equilibrium, mgR = mgLsin(6,) we arrive at the following equation of motion (2 +R?)6+ gl.cos(@,)a=0 From the stiffness and inertia terms in this equation, the natural frequency is 68 (©2009 Cengage Learning, ALL RIGHTS RESERVED. Solutions to Exercises ~ Chapter 3 and the period is 3.46 For “small” oscillations @ about the nominal position specified by the angle £, determine an expression for the natural frequency of the system shown in Figure £3.46. ‘The springs are not stretched in this nominal position. Solution 3.46 For ‘small’ oscillations, the potential energy is cal k,(L0) + 3h (10) -m,gL(cos(A)—cos(f +9)) +m,g2L(cos(B)—cos(f +0) 2 (a +4,)0! gb (2m,—m)(cos()-cos(8+0)) 2 al +h,)O" + gh(2m, —m)( asin 50" cosp Where we have used cos( +0) ~c0s 8 Osin -7.0" cos f ‘The kinetic energy is L apt 52 @ + 5m, (218) (m,+4m,) 0° To obtain the governing equation of motion for this undamped system, we use Lagrange’s equation, which is a Fo 0. a\a6) 30° 09 ** Noting that the generalized force Qo ~ 0, and after evaluating the different terms, we obtain the governing equation of motion LLL (mm +4m,) 6+ (L(k, +2) + g(2m, ~m,)oos 8) ]=—g (2m, —m,)sin B Then, from the stiffness and inertia terms of this equation, the natural frequency of the system is 69 (©2009 Cengage Learning. ALL RIGHTS RESERVED. Solutions to Exercises ~ Chapter 3 7 L(m,+4m;) 3.47 A circular cylinder of mass m and radius r rolls on the interior of a cylindrical surface of radius R, as shown in Figure E3.47. The system is in equilibrium at 0 = 0. Determine an expression for the natural frequeney of the system for “small” oscillations about this equilibrium position. Sok 3.47 If we let the rotation of the mass about its center be denoted g, then the kinetic energy of mass m is J(d-+0) + MCR where Since KO where *) tm(t—r) = (Rr) ‘The potential energy is given by V =mg(R-r)(1-cos0) which for “small” oscillation about @, = 0 can be approximated as Ve * (R-ne? For this undamped system, the Lagrange equation is (cee arenclae os a\ab) 00" 30 Setting Qo = 0, since there are no external forces other than gravity loading, and evaluating the different terms, we obtain JG+mg(R-r)O=0 From the governing equation, the natural frequency is 70 (©2009 Cengage Learning. ALL RIGHTS RESERVED. Solutions to Exercises ~ Chapter 3 3.48 The undamped pendulum pivoted at point O shown in Figure £3.48 has a cylinder of ‘mass mz at its top that rotates without slipping on the interior of a cylinder. At the bottom end of the pendulum, a mass my is attached. ‘The rod connecting the two masses is rigid and weightless. The system is in equilibrium at = 0. Determine an expression for the period of oscillation of the system. Assume that maL2 <2L. Solution 3.48 If we denote the rotation of the mass m about its center by the angle g, then the kinetic energy of the system is given by Ly ily (nga dm pe T= sJob +74a(6+9) +ymlid’ where Since (J, +7,)6 =-ng for no slipping, the kinetic energy of the system can be written as where, Jn ota(t- 22) om Jaa fm (428) +32 (+28) +5mr For “small” angular oscillations about @ = 0 and choosing the pivot point as the datum, the potential energy for the system can be approximated as ¥ =m gh, cos0 +m, 0080 ~& (mm, ~Mmal;)9*— 8 (my —mla) ‘The Lagrange equation for this undamped system is 4(ar) ar av dt\00) 60 00 7 ©2009 Cengage Learning, ALL RIGHTS RESERVED. Solutions to Exercises — Chapter 3 Upon substituting the appropriate quantities in this equation and noting that Qo = 0, we obtain J+ g(mL,-m,L,)0=0 From the stiffness and the inertia terms, the natural frequency is 2 = Mls) Qn Since mala 0 when 324y1- 2 rr ce Using £zero from MATLAB, we find that ¢= 0.6953. 4.14 Consider the mercury-filled (p = 13.6 x 10° kg/m*) U-tube manometer shown in Figure 2.16. The total length of the mercury in the manometer is 0.7 m. When the mercury is displaced from its equilibrium position, damped oscillations are observed. The oscillations are such that the peak amplitude nine periods away from the peak amplitude of the first cycle has decreased by a factor of eight. What are the values of the viscous damping factor and the damped frequency of oscillation? Solution 4.14 From the problem statement x» = xy/8, Then from Eq. (4.38), o-tu(%) u u( 8) Li =0231 Ply 9 % and from Eq. (4.40) 95 (©2009 Cengage Learning. ALL RIGHTS RESERVED. Solutions to Exercises ~ Chapter 4 i = 0.0367 1+ (22/0.231)° To obtain the natural frequency, we note that the mass m =p4gL. and from Eq. (2.27) =2pg4,, Then the natural frequency is o, = P2! PAL and the damped frequency is that rad/s 2921 — 0.0367? = 5.288 rad/s 0, = 4.15 A truck tire is characterized with a stiffiness of 1.25%10° N/m, a mass of 35 kg, and a damping coefficient of 4200 Ns/m, Determine the natural frequency and damping factor for this system, and obtain the free response of the system initiated from an initial displacement of X, m and zero initial velocity. Solution 4.15 The parameters for the system are 0, = fex10 189° rads 35 © 4200 2me, — 2x35x189 89V1-0.317? =179.3. rad/s $0, =189x0.317 = 59.9 =F vi-0317? tan OSI 0.317 g=tan" =1.25 rad By using Eq. (4.33), we have that eo sin(at+ 9) x()= 9% sin(179.3t + 1.25) © Vi-031 =1.055.X,e" sin(179.3¢ 41.25) m Section 4.2.4 4.16 A microelectromechanical system has a mass of 0.30 ig and a stiffness of 0.15 N/m. Assume that the damping coefficient for the system is negligible and that the gravity 96 (©2009 Cengage Learning. ALL RIGHTS RESERVED. Solutions to Exercises ~ Chapter 4 loading is normal to the direction of motion. Determine the displacement response for this system, if the system is provided an initial displacement of 2 m and an initial velocity of $ mm/s. Solution 4.16 The natural frequency is and the velocity ratio is 5x10? 22360.67 x2 «10° After using Eq. (4.42), we find that 0.112 x10°V1+0.112? =2.01 pm 459 rad. and from Eq. (4.41) x(0) = 2.0 1sin(22360.7¢ + 1.459) om 4.17 An empirical formula used for determining the natural frequency associated with translational motions of a steel chimney has the form o, T1004. rads where d is the diameter of the chimney and his the height of the chimney. ‘The damping factor associated with a chimney of diameter 7 m and height of 100 m is 0.002. This chimney is located in a region where wind gusts are capable of producing initial displacements of 0.1 m and initial velocities of 0.4 m/s. Determine the maximum displacement experienced by the chimney and check if it satisfies the construction codes, which require the maximum displacement to be less than 4% of the diameter. If the present chimney does not satisfy this code, what design changes would you propose so that the construction code is satisfied? Solution, From the statement of the problem, we have that X, = 0.1 m, V,= 04 m/s, h= 100 m, d=7m, and ¢= 0.002. Thus, the natural frequency is 71007 100? =4.97 rad/s The velocity ratio is 97 (©2009 Cengage Learning. ALL RIGHTS RESERVED. Solutions to Exercises ~ Chapter 4 Vv, 04 “eX, 497%0.1 ia Then, from Eqs. (4.42), 4, =X, 14 GZ — 0,1 f1 4 08054 0.002)" 9 199 m 1-¢ 1-0.002 -Nl=¢? _ ja vI-0.0027 ta tan —— = 0,892 Patan ey, 0.002+0.803 °°? ™ and, from Eq. (4.7), hoe =ae g=tan! BAG" tan? M1=0.002" 1 569 rad c 0.002 igure 4.18, we see that =0 We notice that ay ~ @. From Eq. (4.43) and 2a when Ogb yan + Pa — 9 =O or Olga © Onl gar ™ Olgas = 9 ~ Py = 1.569 — 0.892 = 0.677 ‘Then, the maximum displacement is Fea) = Ae SIMO shyus + Pu) =0.129¢°%""" sin(0,677 +.0.892) 129 m which is less than the maximum allowable displacement of 0.04x7 = 0.28 m. Hence, no design change is necessary. 4.18 A quarter-car model of a heavy vehicle is shown in Figure E4.18, This vehicle is traveling with a constant speed v on a flat road. It hits a bump, which produces an initial displacement of 0.2 m and an initial velocity of 0.1 m/s at the base of the system. If the mass m of the vehicle is 5000 kg, the stiffness & is 2800 KN/m, and the damping coefficient c is 18 kN-s/m, determine the displacement response of this system and discuss when the system returns to its equilibrium position, Sol m. ‘Since the system can be described by a moving base model, the governing equation of motion is obtained from Eq. (3.31). Thus, @z & => + 2a, a, a ae 98 (©2009 Cengage Learning. ALL RIGHTS RESERVED. to}z= Solutions to Exercises ~ Chapter 4 where 2(0) is the relative displacement given by a) =x()~ (0) AD =RO-IO) It is recalled that x() and y(0) are the absolute displacements of the mass and base, respectively. This is an initial value problem with the initial conditions being (0) = #(0) =0 yO)= 30) =%, »(0)=0 1>0 (After bump, on flat surface) ‘Then the governing equation is with the initial conditions From Eqs. (4.8b), (4.41), and (4.42), we have that 2) = Ae sin(ot +9,) where the amplitude and phase are given by -(6+",) In the equation for gy, the negative signs are explicitly preserved in order to get the proper quadrant. We are given that ¥, = 0.2 m, V, = 0.1 m/s, m= 5x10° kg, k= 2,800 kN/m, and c= 18 KNi(m/s). Therefore, @, —E- 2.810" 53.66 rad/s mV 5x10 __e 18x10° =0.076 in 22.8x10°x5x10" 0,8 = 23.66 0.076=1.798=1.8 rad/s 0, =0,)1-2 =23.66Vi -0.076 =23.59 rad/s ¥, 01 0211 0,Y, 23.66%0.2 99 (©2009 Cengage Learning. ALL RIGHTS RESERVED. Solutions to Exercises — Chapter 4 and =-1.668 rad Since (= 0, 1>0, x(t) = 2(t) = Ae" sin(ot+,) t>0 =0.2e™ sin(23.591- 1.668) 1>0 For £>0, it is seen that the envelop of x(#) decays to 2% of its initial value in about 2.17 s. 4.19 Consider the slender tower shown in Figure E4.19, which vibrates in the transverse direction shown in the figure. It is made from reinforced concrete. An estimate for the first natural frequency of this system is 0.15 Hz. The logarithmic decrement values measured for the tower with uncracked reinforced concrete material and cracked reinforced concrete material are 0.04 and 0.10, respectively. If a wind gust induces an initial displacement of 0.5 m and an initial velocity of 0.2 m/s, determine the peak displacement amplitudes in the cases with uncracked conerete material and cracked concrete material. Solution 4.19 Since the system is underdamped, from Eq. (4.43), we see that the first maximum of the displacement occurs at f= fax When Ogham =P Ps ‘Therefore, from Eqs. (4.42) and (4.41), we find that the maximum displacement is v.+ey 1c =X,f1-G2+(V, +o) ee enim Xan = X(gae) = Xf o,X, For uncracked concrete, the damping factor is 100 (©2009 Cengage Learning. ALL RIGHTS RESERVED. a Van? 40° Solutions to Exercises ~ Chapter 4 0.04 = 0.00637 Var +008 and for cracked concrete the damping factor is o The natural frequency of the system is @, = dimensional initial velocity is 6 ol Van +8 Jan? +01 0.942x0.5 = 0.0159 2atfy = 270.15 = 0.942 rad/s. Then, the non- 02 .425 1 —————- = 1.164 rad Vite, 0.425 + 0.00637 ™ a gq = tan WTS Swe Put. _ 1-564=1.164 _ 9 ggy99 tang, 147.1 XV 1-62. +(V, 45,0)" = 0.5y1+ (0.425 +0.00637)° = 0.5445 Then Kus = 0.5445¢°9™ = 0.5429 m Cracked concrete: neat VE V+. 9g. =tan ieee S = 4, _1.554~1.156 tang. 2.89 1 0.425 +-0.0159 =1,156 rad wtan'—1_.1554 rad 0.015! = 0.00633 Xfl-e2 +(V, +E, ~0.5y/1+(0.425+ 0.0159)" =0.546 m ‘Then 546° = 0.5426 m Thus, for the given changes in the damping due to the cracking of the concrete material, the maximum displacement remains virtually unchanged. 101 ©2009 Cengage Learning, ALL RIGHTS RESERVED. Solutions to Exercises — Chapter 4 4.20 A diving platform is shown in Figure E4.20. The damping factor associated with this platform, which is made from reinforced concrete, is 0.012, and the first natural frequency of the platform-slab vibrations is designed to be 12 Hz. When a diver jumps off this platform, an initial displacement of 1 mm and an initial velocity of 0.1 nvs are induced to the platform. Determine the resulting vibrations, Solution 4.20 The first natural frequency of the platform-slab is 0, =2xf, =20x12=75.398 rad/s and the damped natural frequency of the system is. @,=0,Ni-0.01F =o, =75.39 rad/s ement X, = 1 mm and initial velocity V, = 0.1 mm/s, the For the given velocity ratio V, is yy, Ol @,X, 73.39%0.001 Then, from Eq. (4.42), the respective amplitude and phase quantities are = 7 Gh 2 x, AKO FE) =0.01 h +(1.326+0.012) =1.67%109 m MS ay 1 tan! ___ 9.6417. rad o+V, Gav, 0.012 +1.326 and from Eq, (441), the displacement is x( 326 4 = tant Ae sin(a,t+ 9,4) 1.67 x 10% ¢005"™ sin(75.39t +0.642) m =1.67e°™ sin(75.39¢ + 0.642) mm ‘Thus, the form of the underdamped response describing the motions has been obtained, Section 4.3 421 Consider the vibratory model of the microelectromechanical system treated in Example 3.15. If the stiffness of the translation spring &, the stiffness of the torsion spring k, and the system damping coefficient ¢ are all positive values, would it be possible for the system to have unstable behavior? Solution 4.21 Equation (g) of Example 3.15, which is the governing equation, is (J. +m) b + eLib + (k, +L} +g (Ly -L)/2)$ = kL,x,() 102 ©2009 Cengage Learning. ALL RIGHTS RESERVED. Solutions to Exercises — Chapter 4 For the system to be stable, the equivalent damping c, and the equivalent stiffness k, must be greater than zero. We see that c, is greater than zero and that k, is greater than zero when A, +k + mg(L,—L)/2>0 or 2, msl, . m, Kk, +kE +28 5 M8 AL + TAD > Ta If this relation is not satisfied, the system will be unstable. 4.22 When there is a fluid moving with a speed V through a pipe, the transverse vibrations y of a mass m attached to the pipe can be described by the following equation mj+[k,-eV? ]y= where kis the equivalent stiffness of the pipe and c is a constant that depends on the pipe cross-sectional area, the length of the pipe, and the density of the fluid, If m is always positive, ke is always positive, c is positive, and V is positive, determine when the system can exhibit unstable behavior. Solution 4.22 The system becomes unstable when k,-eV? <0 or when k0 or a > mel > 2 4.24 In studies of aircraft wing flutter, the following simplified model is used to study the system vibrations mi+(c+a)x+kxe=0 where the system parameters are m, c, and k. The damping constant a is due to aerodynamic forces, which change with the angle of attack. Beyond a certain angle of attack, this constant can assume negative values. Determine the conditions on m, c, a, and k for which this system can be unstable. Solution 4, Ifm > 0, k> 0, c> 0, then for the system to be stable c+ 20. For c+ a=0, the system exhibits bounded oscillations. 4.25 Consider the pendulum of length Z and mass m that is rotating about an axis through its hinged point with an angular speed @ At a given @, the pendulum swings out an angle cas shown in Figure E4.aa, a) If the pendulum can oscillate an additional angle g about a, then determine the governing equation of motion for this system. 'b) What are the dynamic equilibrium positions for this system; that is, when @ = @ = 0? ©) Discuss the stability of the system. Solution 4.25 ‘The equation of motion will be obtained by using Lagrange’s equations of motion, To this end, we first construct the expressions for the kinetic energy and the potential energy of the system. The position vector from the point O to a point x shown in the figure below is given by 104 ©2009 Cengage Learning, ALL RIGHTS RESERVED. Solutions to Exercises ~ Chapter 4 r= xcos(a+g)i+ xsin(a+9)j a and the corresponding velocity is given by v=[-xsin(aw + 9)p]i+(-wi)x(xcos(a+9)i)+ [xcos(a +.9)@] j+(—oi)x(xsin(a +) f) xsin(a + 9)p]i+[xcos(a+9)@] j+ ® [osina+—)]k ‘The kinetic energy of the system is given by 14 lh im T= |vnim=5 owas eee m = 2 dx (3) 36 +0°x? sin’ (a+)]7 @) = -i(ae je +0" sin'(a+9)] ‘The potential energy is L Vin ="Ecos(ar+9) 4) a) For the undamped and unforced system, the Lagrange equation of motion d(a ao 5 4% a Evaluating the different terms, we find that the equation of motion is 2 2 ()-(¢ as sin(a + p)cos(a +g) +, : gL sina g)=0 © b) To find the equilibrium position from Eq. (6), we set the time derivatives of ¢ to zero. This leads to [2 Jats cosa) Mesias 9) =0 M whose solutions are sin(a+g)=0 (8a) and = 8b) cos(a +g) = a a L (8b) 105 (©2009 Cengage Learning. ALL RIGHTS RESERVED. Solutions to Exercises ~ Chapter 4 The equilibrium positions are (9) where gps is possible when ©) To determine the stability of motions about each of the equilibrium positions, we consider the linearized equation of motion ¢ about each of these positions; that is, P= G+ 4- Noting that for 60s(4+ Py, +8) ~ €05( +94) -GSiN(a+ 9) . 7 7 (10) sin(a + 9, +9) = sin(a + Py) +Pcos(a +My) Eq. (6) becomes (2 )« [cos?(at +94.) ~sin*(a-+94,) | — Me cose oo} ay 2 = (S» costa 9q) BEE lias) From Eqs. (8) and (9), we see that the right hand side of Eq. (11) is zero. Thus, Eq. (11) reduces to (2 )5-(e [cos(c+ 94,)~sin*(c94,)]= 8 cos(a + ie =0 (12) For small oscillations about gx = ~ 38 (4) If the coefficient of ¢ in Eq. (13) is positive and the system exhibits bourded motion about 1; this means that the system is stable from the notion of boundedress introduced in Section 4.3. For small oscillations about g = —cr, Eq. (12) simplifies to 106 ©2009 Cengage Learning. ALL RIGHTS RESERVED. Solutions to Exercises ~ Chapter 4 mE) 3 |( mi?) >, mgL| (Fete a Since the coefficient of ¢ in Eq. (15) is negative, the motions about g2 continually grow away from the equilibrium position gp. Hence, this equilibrium position is unstable. For small oscillations about gs, Eq. (12) becomes (5) 3 mgl. TSE c0s(a+ Py) > me Jo” [cos* (a+ 43) —sin*(a+9s)] -) >| 2 0. (2 Jo forces -sin org) Mebsode+n)]=0 (16) If (“Je cos(2(2+94)) the motions about gp; are bounded and this equilibrium position is stable. 4.26 A rigid disk rotates with a constant angular speed @. A mass m is mounted in a slot in the disk as shown in Figure F4.26. Assume that gravity acts normal to the plane of the disk. ‘The mass is restrained by two identical springs of stiffness & and each spring is initially stretched by an amount 6,.. Determine an expression for the natural frequency of the system and the effect that d, and @ have on the natural frequency when the displacement of the mass is small. At what speed does the system become unstable? Solution 4.26 ‘The kinetic energy is lie 2 2 and the potential energy is 1 = 54(5, +3) + 24(6, -x)' =K(82 +2?) For this undamped system, the Lagrange equation is a(ar) ar ov dt\ a& a ok Noting that the external force Q, = 0, and after performing the indicated operations on the expressions for the kinetic energy and potential energy, we obtain ¥+(0?-0*)x= where 107 (©2009 Cengage Leaming. ALL RIGHTS RESERVED. Solutions to Exercises ~ Chapter 4 Therefore, the natural frequency of the system is We sce that the initial elongations of the springs do not affect the natural frequency and that the rotational speed of the disk decreases the natural frequency of the system. In addition, the system becomes unstable when @> ay, 4.27 A mass m shown in Figure E4.27 is suspended in a magnetic field by two springs of length L that each have a tension 7>. The magnetic attraction force on the mass is a function of the magnetic flux ,, and this force is inversely proportional to the square of the distance from the magnet; that is, E, mats where fis a constant. For small values of [xj find a relationship between the magnetic flux and the spring tension for which the system is stable. Solution 4.27 jumming the forces on the mass as shown in the free-body diagram below, we obtain nyt ad " TsinO= Tel Fy [om [or = |” x or 7 Tsind= Tyxlk. @ ®) (@) Forces acting on the mass and (b) free-body diagram of the system. meee F L where, for small |x, 108 ©2009 Cengage Learning, ALL RIGHTS RESERVED. Solutions to Exercises — Chapter 4 The system exhibits bounded motions about x = 0 if the coefficient of x is positive. For this system, this means that for stability Section 4.5 4.28 Compare the free-oscillation characteristics of the following systems, when they are set into motion from the initial displacement of 0.1 m and zero initial velocity. a)mét+ci+kx=0 b)métck+kxtax o)m¥+ckt+kx-ax'=0 Assume that the parameter values are as follows: m= 10 kg, ¢= 10 Ns/m, k= 10 Nim, and a= 25 N/m’, Consider another set of free responses, when each of these three systems is set into motion from an initial displacement of 0.6 m. Compare these responses with the previously obtained responses. What can you conclude? Solution 4.28 The results obtained through numerical integration of the respective equations are given below. It is clear that the system responses initiated from (0.1 m, 0 nvs) are virtually identical and one cannot distinguish the responses of the nonlinear systems b) and c) from each other and from the response of the linear system a). On the other hand, the responses of the nonlinear systems b) and c) initiated from (0.6 m, 0 m/s) are clearly different from each other and from the response of the linear system a). All three systems have an equilibrium position at (0,0). In the second case, the initial condition is far away from this equilibrium position compared to the first case, and the nonlinear effects are pronounced. col | ea o a an ce 4 - igure £4.28 109 ©2009 Cengage Leaming, ALL RIGHTS RESERVED. Solutions to Exercises — Chapter 4 ‘MATLAB program us ure for Exercise 428 ‘span-linspace(0.0, 10,200); options-odeset(RelTol le-6,AbsTol'{Le-4 le-4)); m=10; €=10; K-10; Xo~{0.1 0.6); Vo-0.0; Alpha=[0 25 -25]; lab=['k- ‘4 ‘i; for n-:length(Xo) figure(0) hold on for mn=1:length(Alpha) {tx}-ode45(@nonlinearspringtspan,{Xo(n) Vol, optionsym,Alpha(nn) kc); te Debs ») [ab}-legend(q (aq (6yq(6)); set(b(1),fontsize’ 14) xlabel('t,fontsize’,14) ylabel(x()fontsize’,14) title((X_o =" num2str(Xo(a))} ontsize’ 14) set(gca,fontsize'14) box on end function dd-nonlinearspring(t,x,m,Alpha,k,c) (2); -(c*x(2)+K*x(1)+Alpha*x(1)3)/mn]s 4.29 Consider the displacement-time history shown in Figure E4.29 for free oscillations of a vibratory system. Examine this history and discuss if this system can be characterized as having viscous damping Solution 4.29 We notice that the envelope of peak amplitudes does not decay exponentially. In fact, they decay linearly. Therefore, this system is not viscously damped. ‘The observed damping characteristic can be captured by dry friction damping, which is discussed in Section 4.5.2. 4.30 Consider the system with the piecewise-linear spring given in Figure 4.24 and choose the damping factor ¢ to be 0.1. Set the system in motion from the initial conditions (0) = 0 units and the nondimensional initial velocity 4y(0)/dr=V,/(@,d)=20. Obtain the displacement-time histories of the system, and plot them as shown in Figure 4.25 for the following values of the stiffness coefficient ys: a) 0, b) 1, and c) 20. In each case, tabulate the nondimensional period for each cycle of oscillation and discuss the results, Solution 4.30 ‘The results are shown in the figure below. The half periods for each value of ware: 4=0 (Linear case) T2=3.1775 3.1774 3.1773 3.1773 110 ©2009 Cengage Learning, ALL RIGHTS RESERVED. Solutions to Exercises ~ Chapter 4 4=1 (Nonlinear case) 12 =2,2943 2.3197 2.3582 2.4152 4=20 (Nonlinear case) 7/2 =0.7879 0.80301 0.81991 0.84067 We see that in the two nonlinear cases, the periods increase as time increases. Therefore, the frequency is decreasing as time progresses. 20 —— Figure £4.30 ts-linspace(0,tend.Npt); Vodk=20; % Search ranges for fzero. Previously determined from an display of y vs tau ser[2 4; 10; 12 13;23;4.5; 68;9 1050.5 1; 12;23;33.5}; inter=inline( ppval(pp,x);%/pp); opt=optimset( display’ off); cc=0; for k=I:length(mu) [ty]-ode45(@sdofstops2,ts,[0 Vodk}'.]zmu(k)); plot((1:Npt),y(1:Npt,1,e(k,:)) MW (©2009 Cengage Learning. ALL RIGHTS RESERVED. Solutions to Exercises ~ Chapter 4 mae disp(('mu ~' num2str(mu(k))]) +3 coect (kk)=fzero(inter ser(ee,),0ptpp); end d=[r(an) (an disp(((Period {m) r(m+2)r(an+1) r(emt3)-x(m42)]; * num2str(d))) jum2ste(mu(1))},[\mu* num2ste(mu(2))}f\mu~ num2str(mu(3)))); ) plot({0 ts(Npt),(0 0}.e(1.)) xlabel( ‘tau fontsize'14) ylabely(tau);*fontsize’, 14) set(gca, fontsize'14) plot(0:.5:20, Vodk*exp(-[0:.5:20]*2),k:'0:.5:20,-Vodk*exp(-{0:.5:20]*2),k:') function 2-sdofstops2(t,y,z,mu) ifabs(y(1)<=1 b=; else hey(1sigenQ(); end 112 ©2009 Cengage Learning. ALL RIGHTS RESERVED. Solutions to Exercises ~ Chapter 5 Solutions to Exercises Chapter 5 Section 5.2.1 5.1 A vibratory system with a natural frequency of 10 Hz. is suddenly excited by a harmonic excitation at 6 Hz. What should the damping factor of the system be so that the system settles down within 5 % of the steady-state amplitude in 200 ms? Solution 5.1 From Eq, (5.10b), we have afl | Ty = Oly z a 2a xt0x0.22 Lig] 20S o 6/10 12.572 Inf 0.0833, i-g] Using the function £2ero in MATLAB, we find that ¢= 0.1994, ‘MATLAB program used to obtained numerical value for Exercise 5.1 -2*pi"6; wn=2*pi*10; 40.05; 140.2; cestinline('wn*td*xHTog(d*sqrt(I-x"2)/(w/wn))x'wni'w'satd); opt-optimset( display’ off); | xi=fzero(esty[.01 .7],opt.wn,w4,td Section 5.2.2 5.2 A 150 kg mass is suspended by a spring-damper combination with a stiffness of 30 x 10° N/m and a viscous-damping constant of 1500 N-s/m. The mass is initially at rest. Calculate the steady-state displacement amplitude and phase if the mass is excited by a harmonic force amplitude of 70 N at 3 Hz. Solution 5.2 ‘The natural frequency and damping factor are, respectively, fe 30000 _ 14.14 rad/s mV 150 c 1500 2ma, — 2x150x14.14 0.354 113, (©2009 Cengage Leaming. ALL RIGHTS RESERVED. Solutions to Exercises ~ Chapter 5 ‘Then the non-dimensional excitation frequency is Q = ea, = (2afla,) = (27%3/14.14) = 1.333. The value of the amplitude response and phase response are, respectively, 1 H(1.33) = =0.818 + (2x 0.354 1.333) 20.354 «1.333 (1.33) = tan”! =~ = 0,882 rad. ieee i s From Eq, (5.17), we have that the steady-state amplitude is . 700.81 Steady-state amplitude = “* 711,333) = 2X98! _ 9.9919 jeady-state amplit PH333) = T0G0 m 5.3 The dynamic amplification or attenuation of a single degree-of-freedom system is defined as the ratio of the steady-state magnitude of the displacement response to its, static displacement F./k, where the mass is being driven by a tarmonic force of magnitude F,. Find the dynamic amplification or attenuation of a single degree-of- freedom system that is being excited at 100 rad/s and has the following system parameters: m~ 100 kg, k~ 20 kN/m, and ¢ = 6000 N-s/m. Solution 5.3 ‘The dynamic amplification/attenuation factor, which is given by Eq. (5.8a), has the form HQ) @ For the given parameter values @, (B 20x10" 141.42 rad/s mY 100 a= 2-10 4.797 ©) @, 14142 c 6000 Oe 04, Vk J100x20%10° Then, substituting from Eqs. (b) into Eq. (a), we determine that 75 © 107°) + (0.424%0.707)" Since the value of (0.707) is larger than 1, there is an amplification of the input. 14 (©2009 Cengage Learning. ALL RIGHTS RESERVED. Solutions to Exercises ~ Chapter 5 5.4 Consider the two independent single degree-of-freedom systems in Figure E5.4 that are each being forced to vibrate harmonically at the same frequency @, System 2 starts its vibration at ¢= 1, and the excitation on System #2 starts at =f: that is, SiO) =F sin(ot)u(t) LAO) = Fysin(oft~t,)ut—1,) ) Use Eqs. (5.1) to (5.9) to show that the steady-state responses of the two systems are Sia() = F.11(0,6,)in(O2-0(046,)) Xyq(T) = EMO 0, 16 )i( AL 0,C0~ 1,)-0(Q/0,,7%,)) where Q= dm, @,= Ona/ Om, ¥= G/Gi, and 1 i iS £ b) If both systems are operating in their respective mass-dominated regions, then what is the ratio of the magnitudes of the amplitudes of System 2 to that of System 1 and their relative phase. Solution 5.4 (a) Writing Eq. (5.9) in dimensional form, we have for system #1 xu(O= A sin(w -,) ky (1-(o/a,)') +(2¢,0/0,)° where 6, = tan! 2510/0 1-(a/a) For system #2, we have EyO= fi sin(o(t—1,)-) hy ('-(o/o,)') +(26,0/0,)° where tan! 2.0/0, 1-(a/a,) 1s (©2009 Cengage Learning. ALL RIGHTS RESERVED. Solutions to Exercises ~ Chapter S Let 2 =a, & = alm, 7= Gli, T= Opt, and % = apf. ‘Then, the steady-state response of the first system can be written as x,,()=$ 4, sin(ar—a,) Ky(1-2) + 26,0)" where 1 26,0. 1-9 and the steady-state response of the second system can be written as E aq (F) =e =sin(Q(z-1,)-@, © (1-2 2) +2590) Kor *.)-4) ky i-2°/0? 6, = tan where Thus, if 1 (1a) +(2ba)" 1 2ba I-a* H(a,b)= 0(a,b) = tan then, the respective steady-state responses can be written as, Xig(t)= a H(Q,¢,)sin(Qr-0(0,4,)) 2 H(Q/o,,76,)sin(Q/.@,(t -2,)-0(Q/ @,,76,)) Aral) =F (b) In the mass-dominated region, Q >> 1 and Q/q, >> 1; therefore, from Eq. (5.33), HiT) issinar -m)=- XP) Ee? sin(a( t-1,)-2)=- ~paesin k,Q? ‘The ratio of the magnitudes of the amplitudes R is bk 2 Ro; F, k, a; 116 (©2009 Cengage Learning. ALL RIGHTS RESERVED. Solutions to Exercises — Chapter 5 For 12 f, the phase of the response x2 with respect to the phase of the response x; is —0%,, which is a lag. Section 5.2.4 5.5 The control tab of an airplane elevator is shown schematically in the Figure E5.5. The ‘mass moment of inertia JJ, of the control tab about the hinge point O is known, but the torsional spring constant , associated with the control linkage is difficult to evaluate, and hence, the natural frequency @, = ,/J, is difficult to determine. An experiment is designed to determine this natural frequency of the system. In this experiment, the elevator is rigidly mounted, springs with stiffness & and stiffiness fy are attached to the control tab, and the tab is harmonically excited at an amplitude ¢, as illustrated in the figure. The excitation frequency « is varied until resonance occurs at «=, , and this value is noted. Assuming that the damping in the system is negligible, determine an expression for @, in terms of @,, k,,k;yJgyand L.. Solution 5.5 ‘Taking a moment balance about the static equilibrium position, which is assumed to be at = 0, for ‘small’ angular oscillations, we obtain SO +k, +(k, +k )E 10 = kel sin(ot) where ¢ is the amplitude of the displacement provided at the end of the spring kp. Since, the damping is negligible, the resonance frequency is the natural frequency of the modified system; that is, 5.6 Consider translational motions of a vibratory system with a mass m of 200 kg and a stiffness k of 200 Nim. When a harmonic forcing of the form F,sin(/) is suddenly applied to the mass of the system, where F, = 1.0 N, determine the responses of the system and plot them for the following cases: a) @ = 0.2 rad/s, b) @ = 1.0 rad/s, and c) @=2.0 rad/s, and discuss the results. Solution 5.6 ‘The natural frequency of the system is k _ [200 === ==! mas olen fie me M7 (©2009 Cengage Learning, ALL RIGHTS RESERVED. Solutions to Exercises ~ Chapter 5 Case a): Since @ = 0.2 rad/s, Q = 0.2/1 = 0.2. In this case Q ¥ 1, and the displacement response is given by Eq. (5.2 x()= in(wr) - Qsin(o,1)} ra {sin(0.2t) -0.2sin(¢)} =0.005{sin(0.2r) -0.2sin()} m In this case Q = 1, and the displacement Case b): Since @ = 1 rad/s, Q = 1/1 response is given by Eq. (5.23); that is, x= Fpsino -a,!c0s(o,))} i © 200%2 =0.0025{sin(#) —rcos(s)} m sin(1)—1cos(1)} Case ¢): Since @ = 2 rad/s, Q = 2/1 = 2. In this case Q + 1, and the displacement response is again given by Eq. (5.21); that is, x) tg : = noe -Qsin(o,)} 1 ‘ a 21) —2sin(¢) ZOOR GT Sine) Asin} = 0.0017 {sin(2/)—2sin()}_m From these results, we see that the displacement response is bounded when the excitation frequency is away from the natural frequency of the undamped system, But when the excitation frequency is the same as the natural frequency, the response is unbounded. 118 (©2009 Cengage Learning. ALL RIGHTS RESERVED. Figure £5.6 ‘MATLAB program used to generate figures for Exercise 5.6 ‘linspace(0,200,400); subplot(3,1,1) plott,0.0054(sin(0.2*t)-0.2sin(t))k-’) xlabel(t (8), fontsize’ 14) ylabel(x(0)fontsize’ 14) ‘et(gea,fonisize’,14) subplot(3,1,2) ‘linspace(0,200,400); plot(,0.0025%(sin(t)-t*cos(t))’k-) xlabel(t(5),'fontsize’14) ylabel(x(0),fontsize'14) se1(gea, fontsize’,14) subplot(3,1,3) ‘linspace(0,50,400); plot(t,-0.0017*(sin(2*t)-2*sin(t))k-) xlabel(t (s);fontsize’,14) ylabel('x({)fontsize',14) set(gea, fontsize’14) Section 5.3.3 Solutions to Exercises ~ Chapter 5 5.7 A micromechanical resonator is to be designed to have a Q factor of 1000 and a natural frequency of 2 kHz, Determine the system-damping factor and the system bandwidth. Since {= 1/(20), ¢= 5x10%. Then, from Eqs. (5.47), the system’s upper and lower cutoff frequencies are, respectively, 119 (©2009 Cengage Learning. ALL RIGHTS RESERVED. Solutions to Exercises ~ Chapter $ 2, =2x 2000x510 =2 Hz 5.8 If a sensor modeled as a mass-spring-damper system is to be redesigned so that its is increased by a factor of four, determine the corresponding percentage changes in the system natural frequency and damping ratio. Solution 5.8 Using Eqs. (5.41) with a, = 4, we obtain ) 50% a 2, do, = woo 22a.) 100 % ag = 51) -100(.4-1) 100% Section 5.3.4 5.9 Consider the machine of 25 kg mass that is mounted on springs and dampers shown in Figure E5.9. ‘The equivalent stiffness of the spring combination is 9 kN/m, and the equivalent damping of the damper combination is 150 N-s/m. An excitation force F() is directly applied to the mass of the system, as shown in the figure. Consider the displacement x(0) as the output, the forcing F(¢) as the input, and determine the frequency response of this system. Solution 5.9 From Eq, (5.53), the transfer function is X(s) 1 F(s) ms Gs) cs+k Substituting s = je in the above equation and using the given values for the parameters, we obtain 1 Go) = -25m* +150 jo +9000 Section 5.4 120 (©2009 Cengage Learning. ALL RIGHTS RESERVED. Solutions to Exercises ~ Chapter 5 5.10 An air compressor with a total mass of 100 kg is operated at a constant speed of 2000 rpm. ‘The unbalanced mass is 4 kg and the eccentricity is 0.12 m. The properties of the mounting are such that the damping factor = 0.15. Determine the following:(a) the spring stiffness that the mounting should have so that only 20 % of the unbalance force is transmitted to the foundation and (b) determine the amplitude of the transmitted force Solution 5.10 (a) The excitation frequency is 20002 309.44 rad/s 60 Since the fraction of the unbalanced force transmitted to the foundation y= 0.2, from Eq, (9 of Example 5.7, we find that the appropriate non-dimensional frequency ratio is Q= 1-267-426-1472 = yl-2x0.15°(1-0.2") + J(1-2x0.15(0 ~0.27))? -140.27 =2.72 ‘This means that the system natural frequency should be ©, = @/2=209.44/2.72= 76.98 rad/s and the stiffness is = 0,m=(76.98)' x100 =5.926x10° Nim (b) The amplitude of the dynamic force that is transmitted to the base is YE, = ym,e0* = 0.24 x 0.12 x(209.44)? =4211.1 N 5.11 A motor of mass m is mounted at the end of a cantilever beam and it is found that the beam deflects 10 mm. When the motor is running at 1800 rpm, an unbalanced force of 100 N is measured. If the beam damping is negligible and its mass can be neglected, then what speed should the motor operate at so that the amplitude of the dynamic response is less than ay m. Solution 5.11 Using the notation in Section 5.4, the given parameter values are _1800%2 60 Furthermore, for the given static-defleetion value, the natural frequency of the system can be determined by using Eq. (3.15) as F,=100N and o = 607 rad/s 121 (©2009 Cengage Learning. ALL RIGHTS RESERVED. Solutions to Exercises ~ Chapter 5 5813132 rad/s 10107 me =, = 0.0028. kg-m te (607) Since F, = m,a?, we have From Eq. (5.64), the amplitude of the dynamic response is given by [e@|= 47,2) where e_ M= m+m, Noting that the damping is negligible, we have that H,(Q)= which can be written as Then Itis required that ol as the input, the acceleration response %(¢) as the output, and determine the frequency-response function of this system Solution 5.12 ‘The governing equation of motion is Gp Gok 30 Bay ‘Taking the Laplace transform of this equation and assuming that the initial conditions are zero, we obtain ms°X(s)+3csX(s) +X (s) =3cs¥(s)+k¥(s) or X(s)___ Bes +k ¥(s) ms? +3es+k ‘The frequency response function is = Hx] z[v0]] where L is the Laplace transform operator and we have assumed that the initial conditions are zero. After the substitution s = jo, we have s(3cs-+k) ms? +3cs +k G(jo) Go 10 or jo (3cjo +k ew + jok Gyo =-—22Ge ) car + Jak ma’ +36jo +k k—ma* +3 1.8. ‘Therefore, let ky = 1.8k; = 288 kN/m. We have to make sure that the 7R at the operating speed is less than 0.08. ‘Thus, we use Eq. G) of Example 5.13, and for © = 439.8/40 = 11 and y= 1.8, we obtain is? +(2x0.3x11<(+1.8) 1.8?(1-1P)' +(2%0.3x11x(1+1 0229 a) which is less than 0.08. 5.17 A compressor weighing 1000 kg operates at 1500 rpm. The compressor was originally attached to the floor of a building, but it produced undesirable vibrations to the building. To reduce these vibrations to the building, it is proposed that a concrete block be poured that is separated from the building and that the compressor then be mounted to this block. The location of the compressor will permit the block to be 1.8 m by 2.2 m. The soil on which the concrete block will rest has a compression coefficient k. = 20 x 10° N/m®. If the density of the concrete is 23 x 10° N/m’, then determine the height of the concrete block so that there is an 80% reduction in the force transmitted to the soil. The height of the concrete block hy affects the mass and the stiffizess of the system. From Eq, (5.105), the transmissibility ratio is TR = 1 — R= 1-0.8~0.2. The forcing frequency is 1500x 27 oa eer 60, 50x rad/s From Eq, (5.104), we find for ¢ = 0 that the non-dimensional exé could also have been estimated from Figure 5.34) Q=2 = fisTR @, which provides us an expression to determine the natural frequency; that is, = 64.13 rad/s o 50a 1+7R Vi+1/0.2 ‘The mass of the system is, 126 (©2009 Cengage Learning. ALL RIGHTS RESERVED. Solutions to Exercises ~ Chapter 5 m= My + My = AMPH. 1000 keg 8 where Ayu is the area of the block and psy is the density of the block material. ‘The stiffness is k=kAy Nim Since, k aa ®, then Aunbin Po 4 1999 = bw g 2, which leads to Ag Ing =| “4 —1000 o-oo) _{ 20x10" 218%22 _ 999 ( 98 64.13 1.8x2.2x23x10° =1.965 m Section 5.8 5.18 Show that the work done per cycle by a harmonic force acting directly on the mass ofa linear spring-mass-damper system is equal to the energy dissipated by the system per forcing cycle. For harmonic excitation of the form f(t)= F,sin(or) the displacement, from Eq. (5.9) is, x) =X, sin(or-0(Q)) where 127 ©2009 Cengage Leaming. ALL RIGHTS RESERVED. Solutions to Exercises ~ Chapter 5 Next, we determine the work done by the inertia force, the spring force, and the external force. To this end, we integrate each term in Eq. (5.1a) with respect to x over one cycle of forced oscillation; that is, from ¢= 0 to = 2afw. This leads to W,+W,+W, =W, where W, 7 f(Oidt = FX,0 aff sin(wt)cos(at — O(Q))dt = x.0| 5 sion sinvor -a(2))- 5sin-0(2| 2o 2 l =FX,xsin(O(Q)) From Eq. (2.4), the work done by the mass m is, W,, = "f itidt =—mX%0° J sin(or -0(2)) cos(at -8())dt 3 a =-mX?0"| [ss sin(@t — O(Q))sin(@t — A(Q)) — $sin(0(0) - x2] [sin® (2x -0(Q)) -sin*(@(Q)) ]= 0 From either Eg. (2.48) or (5.109), W,=c | xdt=cw'X} | cos*(wt-O(Q))at a a a tae aslo = ca” afte in(or aco] 2y2{ 2m yl rg tint = car al Git 2-00) ~5F sin"( ay} =cmoX} From Eq. (2.10), we have that w= T kxidt =KX3@ J sin(wt -0(Q))cos(wt - 6(O))dt = wxro[ sinc AQ) sin(ot - OQ) —Fsin( CQ) — aay] ° = ee [sin®(2 -0(@)) -sin?(9(Q)) ]=0 Therefore, since W» = Wi=0, 128 (©2009 Cengage Learning. ALL RIGHTS RESERVED. Solutions to Exercises ~ Chapter 5 or cxoX? = FX, 2sin(O(Q)) which can be simplified slightly to yield cox, = F,sin(()) After substituting F, Ane t H(Q) we have that P M(a)=sin(0()) Since c= 2¢/(may), we obtain 22H (Q) = sin(Q)) But 26 sin(@(Q)) = 2¢2H(Q) oY +Q¢ny Thus, we find that 2¢RH(Q) = 26QH(Q) and, therefore, Wa= Wj. 5.19 A spring-mass system with m = 20 kg and & = 8000 N/m vibrates horizontally on a surface with coefficient of friction 4s = 0.2. When excited harmonically at 5 Hz, the steady-state displacement of the mass is 10 cm. Determine the equivalent viscous damping. Solution 5.19 Making use of Eq, (5.114), the equivalent viscous damping is ¢, -Abmg __4x0.2x20%9.8 OX, x2xx5x(10/100) =15.9 Ni(m/s) 5.20 The area of the hysteresis loop of a cyclically loaded system, which is the energy dissipated per forcing cycle, is measured to be 10 N-m, and the measured maximum response X, of the deflection is 2 cm. Calculate the equivalent viscous damping coefficient of this system if the driving force has a frequency of 30 Hz. 129 (©2009 Cengage Leaming, ALL RIGHTS RESERVED. Solutions to Exercises — Chapter 5 Solution 5.20 From Eq, (5.121), the energy dissipated per cycle is E sucrnat = 2kABX? Therefore, E, kp lnk Then, from Eq, (5.122), we have 10 21.11 N/(mis) 2a x2 x30% 0.02 Section 5.9 5.21 Torsional oscillations of a vibratory system is governed by the following equation I,G+6,0+ kp = M, cos(af) + M, cos(o,t) where 20 N-m/(rad/s) rad/s, and @,=2.0rad/s J, =20kg-m*, k, =20N-m/rad, M,=10N-m, M,=20N-m, Determine the steady-state response of the system. Solution 5.21 The steady-state response of the system can be obtained from Eq, (5.155) as. 1 k, YH(,)M, cos( or -0(0,)) where Q, HQ, Me @ (Q,) = tan? 252 From the given parameters, 130 (©2009 Cengage Learning. ALL RIGHTS RESERVED. Solutions to Exercises ~ Chapter 5 = fh =) =1 rows J, V20 20 ¢ “J 2J,0, 2x20x1 0.5 Then, ‘Therefore, +“ cos (21 - 2.678) 20 =0.5cos(t—1.571) +0.277 cos (2t-2.678) rad 5.22 Determine an expression for the output of an accelerometer with the damping factor and natural frequency @, when it is mounted on a system executing periodic displacement motions of the form y=Asinos+ A, sino,t Solution 5.22 We use the material from Sections 5.6 and 5.9 to determine that the accelerometer ‘output is given by 2()=2,()+2,() where, from Eq. (5.98), (1) = 4,0; H(o,)sin (wt -0(@,)) 2,(0) = 4,0} H(o,)sin(a,t-0(«,)) and 131 (©2009 Cengage Learning. ALL RIGHTS RESERVED. Solutions to Exercises ~ Chapter 5 Ha) = +(2£0,/0,) ey] 5.23 A single degree-of-freedom system is driven by the periodic triangular wave excitation as shown in Case d of Appendix B. If the period 7'= 2 s and the amplitude f, = 10 N, then find the steady-state response of the system by considering the first three harmonies of the forcing. Assume that the system parameters are k= 10 kN/m, c = 10 N-sim and m =1 kg. Solution 5.23 From entry d of Appendix B and Eqs. (5.157) and (5.158), we find that the coefficients in the Fourier series are 1,2 4, 1/2)o, ‘Then, considering the first three harmonics, from Eq. (5.159) we obtain the response as 4f, he — 008 (2,r-0(Q, Sne[ ep font F -(2,)) cos(Q,7-9(Q, | 10 2x1x100 2 Fax rad/s 132 ©2009 Cengage Learning. ALL RIGHTS RESERVED. Solutions to Exercises ~ Chapter $ Then the response is x(7)= mals = A (cost 20) 5cos(37/200)+ agcon(re/20)] m 5.24 Consider again the results of Example 5.17, where the response of a single degree- of-freedom system to a periodic pulse train was studied. If we were ‘o differentiate Eq. (©) of this example with respect to rand set the result equal to zero, then the value of r= ax that satisfies this equation in one period is the time at which the maximum value Xpax =x(Tingx) Occurs. Use MATLAB (or a similar programming language), to deteninine xnay a a function of the damping ratio for Q, = 0.042426 and a= 0.4. The results should look like those shown in Figure E5.24. Solution 5.24 Upon differentiating Eq. (c) of Example 5.17 and setting the results to zero, we obtain Fan) = YE (QQ, €05( 2,7, —Q,) + ¥,) =0 where the quantities appearing in the equation are defined in Example 5.17. The value of ‘oax that satisfies this equation in one period gives the value of x(t) = Xmaxe Which is determined from Equation (c) of Example 5.17; that is, pax = ¥Enge) = ab, [ +Ye (Q,)sin(Q,7,,4, —(Q,) +y, | The results are plotted below. 133 (©2009 Cengage Learning. ALL RIGHTS RESERVED. Figure E5.24 function ExerciseS_24 alpha=0.4; Omo-0.0424; L2-0.05; z-linspace(L7,.7.25); opt-optimset( display fofP); xmax=zer0s(length(2),1); for k=I:length(z) =fzero( @xmaxpulsetrain,{120 123],0pt,2(k),alpha,Omo,0); -xmax(k)-xmaxpulsetran(,2(K).alpha,Omo, 1); end plot(2,xmax,'k-) veaxis; w(I)=Lzs axis(v) xlabel( zeta’ fontsize’4) ylabel(x_{max}',fontsize'14) set(gea fontsize’14) function xdot-xmaxpulsetrain(tay,2,alpha,Omo,z2) n=1:200;, Om-Omotn; psi=atan2(sin(n*pi*alpha),0); th=atan2(2*z*Om, 1-Om."2); alpha*abssin(a*pitalpha)/(n*palpha))Jsqr(I-Om."2) 24(2*2*Om)."2), itz=0 o=<.*Om; xdot-e*cos{Om'*tau-repmatth | length(tau)}repmatps' | engthau)); else xxdot-alphae*sin(Om'*tau-repmat(t,length(tau)trepmat(si | ength(au); end 134 (©2009 Cengage Learning, ALL RIGHTS RESERVED. Solutions to Exercises ~ Chapter 5 5.25 Repeat Exercise 5.24, except for the abscissa use the nondimensional bandwidth of the system. Explain your results, Solution 5.25, The results are shown in the figure below. The bandwidth in terms of the non- dimensional frequency coefficient is obtained from Eqs. (5.47) and (5.48a); that is, 267 4261-6? - 1-267 -2¢ fl-?_ ¢<0.3827 =Vl-267 +26 1-6? = ¢>0.3827 A discontinuity occurs at ¢= 0.3827, which corresponds to By ~ 1.189. At values greater than this value, the bandwidth is a function of the upper cutoff frequency only, which decreases for values of ¢ beyond the value {= 0.3827. Recall Figure 5.2a. B, 18) 17] 15) 1a) 13 02 0408 o8 1 42 By Figure B5.25 MATL used i te figure for Exercise 525 function Exercises 25 4; Omo=0.0424; 20.08; opt-optimset(‘display’ oft); for k=I:length(z) 1(k)>fzero(@xmaxpulsetrain,[120 123] opt,2{k),alpha,Omo,0); xmax(k)=xmaxpulsetrain(1(k),2(K),alpha,Omo,1); if (k)<-0.382 mdo-sartl-2*2(ky242*2fk) sank) 2) 1-2*2(K)-2-242) Sqr -20)'2)); else [ist meni sone 135 (©2009 Cengage Leaming, ALL RIGHTS RESERVED. Solutions to Exercises ~ Chapter 5 end plot(Bw.xmax, veaxis; v()-Bw(1); v1.25 axis(v) xlabel(B_w,fontsize’14) ylabel('x_{max}';fontsize’14) ‘set(geafontsize’,14) function xdot-xmaxpulsetrain(tau.z,alpha,Omo,zz) = 1:200; Om=Omorn; psi=atan2(sin(n*pi*alpha),0); tan2(2*2*Om,1-Om."2); alpha*abs(sin(n*pi*alpha).((n*pi*alpha)) /sqet((I-Om."2)."2+(2*2*Om)."2); 0 -c.*0m; xdot-e*cos(Om'*tau-repmat(t | Jength(tau))*repmat(ps,Jength(tau)); else xxdot=alpha-c*sin(Om'*tau-repmat(t I length(tau))+repmat(ps I length(tau)) end. 5.26 Consider the base excitation of a single degree-of-freedom system shown in Figure 3.6 and whose motion is described by Eq. (5.76). If the displacement of the base is the periodic sawtooth waveform described by Case ¢ in Appendix B, then obtain an expression for the displacement of the system's mass. Solution 5.26 Since the force created by the displacement of the base of the system is f=% 24 y where vonr(5 aly (in) Yc0s (10,7) Therefore, . S()= fl)+ KO) where 269.2. Se5(i Yeos(iQ,7) We shall solve this as two separate problems and then combine (sum) the results, For AA2), from Eqs. (5.158) and (5.159), the Fourier series coefficients are 136 ©2009 Cengage Learning. ALL RIGHTS RESERVED. Solutions to Exercises - Chapter 5 a,=0, and b=-2 is Thus, the corresponding displacement response is (j= 2S MO) 59(i0,-—a¢i0,)) er For fi(2),the Fourier series coefficients are > =0, b=0, and ‘Then, the corresponding displacement response is x(t) = -22.. 5° H(Q, oo (i0,r-0(10,)) ete Combining the results, we obtain SH(Q,)c0s (i H@,) = sin(i2,r-(2,)) ALi Yo _ Yo HiQ,)) +5 7 5.27 Consider a sine wave forcing of magnitude F, and frequency 0 that has a duration of N periods fp, where fy = 2/@,. This sine wave is repeated every period T, where T = Mry = 271@, MN, and N and M are integers. If this periodic waveform is applied to the ‘mass of a single degree-of-freedom system, then SO=F,sin(o)[u)—uat-4,)] OStsT, O<4, + 0.38— +4 x-Sx? =0.15c0s(0.333r) ar ir where r= apf. The solutions obtained after numerical integration carried out from the initial conditions (0, 0) are shown below. Itis clear that the response of the linear system contains a dominant peak at @ = 1.49 rad/s while the response of the nonlinear system contains peaks at 0, 30, and Sa, Figure 5.28 143 (©2009 Cengage Learning, ALL RIGHTS RESERVED. Solutions to Exercises ~ Chapter 5 MATLAB program used to generate figure for Exercise $28 function Exercises_28 z2ta0.19; Omega=1/3; Fo-O.15; ‘T=140; Nsamp~24000; tspan-linspace(0,T,Nsamp-1); options={]; [txlin]-odes5(@ForeedOse,tspan,(0.01 0.1,options,zet,0,Omega,I,Fo); ‘figure(1) indx=find(¢==100), plot(t(indx) xin(indx,1)/ hold on xlabel( ‘tau fontsize' 4) yilabel(x(\tau)''fontsize14) titleCalpha = 0''fontsize14) set(gca fontsize’14) figure) [t,xnonlin]~ode45(@ForcedOsc,tspan,[0.01 0.1},options,zeta,5,Omega,1,Fo); plot((inds),xnonlin(indx,1)/k~) xlabel(\tau'fontsiz’,14) yilabel(x(\tau)'"fontsize14) titlealpha = 10000 Nim3' ont set(gc, fontsize',14) N-2°14; Nstart~5000; Fs-Nsamp/T F-(Fs*(0:N-1)/N)*2.0*pi; AmpLin-abs(fitlin(Nstart.Nstart*N,1).N)N; ‘AmpNonLin=abs(ffi(xnonlin(Nstar:Nstart+*N,1),N))IN; figure(3) inde=find(F=2), 10*log10(2* AmpLin(inde)); max(zlg); plot(4.47*flinde).2lg-zmax, ‘ylabelB fontsize’ 14) xlabel( omega (rad/s;'fonsize’14); set(gea fontsize’14) titl(\alpha ~ 0, Tontsize14) figure(4) lg=20"log10(2*AmpNonl in(inde)), 2zmax-max(zlg); plot(4.47*tinde),2le-2max ylabel(dB'fontsize’ 14) xlabel(‘omega (radis),'fontsize14); set(gca fontsize'14) title(\alpha = 10000 Nim'3'fontsize',14) function df-ForcedOse(t,x,221,alph, Omega,betFo) Af [x@); -2°zeta*x(2)-bet°x(1}alphi"x(1)°3 +Fo*cos(Omega")]: 5.29 Consider the following nonlinear single degree-of-freedom system subjected to a harmonic excitation, with the same values of parameters as in Exercise 5.28, except that now the excitation frequency is at = a. mié+ci+k-+ax = f, cost 144 ©2009 Cengage Learning. ALL RIGHTS RESERVED. Solutions to Exercises — Chapter 5 Compute the response of the system and compare it with that of the corresponding linear system. In addition, for this harmonic excitation compare the differences between the Fesponses of the system of softening stiffness discussed in Exercise 5.28 and the system of this exercise with hardening stiffness. Solution 5.29 The numerical results obtained through integrations from the initial conditions (0, 0) are shown below. It is clear that the response of the systems with the hardening spring and the softening spring have only one dominant peak in the spectrum as in the linear case. The amplitude of the steady-state motion is largest in the linear case, followed by the amplitude of the system with the hardening spring, and the amplitude of the system With the softening spring. The responses of the two nonlinear systems do not show any discemible nonlinear characteristics. If the excitation frequency is chosen to be ‘slightly” larger than the natural frequency, then the response of the system with the hardening spring will also show discemible nonlinear characteristics. On the other hand, if the excitation frequency is ‘slightly’ Jess than the natural frequency the response of the system with the softening spring will show discernable nonlinear characteristics, WTA | Hi i ud | HA Wii | II Ih 4AM 145, (©2009 Cengage Learning. ALL RIGHTS RESERVED. Solutions to Exercises — Chapter 5 tt I : i ie MATLAB program used to generate figure for Exereise 5.29 ‘Omega=1; Fo-0.15; 140; Nsamp=24000; tspan-linspace(0,T;Nsamp-1); N=214; Notart~5000; F-Noamp/T; FFs* O:N-1YIN)*2.0% alphok=[0 5-5]; ForeedOsc=inline([x(2); -2*zetatx(2)-bet*x(1)- alph*x(1)°3+Foteos(Omega*0)x'/dummy’,zeta'alph' Omega’ bet,'Fo' for j=1:length(alphok) [tx}-ode4s(ForcedOse,tspan,[0.01 0.1),],zet,alphok(),Omega,1,Fo); figure@*G-1)+1) indx=find<=60); plot((indx),x(indx,1),-) hold on xlabel\tau’fontsize, ‘ylabel(s(\tau)’ fon title({\alpha =" num2str(2000*alphok(j))],fontsize’,14) set(gea, fontsize’,14) figure@*(-1)12) AmpL in=abs( M(x(NstartNstart+N,1),N)VN; inde=find(f==2), zlg~20*log10(2* AmpLin(inde)); zzmax-max(zlg); plot(4.47°Mlinde),zlg-zmax,k-) ylabeldB',fontsize',14) xlabel(‘omega (rad/s, Yontsize'14); set(gca,fontsize'14) title((alphia =" num2str(2000*alphok())], fons: end 14) 146 ©2009 Cengage Learning. ALL RIGHTS RESER) Solutions to Exercises ~ Chapter 6 Solutions to Exercises Chapter 6 Section 6.2 6.1 Determine the response of a vibratory system governed by the following equation. (0) + 0.2%( 0) +3.5x(1) =1.Ssin(ot)u(t) +45(t-3) ‘Assume that m = 1 kg, the initial conditions are x(0) = 0.1 m and %(0)=0 m/s, and the tion frequency «= 1.4 rad/s. Plot the response. exci Solution 6.1 From the coefficients of the terms in the equation, we note that c = 0.2 N-s/m, and k= 3.5 Nim. Then c 0.2 = 0.0535 $= Fina, ~ Delx1.87 We shall break the solution into three parts by solving the responses to the initial conditions, the harmonic force input, and the delta funetion input separately and use the principle of superposition to determine the combined response of the system. The first part is the solution to an initial value problem with the initial velocity equal to zero. ‘Then from Eq. (4.33) where ten’ vi-0.05357 0.0535 0, =0, 87V1-0.0535? =1.868 rad/s 0,8 =1.868x0.0535 = 0.1 =1.517 rad ‘Then the displacement due to the initial conditions is OL x) -™ sin(1.867t +1.517)u(t) =0.1e™ sin(1.868¢ +1.517)u(t) m 147 ©2009 Cengage Learning. ALL RIGHTS RESERVED. Solutions to Exercises ~ Chapter 6 ‘The second portion of the solution is obtained from the suddenly applied sinusoidal excitation. For this solution, we use Eqs. (5.7) and (5.8). Thus, nei 4t—0(0)) + 26 “sin(ayti—2 * ae) ao 6a E, part) = k where Q = @/ a = 1.4/1.87 = 0.749, Fo/k = 1.5/3.5 = 0.429 m and, 1 1 H(Q)= - = 2088 (1-2) +(2gny’ (10.749) +(2«0.0535x0.749)" (0) = tan? 262 = tay? 20.0535%0-749 9.1799 rad - 1-0.749° (0) tan 26 gy 220053500535 941 yg 27-2) 2x 0.0535" —(1-0.749") ‘Then Suet) =0 s2sx.2%4sin( = 0.1799) + 0496 sin(1.8681-+2.901) fucty Vi-0.053: = [0.9535 (1.41—0.1799) +0.713e°°™ sin (1.8681+2.901) ]u()_ m. ‘The third part of the solution is due to the delta function of magnitude 4 N-s. For the delta function input, we use Eq. (6.1a) to determine the response and find that Fatal) = ) sin(w,[t—)5(9—3)dq, ae) sin(o,{t—3)u(t-3) mo, =2.141e°") sin(1.868[¢—3))u(t-3)_m The response of mass is (0) =X (0) + Xa + Xa which is plotted below. 148, (©2009 Cengage Learning. ALL RIGHTS RESERVED. Solutions to Exercises ~ Chapter 6 MATLAB program used to-generate figure for Exercise 6.1 13.5; m=1; €=0.2; Xo=.1; w=1.45 Fo=l wn=sqrt(im); zel@*m*wn); ‘Ao-Xo‘sqri(I-2"2); phiratan(sqrt(I-2°2)/2); wd-wn*sqr(1-2”2), inspace(0,60,600); xic=Aotexp(-z* wn"?).tsin(wd*tt phi; rewivn; He/sqet((1-'2)' 24242412), Theatan2(2*2*r 11°2); ‘The-atan2(2*2*sqet(I-2"2)2%772-(1-42)), xhar-Fo/k* H*(sin(w*t-Th) !1¥exp(-z*wn').*singwd*t#Taysqrt(1-2"2)); xdelta4*exp(-ztwn4(-3) *singd®(t-3)).%(=3)imiwd plot(xietxharéxdelta,I-) label(t (5)'ontsize’ 4) ylabel(x(®) (m)fontsize’ 14) ca, fontsize’ 4 6.2. Consider the when the system i lowing single degree-of-freedom system excited by two impulses itially at rest FO +24) + 4x) = 6 + 500-5) Determine the displacement response of this vibratory system and plot the results. 149 (©2009 Cengage Learning. ALL RIGHTS RESERVED. Solutions to Exercises ~ Chapter 6 Solution 6.2 We use the principle of superposition to determine the response of the system to the two impulse inputs. From the coefficients of the terms in the equation, we find that f =2 rad/s 2 2x12 2V1-0.5? =1.73 rad/s So, =0.5x2=1 rad/s 2, c ‘Then, from Eq. (6.1a), we find that na fe moog 5 ma, x) HP sin(co,[t—m)[5(n) + (9 -S)] dy [eM sin(o,p)u(s) +e" sin(o,{¢ -S)u(t-5) | = [ersin(1.739)u(1) + 6° sinG.73[1—Spu(e—5)] The result is plotted below. 6 2 t@ Figure £62 150 ©2009 Cengage Learning, ALL RIGHTS RESERVED. Solutions to Exercises — Chapter 6 MATLAB program used to generate igure for Exercise 6.2 ke; m=; 0-2; wn-sqrt(him); z-el2*m*wn); vwd-wn*sqrt( 1-22); inspace(0, 12,300); xt-(exp(2* wn) sind) expCz*wn"(-5).*sin(wd(C-) plotitxti-) xlabei(t ()'"fontsize'14) ylabel(x(t) (mfontsiz' 14) set(gea fontsize' 14) “(e=S)yimiwds 6.3 From extensive biomechanical tests, the spinal stiffness k of a person is estimated to be 50,000 N/m. Assume that the body mass is 80 kg. Let us assume that this person is driving an automobile without wearing a seat belt. On hitting an obstacle, the driver is thrown upwards, and drops in free fall onto an unpadded seat and experiences an impulse with a magnitude of 100 N-s. Determine the resulting motions if an undamped single degree-of-freedom model is used to model the vertical vibrations of this person. Solution 6.3 ‘The response of the system after the fall can be determined from the model (o where m = 80 kg, = 50,000 N/m, and the forcing is (0) =1005() N [50000 Making use of Eq. (6.4a) with £= 0 since the syste f 2 sin(o,t)u(t) me, mi + kee In addition, we see that 25 rad/s is undamped, we arrive at x(r 5 fim 25uCD) =0.05sin(25¢)u(t) m 6.4 Determine the response of the vibratory system discussed in Example 6.4 when the forcing due to the wind spectrum is of the following form [FUp|=200f6°°" N where fis the frequency in Hz. Plot the result for the values of 45 given in Example 6.4. Solution 6.4 Using Eq, (¢) of Example 6.4, the displacement response in the frequency domain is determined as 151 (©2009 Cengage Learning. ALL RIGHTS RESERVED. Solutions to Exercises — Chapter 6 254 2 bean |=? (129609 £ k k ‘The results for the three values of k used in Example 6.3 are plotted below. Therefore, in order to satisfy the requirement that |X(/f)|max < 5 cm, a lamppost with the stiffness k; is needed, F(t) Figure B6.4 Mi used to generate figun 64 function Exercise6_4 felinspace(0,10,300); k=[20000 30000 40000); " length(k) jindload2(k(n),m,2); plot(f,100°X,c(a,:)) hold on end {a bJ-legend((k_1 set(b(1),fontsize14) Jk 2 =" num2str(k(2))' Nim],['k_3 =" num2str(k(3)) "Nim; function (X,weib]-windload2(f,m,2) weib=2004F.exp(0.5 152 (©2009 Cengage Learning. ALL RIGHTS RESERVED. Solutions to Exercises ~ Chapter 6 2 *pisFsqrt(avky, HL Jsqr((I-r2)°242%241) 72); Xeweib.* Hk; Section 6.3 6.5 Determine the response of an underdamped single degree- subjected to the force f(t) = Fe. Assume that the system is initially at rest. Solution 6.5, We start with Eq. (6.16) and determine the response as x(2)= aiee Jestsn(e i=2*) pr) where T= Onl, f@=hew and @% = aa. ‘Then, “a! Ve © sin(EVI=G") ag By using MATLAB’s symbolic toolbox and Eq. (D.12) in Appendix D, we find that x(2) “que nel eo ae ~2agsin(rVi=<* he ~w) |e) K(1+@2 -2a£)Vi-¢* -g? where When a = 0, the results reduce to that of a single degree-of-freedom system subjected to a unit step function given by Eq. (6.25). 6.6 Repeat Example 6.5, when the step change in road elevation a is 4 em and the vehicle speed is 100 km/h. Solution 6.6 For the given vehicle speed, we find that fo = (160 m)x(3600 s/hr)((100%10* m/r) = 5.76 s. Then, the displacement response is 153 ‘©2009 Cengage Learning. ALL RIGHTS RESERVED. Solutions to Exercises — Chapter 6 ") sin(w, (¢-1,))u(t—t,) een) in(e, (t-1,) +s) -t,) 0.3580, 8 358191-5) sin (17.81 (5.76) u(t —5.76) vVi-0358" asta 0K-578) $0.04) | — Jom sin (17.81 (¢ - 5.76) + 1.21) |u(t - 5.76) ( vi-0.358" (781¢ J | J = 0.0316" sin(17.81(1-5.76)) u(t 5.76) +0.04 0.04286 °°" sin(17.81(r-5.76) +121) m The results are plotted below. 85 6 te) Figure £6.6 ‘MATLAB program used to generate figure for Exercise 6.6 xdis=inline( /sqr(1-22)texp(-z*wntt).*sin(wm* sqrt(I-22)*t+phi 105.76; 2-0.04; phi 1.21; wn=19.07; 20.338; ‘linspace(5,7,250); xt-(2*atz*xdis((-10,z,n,0)+a*(|I-xdis(t-0,2,n,phi)).*(=t0); plot(txtk-'{5 Tha a ylabel(x() inold on 154 ©2009 Cengage Learning. ALL RIGHTS RESERVED. Solutions te Exercises ~ Chapter 6 xlabel(t (5), fontsize,14) set(gea,'fontsize’ 14) reaxis; | 1)=-01; | axis(r) 6.7 Refer to the Kelvin-Voigt-Maxwell combination shown in Figure E6.7. Obtain an expression for the displacement response in the Laplace transform domain of the mass when the mass is subjected to a step-function force of magnitude F. 6. Fora step function force, Solu SO = FRO The Laplace transform is given by pair 6 of Table A.1. Thus, F(s) Then Eq. (a) of Example 4.7 becomes, after a change in notation, mE cH heh (x-2,)= FO a, k(x-x,)=¢ Se i(x— x)= 67 or, using nondimensional quantities, E+ 2GK4(l+y)x-yx, YE 74 = Shy where and the over dot indicates the derivative with respect to x. Taking the Laplace transform of these equations using pair #2 in Table A.1 of Appendix A gives (s° #254147) X()-7X,(8) = G(s) 1X(3)-(7 +2783) X,()=0 where it has been assumed that x,(0) = 0 and £ Gs) = FO 626 +5)x(0)450) = when the initial conditions are zero. 155, ‘©2009 Cengage Learning. ALL RIGHTS RESERVED. Solutions to Exercises ~ Chapter 6 Upon solving for X(s), we obtain G74 2y7.S8 +(7 447.5") s + (27S (147) +267) s+7 The result can be compared with equation (h) of Example 4.7with c = 0 and by noting that while ¢-» 0, 2%. 2¢, where in the present notation, 2¢= cy/(ma,). X(s)= Section 6.4 6.8 A machine system of mass 30 kg is mounted on an undamped foundation of stiffness 1500 N/m. During the operations, the machine is subjected to a force of the form shown in Figure E6.8, where the horizontal axis time ¢ is in seconds and the vertical axis is the amplitude of the force in Newtons. Assume that the machine system is initially at rest and determine the displacement response of the system. tion 6.8 ‘The governing equation of motion of the system can be written as mi+ke= f(t) where m = 30 kg, k= 1500 N/m; hence, @ = 7.07 rad/s. The forcing is Se (t-1,)-ut—1)] ! FO=F-[u—ule=t,)]- 7 ite where, from the figure, F,, = 2500 N, f, = 0.5 s, and 4; = 2.0s. If we let @pl, then we can write the above equation as f SO) =F [u(e)—uee—1)]-F, u(e—,)—u(r -1))] Upon rearranging this result, we find =F yy—-— eh = 7 7 7 FO)= Fate) meee 1,)u(t—1,)+—*—(r 1, )u(r—F,) Then, from Eq. (6.1b), with ¢= 0 FE k x0 [2 foor-pemerag— | y hiner (Ene rae a(t, -F) 9 A fomce—sne-a)ue-nys] From the discussion following Eq, (6.36), this equation can be written as 156 (©2009 Cengage Learning, ALL RIGHTS RESERVE] Solutions to Exercises ~ Chapter 6 u(t—t,) * rn -De SG) euaeea Mean) “singer, -5eas| [2 Joiner -gae — mo Nye) MED eyo MEDDe-s)| a 1,(7-t) iF where Wa) Using the given parameters, we have that 7 = 7.07%0.5 = 3.64, 7 = 7.07*2.0 = 14.14, and, therefore, 2500/ u(r u(r -3.54 u(e-14.14 (Fr) BOO HO) HERD Hea yo MD eg 19| =1.667[0.2825h(r)u(r) - 0.0266h(r —3.54)u(r —3.54) 40.0943A(r —14.14)u(r—14.14)] -sint 6.9 In Example 6.7, assume that the motions of the slab floor are damped with the damping factor being 0.2. Determine the response of this damped system for the forcing and system parameters given in Example 6.7. Also, find the earliest time at which the ‘maximum displacement response occurs. Plot the results. Solution 6.9 From Example 6.7, we have that F, = 1000N, k =2x10° N/m, m = 1000 kg, @ = 20 rad/s, and f, = 1 s. Then @yf = 20, = 19.6 rad/s, and ¢é, = 0.2x20 = 4 rad/s. Thus, from Eqs. (6.37) and (6.38) we obtain, a= [ul -H—1utt-1)] 2000 faut) Mt —t.att—1.)} mo =S[h(u()— het, )u(t—1,)] mm where 157, ‘©2009 Cengage Learning. ALL RIGHTS RESERVED. Solutions to Exercises ~ Chapter 6 2 +a,t+e [acento in wa} 1 Fe 2x02?-1., =5-4-2x0.2+201+e™| 2x 0.2cos(19.61) += a [ (19.6) vi-0.27 0.02 +1 +.0.05e“"[ 0.4cos(19.6) - 0.939 sin(19.61) ] The results are plotted below. The earliest maximum value occurs at f= 1.2854 s where the value of the displacement is 5.272 mm, 6 . PAS / ‘ ‘a ts) wcramimariauatTkt — Fo=1000; k=200000; wn=20; teo=1; 2-0; wd=wnt sqt(1-2°2); inspace(0,2,400); €=1000*F ok; xt-c*(ht(wn, wd.z,t20)-htt-t20,wn,wd,z.120).*(0=t20)); plot(xt'k-\(0 2}, ylabel(x() label(t (6 set(gca,fontsize,14) opt-optimset(Display’joff); [tmin xtmin]fminbnd{@floorslab2, 1,1 4.opt.c,wn,wd.z,t20); ddisp((Earliest _max =" num2str(-xtmin) "mm at t=" num2str(tmin) 's') hold on | plt{min min 0 -xtmin) 14) 158 ©2009 Cengage Learning, ALL RIGHTS RESERVED. Solutions to Exercises ~ Chapter 6 text(imin,-xtmin®1-05,'x_{max}", fontsize’, 4, HorizontalAlignment, Right) text(tmin'-05,0.2,_{max}'Yontsize’14,'HorizontalAlignment Left) function zxht(t,wn,wd,z,tz0) zx (-2*z+ wnt exp(-z* wnet).*(2*z*cos(wd*t)#(2*2"2-1)¢sin(wd*th/sqri(1-2°2))/(wn*tz0); function m=floorslab2(t,wn,wdz,tz0) m=-c*(bt(,wn,wd.z.tz0)-i(t wn, wd, tz0).*(>~tz0)) 6.10 Consider the Boltzmann sigmoidal function, whose general form is given by S(taybja,r,)=a+b(14e"?) where a, b, a and 7 are constants and S(t, a, b, a, t) = a+ 6/2. This function can be used to create a step-like function as shown in Figure E6.10 for S(t 0, 1, 60, 0.1), Determine the response of a system for (2) = F,S(, 0, 1, 60, 0.1) and f(z) = F,S(r, 0, 1, 6, 1) and graphically compare them to the response of the system to the step input given by Eq. (6.23). The solutions have to be obtained numerically. Solution 6.10 60, «,=0.4 Figure E.10 MATLAB program used to generate figure for Exercise 6.10 function Exercise6_10 alp={60, 6]; to={0.1,1]; 0.25; 7-0. linspace(0,¢,600); 159 ‘©2009 Cengage Learning, ALL RIGHTS RESERVED. Solutions to Exercises ~ Chapter 6 for k=1:2, [ttyy}-odes5(@od,{0,35],(0 0},1,2,0,1,t0(k),alp(k));, plot(ttyy(:,1),col(k:)) hold on 142,[0,35],(0 O}.0.2); xlabel( size 14) ‘ylabel(x(\tau)/F_o''fontsize',14) [a, bl-legend(("alpha-" num2st(alp(1) tau_o=" ‘num2str(to(1)). Crap num2strto@2))|S set(b(1),'fontsize’,14) function hh~Sigmoid(a,b,t0,v0) hh=atb /(1+exp(vo*(to-t))); function yy=odtt.y,7,,b,to,v0) yO) 2*24¥2)}-y(1 10id(.a,b,t0,v0)]; funetion yy=od2(ty,2) yyrty(2)s-24z4y2)-y()+ Section 6.6 6.11 Determine the response of the damped second-order system described by H() + ci() + a(t) = FO to the rectangular pulse shown in Figure £6.12. Plot the displacement response for m = 1 kg, $= 0.1, a= 4 rad/s, fy = 1 N, and t, = Solution 6.1 Using Eqs. (6.52), (6.56), (6.57), and (6.8), we have x)= Lfecou— st )ut-)] where sin(@,t+ 9) From the given values, 160 (©2009 Cengage Learning. ALL RIGHTS RESERVED. Solutions te Exercises ~ Chapter 6 rad $0, =0.1x4=0.4 rad/s ff. 19.9625 m k mon 1x# an. 2 4 6 8 10 1@) Figure £6.11 ‘MATLAB program used to generate figure for Exercise 6.11 wn=4; 70.1; t1= vwd=wn*sqrt(1-22); phiratan(sqrt(1-2"2)2);, Aefolm/wn’2; ‘gt-inline(1-exp(-z* wn*t).*sin(wd*tphi) tlinspace(0,10.200); Xt=A4(at(twn,wd,z,phi)-at(t-t1,wn,wdz,phi).(>=t1)); fo=1; 161 (©2009 Cengage Learning. ALL RIGHTS RESERVED. Solutions to Exercises ~ Chapter 6 6.12 Plot the energy density and then determine the bandwidth of the following pulse: f() =0.5(1-coso,t)[u() -u(t-27/@,)] Solution 6.12 Let f= ata, then F(0)=0.5(1-cose,t)[u(t)—ut-1,)] =0.5[u(t)—u(t-1,)]-cose,t[u(t) —u(t-1,)] We notice that os @, (¢-2/@,)]= [o(~-4)] Then FO=R(-hO where (= 0.5[u()—uer~1,)] A(t) =0.5cos(a,) u(t) —0.5cos[a, (t—t,)]u(t-1,) Taking the Laplace transform of /(#), we find that F(s)=G(s)-H(s) where, from pair 8 of Table A, and from pairs 3 and 18, 0.58 H(s)= Then, [Note: This result could have been obtained directly from the definition of the Laplace transform by performing the respective integrations] 162 (©2009 Cengage Learning. ALL RIGHTS RESERVED. Solutions to Exercises ~ Chapter 6 The magnitude of the amplitude spectrum is obtained by making the substitution s = jo and then determining the magnitude of this quantity. Thus, ) 05 al(1-0°/0;)) [1-2e0ser, +c0s* or, +sin* or, |” which can be written as 4, \FUa)|= 3} ‘The cutoff frequency @, is determined from the value of the ratio @/a, that is a solution to 1 1-22 /o,' in(x@, /,) F Uo.) 0,10, 1/2 ‘After using the MATLAB function £2ero, we find that @/@, = 0.720. ‘The plot of the amplitude density is shown below. 163 ©2009 Cengage Learning, ALL RIGHTS RESERVED. Solutions to Exercises ~ Chapter 6 05) 04s} 04] 0.35} “I 0.5} Foyt) 5 on .05| — = 05 1 15 2 Figure £6.12 ‘MATLAB program used to generate figure for Exercise 6.12 Om=Hinspace(0.01,3,101); F-{0.5 0.5*abs(sin(pitOm).(pi#Om)).*abs(1 (1-Om/2))} plot((0 Om),F,k-) xlabel(\omega/iomega 0, fontsize'14) ylabel(|F(jiomega)'Of'fontsize’4) 164 (©2009 Cengage Learning. ALL RIGHTS RESERVED. Solutions to Exercises ~ Chapter 7 Solutions to Exercises Chapter 7 Section 7.2.1 7.1 Consider the “small” amplitude motions of the pendulum-absorber system shown in Figure 7.9 and derive the equations of motion by using force-balence and moment- balance methods. TA The free-body diagram of the system is shown at the right, along with the inertial forces. We have used Eqs. (f) of Example 1.1 me to construct this figure. The summation of the = fo forces in the horizontal direction and the sum of | +=— ae the moments about the pivot of the pendulum gives, respectively, Solu | -m Wdsino +800) I 1m i-+m,i+m, (Lb cos0— 16" sind) + ‘mt +1 (LBcond— 14" sind) ke=f) m, 06 +m,kLcos0 +m,gLsin@ =0 mi+m,i-+m, (L6.cos0- 16" sind) + k=f) m,L6 + m,iLcos0 +m,gLsin 0 =0 Considering ‘small” angular oscillations about the equilibrium position, we obtain (m, +m, )¥+m,LO + ke = f(0) m,L6 +m,L +mgL0 =0 which agrees with Eq. (h) of Example 7.7. 7.2 Derive the equations of the hand-arm system treated in Example 7.11 by using force- balance and moment-balance methods for “large” and “small” oscillations about the nominal position. Solution 7.2 The equations of motion will be obtained by carrying out a force balance along the horizontal direction for masses my and m2, a force balance along the horizontal and vertical directions about the center of mass of the rigid body m3, and a moment balance about the pivot point at the lower end of mass ms. The free-body diagram for the masses ‘m,, m2, and ms are shown below, along with the inertia forces and moments. 165 ©2009 Cengage Learning, ALL RIGHTS RESERVED. Solutions to Exercises — Chapter 7 toca Aix) ~~ kay), kG) «(aa | es) slim) ka (0 +) +en6, The governing equations of motion for large os From kinematies, we have lations are obtained as follows. 5, = 8, ~ 1,6, sin(9 +0.) - 1,6; cos(0 +.) Say = 55 + L,8,c05( +0,) - 1,6 sin(5 +0) x =%+L008(0+0,) and 5, =%,-16,sin(O-0,) y= +Lsin(G+0,) and J, = 3, + 10,c05(0+0,) Force balances along the horizontal direction for the masses m; and m, lead to mi, + (ky +k) x, hax, + (c+ ey) 5% gt, = hx, +e, @ imi, + (by +) 44 kam — es +(e, +65) yep eps =0 * Considering mass ms and carrying out a force balance along the horizontal direction and a force balance along the vertical direction results in m,| 8, ~1,6,sin(4-+0,) ~ 1,63 e0s(8+8,) |+(k, +k) +k Leos(9 +8,) + (c, +e) esx, ~e,0,Lsin(8 +0, a ae oe © m,| 5i,+1,0,c0s(0+0,) -1,6% sin(O +0,) |+k,| », +2, sin(+0,)] +65[ 5, + £,0,008(6 +8,) |+mg =0 ky Carrying out a moment balance about the pivot point at the lower end of the mass m3 leads to Iygbj — msi gL, in (5 +8,) + m,5i gL, 008(0-+0)) +k, (9+8,) + c,.0, —k,x,Lsin( +0,) —¢,i,Lsin(0 +6.) +k,y,Le0s(0+6,) ) +e,j,Lc0s(5 +0.) +mgl, cos(6 +8) =0 166 (©2009 Cengage Learning. ALL RIGHTS RESERVED. Solutions to Exercises — Chapter 7 After substituting for ¥,,,s.,%s4,Js» and j, and rearranging terms, we obtain [Ye + mE; J6, +m, [-X, sin(6 +0) + 5,005(0+6,)]+k,(0+8,) ~kLsin(9-+6,)| x, + Leos(9+0,)]+k,Lc0s(4 +6,)| y, + Lsin(d+0,)] +¢,6;~cLsin(0 +0,)] %, - 16,sin(d+0,)] +e,Lc05(0+0,)| 3, + 10, cos(d +0) ]+mgb., cos(3-+8,) =0 © Equations (a), (b), and (c) are the equations of motion for large oscillations. Linearization and governing equations of motion for “small” oscill: We now linearize the nonlinear system of equations given above about the nominal position to describe “small” oscillations about this position. To this end we use the ‘Taylor series expansions sin(9+0,) ~sind +0, cos cos(d+0,) = cos-0, sind ‘Making use of these expansions in the nonlinear equations of motion and retaining only linear terms, we obtain the following system of linear equations that can be used for smail oscillations. mi, +(k, +k) a + (6, +6) Gt, = hx, ted, —C,% =0 mks + (ly hy) % hax hax +(6, +6) Gy kL, sind Loos6 1,6, sin |+(ky +h.) 45% +) 3% —G%, —¢,L6, sin m,[ 5, + 1,0, 0080 ]+k.[ y, + £8, c0s0] +c,{ , + L8,cos0|=-m,g—kLsind ind (Jag mF; )6, + mL, [- j, 0080 | +k,,0, —kyLsind| x, - 10, sind | 20, 00s? 8+ k,L.00s6| y, + L0, cos |- 170, sin” 8 +00, —c,LsinO[ x, - 18, sind |+ ¢,Lcos6[ J, + £0, cos |-m,gt,0, sin = —mgl, 0080 —k, 6 +(k, -k,) 2 sin cos ‘These equations can be assembled in the following matrix form 167 ©2009 Cengage Leaming, ALL RIGHTS RESERVED. Solutions to Exercises — Chapter 7 m0 0 0 0 i 0 m 0 0 0 ¥ 0 0 m; 0 mL, sin |} x, b+ 0 0 0 m, mL, C088 || 5, 0 0 -mL,sind m,L,cosd Jy, +m? ||6, G+, -C, 0 0 0 x, “Gt 0 offs -¢, ete, 0 ~c,Lsin8 |) b+ 0 0 0 & c,Leosd || j, 0 0 -c,Lsind c,Lcosd C55 6, kth ky 0 0 0 x kx, tex, -k kth -k 0 0 lia, 0 0 -k, kth, 0 -k,Lsin6 |} x,4=4 -k,Leosd 0 0 0 ky k,Lcos6 || y,| |-myg—k,Lsind 0 0 6 6 ky |18 6, where 65 =¢5 +¢,17 sind +c,1? cos gg = ky +E (sin? O—cos* 8) + kL? (cos* O-sin® 6)—m.gh, O, =-m,gl, cos —k,,6 +(k,—k,) E sind cos It is clear from these equations that the system inertia, damping, and stiffness matrices are symmetric matrices. These equations form a system of five second-order ordinary differential equations that can be used to study “small” motions about the nominal position 0. It is noted that this position is not an equilibrium position, but rather an operating point where 8=0. It is noted also that depending on the application, linearization of a nonlinear system may need to be carried out about a reference position different from the equilibrium position. 7.3 A container of mass me is suspended by two taut cables of length L as shown in Figure E7.3. The tension in the cables is 7,. Inside the container, a mass m is elastically supported by a spring k. a) Determine the equivalent spring constant for the cable-mass system and sketch the equivalent vibratory system. b) For the equivalent system determined in part (a), determine the equations governing the motion of this system. 168 ©2009 Cengage Learning, ALL RIGHTS RESERVED. Solutions to Exercises ~ Chapter 7 Solution 73 (@) Let the mass m. move only in the vertical direction by an amount xy. If )ry/Z| << 1, then from Eq. (d) of Example 2.8 the ‘equivalent spring constant for this two cable combination is 2r L The vibratory system can be modeled as shown at the right. k, (©) For oscillations about the static equilibrium position, the equation of motions are obtained form Eq. (7.1a) by noting that kt = keg, C1 = 02 = = 0, ke = k, my = mg and m, =m. ‘Thus we obtain im. + (ke + KX, — hey mK, + ey — ko, 7.4 A two degree-of-freedom system with a nonlinear spring element is described by the following system of equations: mk, +x, +x; +k, (x, =x) + a(x, — x) my +h (x —) + hax, —%)" Determine the system equilibrium positions. Assume that m, mp, ki, and ky are positive and qis negative. Solution 7.4 The system equilibrium position (x10, x20) is a solution to the algebraic equations Bi, + 0% + (Sy ¥p.) +a, —¥,)' =O Ka (x, — x.) +h (a —%) = 0 From the second equation, we see that xip= 2x2 Therefore, from the first equation, we obtain Kx, tax, =0 which leads to Since @ < 0, we have two additional equilibrium positions apart from the trivial equilibrium point. ‘Thus, the system has three equilibrium positions. 169 (©2009 Cengage Leaming. ALL RIGHTS RESERVED. Solutions to Exercises ~ Chapter 7 Section 7.2.3 7.5 Derive the equations of motion of the systems shown in Figures E7.5a and E7.5b and present the resulting equations in each case in matrix form. Solution 7.5 ‘The governing equations of motion can be obtained by using either force balance or Lagrange’s equations. (@) The system shown in Figure E7.5a is a special case of the system shown in Figure 7.Ja, and the solution can be obtained directly from Eqs. (7.5b); that is, the mass, stiffness and damping matrices are given by ‘m 0 0 [M]=|0 m0 0 0, kth -k 0 [K]=| -b bth Wh ° =k ky thy qty, (0 [l=] -a ate -e 0-4 Gty and the governing equation of motion is ¥ 4 x) (0 [v]} ¥ p+1C]})% $+[K]y p= 40 % ES fe (0) where x1, x2, and x3, are used to represent the displacements of masses m, m2, and ms, respectively. ‘The equations can also be obtained using Lagrange’s equations. The kinetic energy, the potential energy, and the Rayleigh dissipation function are, respectively, eee Le im? +imi +m 2 2 2 12,1 2, 1 2,1, 2 V =—kx? += (x, - x) +k, — =k, ph +h my + pha) +k ereees sa sscssateaeasmaasess liz sail +7044) + eC 8) + pe From Eq. (7.7), and noting that we do not have any extemal forces, the Lagrange equations are 170 (©2009 Cengage Learning, ALL RIGHTS RESERVED. Solutions te Exercises ~ Chapter 7 which yield mi, + hx, +h) — x) +64 +68, -4) = 0 44) + (4 — 4) A — 2) + Ga 9) = 0 gy — 05 —%s) + Cah — hy — 5) + ek = 0 ms, ~ O,(%, ‘These equations can be rewritten as =0 IMS + (Cy + C3) ¥q ~ Cok, — Cys + (hy + Ke )Xa — bax Bx = 0 Ms, + (Cy + C4 Vhs — Cha + (hy + hy )X5 — hgh = 0. my + (ky + hy) — baXy + (6, + 05) — C2%y Placing these equations in matrix form gives the same form as that shown above. (b) For the system shown in Figure E7.5b, we again use x), x2, and xs, to represent the displacements of masses my, mz, and ms, respectively. Then, the kinetic energy, potential energy, and the Rayleigh dissipation funetion are determined to be 2g dy tg? mai + maka tym iA Eg (s a) Fh a) +S 5) 4 kd D=teai +e -ay ey # Using the Lagrange equations given in Part (a), we obtain mix holm) h(a) Heh, Healy —H) =O m%, ~~) +h, — 4) = 0 mis — 64 ~ ¥3) + Cs ~ hy (Xp — %3)— hy = ¥4) + kX = 0 ‘These equations can be rewritten as Imi, + (+ ky + ka) — fata — has +(G, + ee) ee: MS + (hy + hy) — kaX — bX (ky + he) hots — kya =0 yi +(e, +65)%5— Putting these results in matrix form yields m1 (©2009 Cengage Learning, ALL RIGHTS RESERVED. Solutions to Exercises — Chapter 7 m 0 0 [¥]=|0 m 0 0 0 m, Kthth Wk ky [K]=| -h thy hy ak, mk kth th, 4 0 ete, and the governing equation is of the form ¥ % x) (0 [M]}% p+[C]}% p+[K]} 5} =40 Ea a x} [0 716 Derive the equations of motion for the model of an electronic system mp contained in a package mi, as shown in Figure E7.6, and present them in matrix form. Solution 7.6 Let x; and x2 be measured from the static equilibrium position of masses m and ma, respectively. Furthermore, let x3 be a prescribed displacement. ‘The kinetic energy, potential energy, and the Rayleigh dissipation function are, respectively, T= jms + imi? Kasay thay (3-8) + LeG 4) From Eq. (7.7), and noting that we do not have any extemal forces, we obtain the Lagrange equations $(Z) ar aD av 2(S)-2 4, at\ a, ) a,” ai,” ax, pai ft late tad ioe at\ a) ex,” Gi which yield mB — G(s — 8) + 6% — Hp) — hy(%3 — 41) +H —) = 0 myXy ~ (4, — %)— ky —)=0 172 (©2009 Cengage Learning. ALL RIGHTS RESERVED, Solutions to Exercises ~ Chapter 7 These equations can be rewritten as mi, + (ky +b) bax + (6, +.) ny =e HS Imps + Gy, Cp + 2% Hk, = 0 Putting these results in matrix form yields m 0 |f%, el kth, kh) f% | qte, -@] fm] _ fat thx 0 mile) Lh ble Be Gilalial: 20 Note that gravitational loading has not been considered, because oscillations around the static-equilibrium position have been assumed. 7.1 Derive the equations of motion of the vehicle model shown in Figure E7.7. Solution 7.7 Assuming that the displacements yi, y2, and ys, are measured from the static equilibrium and y, is a prescribed displacement, the kinetic energy, the potential energy, and the Rayleigh dissipation function are, respectively, Pasms?+tmjt+tm,je pms tymaia ty msIs al ol 2 Vash.) +g han) +7 ha — I) 1 ‘ 1 . 1 2 D=zah.- I) +zAa— HW) +7) From Eq. (7.7), and noting that we do not have any extemal forces, we obtain the Lagrange equations a which yield m3, — Ks) adh m,3y a. — 2) + Ka — Ys) + x2 ~ Ya) ~ Ca Fe = Ia) + Ga — i) + Da — Ir ms — kya — Ys) a — Is) = ‘These equations can be rewritten as 173 (©2009 Cengage Learning. ALL RIGHTS RESERVED. Solutions to Exercises ~ Chapter 7 my, + bY, — kyr + O91 — 9, =0 MS. + (ky + hy +h) Va yy kaa + (Cp +0, + )P2 — OH Css = He HOI. mJ, + ks — kya t+ Is — C552 =0 7.8 Derive the equations of motion of the pendulum absorber shown in Figure E7.8 for large oscillations and then linearize these equations about the static-equilibrium po: corresponding to the bottom position of the pendulum. Present the final equations in matrix form. Solution 7.8 Let x be measured from the initial position of the system. The position vector of the pendulum mass from point O and the corresponding velocity vector are, respectively, 1, =—lsingi+(x-leosg)j -Ipcosgi +(+/@sing)j ‘Then, the total kinetic energy is 7 1 ui? +1 m(y, Cy, ) 2 2 Pah 1 2 1 2 Me + jm[6* cos’ 9 +. Fe +m)x + imi +mligsing ‘The total potential energy is and the dissipation function is ‘The Lagrange equations are 174 ©2009 Cengage Learning, ALL RIGHTS RESERVED. Feit +} ml (-Iacoso)' + (3 +19sing)] ? + 2ilpsing +1°¢" sin’ ] kx? +mg(x—lcosp)+ Mgx ec +(M + m)gx—mgleosp Solutions to Exercises ~ Chapter 7 2/8 (ar) or dt\ a) a& d (Z) er , @D 0G Noting that Q, = F,cosoy, the first of the Lagrange equations gives £((M +m) mip sing) 0-68 +k g(M +m) = F,cosat (M+m)x+ml[dsing + cose ]+ci-+ kx = F, coset ~ g(M +m) Since Qo= 0, the second Lagrange equation gives {(ni"p+miising)—mispcoso + cp +mglsing =0 mG + mlising + mlig cose - mlig cose + ¢,@ + mglsing =0 ml’@ + miising + c+ mglsing =0 Static-cquil Po: ns, At the static equilibrium position (xo, g.), the above equations reduce to kx, =-(M + m)g ml sing, =0 We see that the static-equilibrium positions are =—temg. : k 9. =0 ort Linearization Let x(t) =x, +3) P=, + 90) Then for oscillations about g, = 0, #0 =80, xH= 380 PO=9O0, $= sing =sin(y, +9) =sing, + pcose, = cos = cos(, + 6) = cose, —Gsing, =1 By using these expressions in the governing nonlinear equations and only retaining the linear terms, we obtain 175 (©2009 Cengage Learning. ALL RIGHTS RESERVED. Solutions to Exercises — Chapter 7 (Mtm) ite +h(x, +2 E,cosot—(M +m)g mPO +6,0+mglp =0 After using the static equilibrium value for x,, we arrive at the following uncoupled linear equations in matrix form [lle SHl-Lo onlls 5") 79 Derive the equations of motion of the system shown in Figure E7.9 by using Lagrange’s equations. Solution 7. Ix and x) are the absolute displacements of the masses m; and ma, respectively, the total kinetic energy is al) jmat rhs The total potential energy is 1 Jagat 2 2 lbh at +2 hai) 1 i +p hGi—ny thi my and the dissipation function is 1 2 Das +e)i Since there are no external forces, the Lagrange equations are which on evaluation leads to. Emi) 08 (6 He 41h +s HO +R) =0 mi, + (6, +e) + (ky +h +h) (hy +h) =0 176 (©2009 Cengage Learning. ALL RIGHTS RESERVED. Solutions to Exercises — Chapter 7 d ‘ alata) 0+ 0 (hy + h(a) =0 mss + (ky the, Uh +h) =0 In matrix form, we have [Flee ole SIH Notice that the system shown in Figure E7.9 is equivalent to that shown in Figure 7.1 with c replaced by c; + e2 and c2 = cs =0, ki replaced by ky + ky, and kp replaced by ky + ks. 7.10 Replace each of the linear springs ty and ks shown in Figure E7.9 by a nonlinear spring whose force-displacement characteristic is given by FQ) =k(x+ax") and determine the resulting equations of motion. Solution 7.10 For this nonlinear system, the kinetic energy remains the same as that obtained in the solution to Exercise 7.9, while the potential energy obtained in the solution to Exercise 7.9 is modified to 1 re Pre 21 coos 1 4 al + Shes +S a) +p 2) + Gs) = pak 5) 2 HF rR Eh +N + hath, +h Xen) where we have used Eq, (2.24). Using the Lagrange equations given in the solution to the preceding exercise, we obtain the following coupled nonlinear equations: mi, +(6,+)3, +k +h, +h +h), (kh thy +a(k, + Cam) =0 imi, + (ky +), Uh +h)s, a + YG)" =0 7.11 Derive the equations of the milling model shown in Figure 7.4b by using Lagrange’s equations. Solution 7.11 If the displacements x1, x2, and x3 are measured from the static equilibrium position, the kinetic energy, potential energy, and the Rayleigh dissipation function are, respectively, 177 ©2009 Cengage Learning. ALL RIGHTS RESERVED. Solutions to Exercises ~ Chapter 7 Tama} pmsl Sm eo 1 2 phe + Shs) +k a) Lely 2 Fei + Fea) + pe a) 2 and the generalized forces are Q=-fO and O()=Q()=0 From Eq. (7.7), we obtain the Lagrange equations a(or)_ar aD wv fal (oa ate eDaora at\ @,) a,” a, * ae, which yield R(x, 4) - 4% - 4) =-hO IMB — Gy — 4) + 0% ~ 41) ~y ( —,) +(H—) =0 my, + esi, + 6,8, — 3) Hex, +h (a, —m)=0 After rearranging these equations, we obtain Eqs. (a) of Example 7.1. Note that gravity loading does not appear in the equations of motion, since motions about the static equilibrium position have been considered 7.12 Derive the equations of motion of the system shown in Figure E7.12, which is an extended version of a two degree-of-freedom system discussed in Section 7.2. Let M.(0) be the external torque that acts on the disc whose motion is described by the angular variable gy. Solution 7.12 In terms of the given coordinates, the kinetic energy, potential energy, and the Rayleigh dissipation function are, os ready, id +5 Fai +4 L168 i huni +h hale, 9.) + Fhe, 9.) pe erg a eige ent +3 ead +5 ea 178 (©2009 Cengage Learning. ALL RIGHTS RESERVED. Solutions to Exercises ~ Chapter 7 and the generalized forces are Q,=M, and Q,= From Eq. (7.7), we obtain the Lagrange equations afar) ar dt 86, which yield Jari + ky, + Kal P, P12) + G0 = M,, Jo2P2— kal Pr~ 92) + bisa — Ps) + Car Tess — kes P2— Ps) + CPs =O ‘These equations can be rewritten as JB + Cu; + Fh + ka) —ha?s = M, JoaPr + COs + ha + hi). — hay ~ ks = oss + 604+, —kyQ, =O 7.13 The experimental arrangement for an airfoil mounted in a wind tunnel is described by the model shown in Figure E7.13. Determine the equations of motion governing this system when the stiffiness of the translation spring is &, the stiffness of the torsion spring is ki, G is the center of the mass of the airfoil located a distance / from the attachment point 0% m is the mass of the airfoil, and Jc is the mass moment of inertia of the airfoil about the center of mass. Use the generalized coordinates x and @ discussed in Exercise 1.19. Solution 7.13 We assume that a displacement along on the upward vertical direction is positive and a counterclockwise rotation @ about positive. Then the location of the center of mass from O is -Icos6i+(x—Isind)j r, and the velocity of the center of mass is 6sinOi+ (%-16.c0s6) j ‘Then the kinetic energy is 179 ©2009 Cengage Learning. ALL RIGHTS RESERVED. Solutions to Exercises ~ Chapter 7 6+(#-lbcose))) SO + zm(Po sin? 0+ i? - 21:6c0s0 + /°6* cos’ 8) (0 +hm(Po +5 -21%0c080) and the potential energy is kee hie +mg(x-Isin@) 2 Noting that we do not have any damping and any extemal forves, the Lagrange equations are a) EE ai\ a) ex” x 4 (zr) ar, i(Sa}- a6" 06) 00" a0 On substituting for T and V and evaluating, we obtain “( i - mlOcos0) —0+ ke-+mg=0 (6+ mI°6 — mli-cos@) — mli@ sin 8 + k,0 — mglcos0 = 0 or mit — ml cos0 + m16" sin + kx =—mg (Jo +ml?) 6 -micos 0 + k,8 = mglcos8 7.14 A multistory building is described by the model shown in Figure E7.14. Derive the equations of motion of this system and present them in matrix form. Are the mass and stiffness matrices symmetric? Solution 7.14 Making use of the given coo respectively, tates, the kinetic energy and the potential energy are, eee lees myig + — m3 +m + maa 2 2 2 af +a (sm) +4 (m5) +h (35-2) Since there are no external forces, the Lagrange equations are 180 ©2009 Cengage Learning, ALL RIGHTS RESERVED. Solutions to Exercises ~ Chapter 7 d(ar) ar a, _4 at\ a) ax, a, ax, ar , aD , ov ar,” i,” Ox, which on evaluation leads to mi, +2kx, + 2K (om — xy myx, — 2k, (x, —X,) +2 (x — 2, mi 2k, (x, —4) + 2k, (2, -¥,) =0 m,&,—2k,(x,-x,)=0 ‘These equations can be written as m8, +2(k, +), —2kyx, = 0 Img +k + Ky), — 2kgx, — Why, img, + 2(k + hy) —2kyx, — 2k gig + kx, —2kx, =0 Putting these equations in matrix form, we have m 0 0 0[%) 2K +k) ky 0 0 V[m)} fo 0 m 0 O}f%{ | 2 2, +h) 2 0 IIx, _ Jo} 0 0 m O]}x, 0 2k, kth) -2k |}, Jo 0 0 0 miley 0 0 2k, 2k, |x lo ‘We see that the mass and stiffness matrices are symmetric. 7.15 Obtain the governing equations of motion for large oscillations of the system shown in Figure £7.15 in terms of the generalized coordinates x;, x2, and @ The spring ks is attached at the midpoint of the bar. Linearize the resulting system of equations for “small” motions about the system equilibrium position and present the resulting equations in matrix form. Point G is the center of mass of the bar. Solution 7.15 Let the distance from the center of gravity to the point where the spring ks is attached be Zs. Then, 181 ‘©2009 Cengage Learning. ALL RIGHTS RESERVED. Solutions to Exercises ~ Chapter 7 Lt 2 2 The total kinetic energy and the potential energy are given by Lah Noting that the system is undamped and not subjected to any external forces, the Lagrange equations are (ar (ome van ‘Then, the first governing equation is Loma) +h (x, +L, sind) ~ k (L, sind ~ x,) +k, (x, + L,sind —x,)—mg =0 mii, + (ky +k, +k) x, — hye, + (hy, —kyLy + kyl) sind —mg =0 the second governing equation is Sms) bola +ysind-x,)—mg =0 my, + kX; — kx, ~ kL, sind—mg =0 and the third one is £(,6) +k L(x, + L,sin0)cos0 + kL, (L, sin — x,)cos0 + k,l, (x,L,sin8 — x,)cos6 =0 To + (kily kala + kyls) x, coSO—k,L,x, 0080 + (kL +k, L3 +k,L,)sindcos0 = 0 The static-equilibrium position is determined from the solution to thth Wk kh-bh+hh){ x.) [mg hy ky kl, Xoo f= MB kL -bhtkhl, kl, kh +R +kL)|sing,) (0 since the cos@, term cancels in the third equation. Although cos6, = 0 is a valid solution to the third equation, 0, = 7/2 is not physically acceptable, Notice that we consider that sin@, is the unknown, not 6,. The value of @, is then determined from sin’',.. 182 (©2009 Cengage Learning. ALL RIGHTS RESERVED. Solutions to Exercises ~ Chapter 7 To linearize the equations about the static-equilibrium position, we assume that nO=%+HO 2,0 = 4 200 A(t) = 0, + Ot) and note that, () J=12 WG sind =sin(0, +6) = sind, + Ocosd, cos0 =cos(0, +6) ~cos0, —Osind, We then substitute these relations into the three equations of motion and obtain mii, + (ky + ky +s) (2, +8) ~ (5, +42) + (hil, kala + yls)(sin 8, + 6.0058, ) imsk, +h; (2, +8) (x, + 4) ib (sin 8, +0080, )—m,g =0 Jeb + (ki, ~kala +hyb) (2, +%)( £086, ~B sind, )— kyl (2, +5,)( e080, Osi (ki +hL3 +k,B)(sind, +4050, )(cosd, -Asind, ‘After using the static-equilibrium equations in the above equations and reta linear terms, these equations simplify to (kil, ~ Habs + kyla) Bc080, =0 Mk, + k& — kB, —kyL,0 e088, = 0 SO + (kyl — kyla + kyl) 2, 0080, ~ k,Ly, 0080, +[—(k,L, — hgh; +k,L,) x. sind, +hyla%, in8, + (ky; + kya + kB) (cos? 0, — sin’ @,)|6 = mi, +(k, +k, +k) kk, In matrix form, these linear equations can be written as m 0 0 0 m 0 0 0 SSG k tk +k, k, (4b -kL +kl,)cos0, |(%,] [0 + ~b ky KL, 0080, gb={o (kL, kala +hyls)o0s0, —kL,0050, ks 6) 0 where key =~ (kil, ~ oly + bubs), Sin, + kL, sin, +(k, lh + k,B3 + k,L3) (cos? 8, sin’ 8, ) 183 (©2009 Cengage Learning. ALL RIGHTS RESERVED. Solutions to Exercises — Chapter 7 7.16 A pair of shafis are linked by a set of gears shown in Figure E7.16a. An equivalent system for the system shown in Figure E7.16a is determined as shown in Figure E7.16b, where J,=J,73, ,=k,Tj and the transmission ratio Tr = @y/@s. Determine the governing equations of motion of the system shown in Figure E7.16b by using the generalized coordinates ¢,, g,, and @, where 4, = 4,/T,. Solution 7.16 ‘The kinetic energy and the potential energy are, respectively, fag (J+ 5)@ +4163 1 ay lt ie av V=Fki(i- G2) +5 ha (A - 6.) For this undamped and external force-free system, the Lagrange equations are a (or oT aoa) a0," d(ar)_ ar at\op,) a6, 0 which, on evaluation, lead to (i+ 2)e IG.- a4 ) Upon rewriting these equations, we obtain SG, + kn he s+ Ja) Os +(Ku + hea) Or — kur, ha Tbe * ka, head, =0 Placing these equations in matrix form, we obtain, A 0 OV) kr ky O Tf y) fo) O Std, ON GF +) —ky Kathe he 130, p=30 0 0 FSG 0 -k, ky |e) (0 184 (©2009 Cengage Learning. ALL RIGHTS RESERVED. Solutions to Exercises ~ Chapter 7 7.17 As shown in Figure E7.17, a rigid weightless rod of length 21 is attached to a pivot at its center. At each end of the rod, a mass m is attached. To one of the end masses another spring-mass system is attached. A torque T(t) is applied to the rod at the pivot as. shown in the figure. Assume “small” oscillations and the gravity acts normal to the plane of the system. Obtain the equations of motion for this system. Sol 1. We use the coordinates shown at the right and assume that the rotation of the rigid rod is such that y/L| << 1. Then, the kinetic energy is Twat 4 tt + bn at 2 2 2 =4m(16)'+4m(-10)° + 4m 2 2 2 1 =mb6? +4 m2 2 and the potential energy is Ve ta iT (x, 2 2 1 ad ie = zh(Loy toh (LO x) ty kxt ‘The Lagrange equations for this undamped system take the form tal( OTe one OES a\06) 30° 30 ~* d(ar) or wv 42 |-2 +29, at\ a% ) Gx,” ax,” Noting that Qo = 7(¢) and Q, =0 and substituting for 7 and V and carrying out the different operations, we obtain 2mE6 + k,PO+kL(LO+x, mk, +k, (LO+x,) + kx, which can be rewritten as 2mLO +(k, +k.) 20 +k Lx, =T() mi, +(k, +) x, +10 =0 7.18 Consider the vibratory system shown in Figure E7.18, which consists of two masses that are connected by rigid massless rods. Determine the equations of motion of the system. Assume that gravity acts normal to the plane of the system. 185 (©2009 Cengage Learning. ALL RIGHTS RESERVED. Solutions to Exercises - Chapter 7 Solution 7.18 For ‘small’ oscillations, the rotation of the rigid rod can be approximated as Yay L Therefore, the displacement of the left end of the rod of length Ly is Ne=H hoa HP vn) (bth), b ( L ps _” ‘The potential energy is and the kinetic energy is, Lin gt adn 2 Tazmyi tymya Since there are no external forces, the Lagrange equations are After substituting for 7, D, and V in the above equations, we obtain my, +o, +k, (424) wok (G34), ao ; Jona my, + kay, —k, (24), fal In matrix form, we have 186 (©2009 Cengage Learning. ALL RIGHTS RESERVED. Solutions to Exercises — Chapter 7 seas] (eg SIH RH gy A BS z } ky +k (h/La) 7.19 Consider the vibratory system shown in Figure E7.19. a) If the connecting rod is rigid and uniform with a total mass m, then determine the ‘equations of motion for the system. b) If the rod connecting is rigid and weightless, then determine the equations of motion for the system. ©) If the rod connecting is rigid and weightless and the pivot is removed; that is, the rod can translate vertically and rotate in the plane of the page, then determine the equations of motion for the system. Solution 7.19 We shall use Lagrange’s equations to determine the governing equations of motion in all three cases. (a) From Table 2.2 and the parallel axis theorem [Eq. (2.1)], the mass moment of inertia of the rod about the point ‘O” is —— Li x Referring to the figure at the right, the kinetic energy is T= fms + hm + hoi ‘The potential energy is vad kal otha -x) +t + bale tay For ‘small’ 0, x, = Land xp~ 20. Then, the potential energy becomes VL (0) +4 (LO-x) +36 (L20)" + ky (L0-+)° where we have assumed oscillations about the static equilibrium position. Since we have free oscillations of an undamped system, the Lagrange equations are 187 (©2009 Cengage Learning. ALL RIGHTS RESERVED. Solutions to Exercises ~ Chapter 7 ar a oreo Ox, ex, or , av 2. ar," Or, “(3 | +f ai\26) 00° 30 from which we determine the governing equations as im, —k,(L0-x)=0 mi, +k,(L0+x)=0 Jo6 + ky L2O+ kyly (40 —x,) + kL30-+ kL, (L,0+ x) =0 which can be rewritten as mi, + kx, byl, 0 =0 mk, + kyx, + kyL0=0 SO +((ky +h) Ei + (ky +k) B)O-kLx, +k Lx, = (b) If the bar is massless, then we set Jo = 0 in the third equation of part (a) and obtain 0 =a, (kx, kx) where 1 (hth) E(k the Then the first two equations of part (a) become mi +x, ~byL, (ak Lx, ~akyL,x,) = 0 Img + ky + Kl (ayo, ~ kyly%2) = 0 (©) Inthis case, the kinetic energy of the system is, Tad ng? +d mi? 2 2 ‘The potential energy is (sx) + pst + ph (mtx)! Vata 2 ‘The Lagrange equations for this undamped and unforced system take the form 188 ©2009 Cengage Learning. ALL RIGHTS RESERVED. Solutions to Exercises — Chapter 7 Thus, each mass is restrained by a combination of springs in series, as indicated by the governing equations. 7.20 The mass m shown in Figure E.7.20 slides in a gravity field along a massless rod. Wrapped around the rod is a massless spring of constant & that has an unstretched length 189 (©2009 Cengage Learning, ALL RIGHTS RESERVED. Solutions to Exercises ~ Chapter 7 L. One end of the rod is attached to a vertically moving pivot that oscillates harmonically as 2(1) = 2, cost The position of the mass along the rod is u(f), which is measured from the unstretched position of the spring. Use Lagrange’s equations to obtain the nonlinear equations of motion. Solution 7.20 This exercise is an extension of Exercise 3.38, where we had a single degree-of-freedom system. Here, the radial coordinate 7 of the pendulum provides an additional degree of freedom. ‘The I position vector and the velocity of the mass with respect to the point O are, respectively, 2+(L+u)cos6 ]i+(L+u)sindj 2 +iiec0s0—(L+u)Osind |i+[usind +(L+u)600s0]j The total potential energy of the system is v ~mg[ 2+ (L-+u)eos0] +3 a? ‘The kinetic energy of the mass is m(V,,-V,,) = ([+ #0080 -(+1) sino} +[asind + (1+ u)dcoso)}) 2 which can be written as m(2+(L+u) +i? -2(L+ 1) 26sin8 + 2zicos0) ‘The Lagrange equations for this undamped system take the form d ( 2) or av F\ — |- di\26) 00° 30 4(Z 2, at\ oi) du” Gu ‘The first of the Lagrange equations can be evaluated to obtain the first governing equation as fu u)'6-2(L.+u)zsind) ~m(-(L+u)20c0s0— ziisind) +mg(L+u)sind =0 or 190 (©2009 Cengage Learning. ALL RIGHTS RESERVED. Solutions to Exercises — Chapter 7 m((L+u)'6+2(L+u)id-(L-+u)zsin0)+mg(L-+u)sin The second Lagrange equation can be evaluated to obtain the second governing equation as m4 (i+ 20050)-m((L4u) 6° = 20sin0) + ku—mgcos0 =0 or i+ 2—(L-+-u) 6) + ku—mgcos0 =0 On substituting 2=2,coset the governing equations of motion become m((L+u)' 6+2(L+u)id) +m(L+u)[ ¢+2,07 cosa sind =0 m(ii—(L-+u)6) + ku—m[g + 2,07 coset Joos = 0 From the form of the equations, it is clear that these equations could have been obtained by carrying out a moment balance about the pivot point and a force balance along the tangential direction of the pendulum motion or force balances along the tangential and radial directions. 724 Consider the auto-parametrie vibration absorber shown in Figure £7.21. ‘The system composed of mass m, and spring ky is externally excited by a harmonically oscillating force SO = f,coseat The oscillations of this primary system are to be attenuated by attaching another system composed of a mass ms that is attached to a rigid rod of mass mz and length /, The base of the rod is pivoted on mass m;. Attached to the rod at a distance J, rom the pivot is a spring ho. a) Determine expressions for the kinetic energy and the potential energy of the system. ) Show that the nonlinear equations of motions for this system are @X doy ao —n|( 22) coso+ 22 sino |= atk “ll | cos + sin | £0, FX sing 2 sind cos =0 de 0; {costar where 191 (©2009 Cengage Learning. ALL RIGHTS RESERVED. Solutions to Exercises ~ Chapter 7 k, kl? +m, +m,” gly / B+mE 1m, +2m/l wt nee) OF mba me We shall use Lagrange’s equations to determine the system equations. First, we start by determining the position vectors to the center of mass of the mass mz, the point mass ‘ms, and the point mass »m, from the static-equilibrium position of the system. This leads to 1 =Lsindi+(x+l,cos0) j Seino s(s +howo)j nag where x is the vertical displacement of the mass m from the static-equilibrium position and js the angular displacement from the vertical. Note that i and j are the unit vectors along the horizontal direction to the right and the upward vertical direction, respectively. ‘Then, the corresponding velocities are V, =1bcosdi+(i-1 sind) j w= Ldcosors(s -4 sind )j y The kinetic energy is 1 Jed + Sm Meh m WW) +m KK) where 1 ie aj 3 +m,| 2] =—m,l pntintion i] dnt Substituting and evaluating, we obtain Slot hm “im (40m) «(Sos +(-10sino)'| -L6sin o|e5m [86 +3 -21,20sin 0] 192 (©2009 Cengage Learning. ALL RIGHTS RESERVED. Solutions to Exercises ~ Chapter 7 If we ignore the effects of gravity, the potential energy is 13412 7 eno)? Vays? + yh (I sind) For this undamped system, the Lagrange equations are 4(3)-2 at\ 6) 00 Noting that Fe micemsams m2) sind mi sind ae 2 or ae Pens =J,0+m, 1) 6m A sind +m,20—my,esind 06 2 2 z mi 1) sbcos0—midcoso LD _ 1p sincos0 00 The first governing equation is ax 1) ao 1) doy (im +m, +m, Fe — my (2) SPsino—m, (2) cos mb Gesino mi) cosd +k,x = f(t) = f, coset or (menen) m( on, tae tm, +m) | ae -[m(é oe cos k= f,cosat or 193 (©2009 Cengage Learning. ALL RIGHTS RESERVED. Solutions to Exercises ~ Chapter 7 By employing the notations in the problem statement, we obtain 2 2 m, (E)+ma (2) 6080+ #8 sind |+hiyX = f,cosQr 2 dt de or The second governing equation is JoO+m, (8) 6 ~m( 5} sino, (5) sHe0sa+mi-mjssina Acos0=0 —m,x0.cos+m, (2) sBeoso+miidcos0-+ky Upon substituting in the expression for Jo and collecting terms, we obtain (Smt emsi)e-[m (4}+m, jrsina +k J? sinOcosé =0 By employing the notations in the problem statement, we obtain Le Pamp) eo gins tims (Sm +m, \e m, 4) em, J sino + ky? sindcos0 =0 a2 dr or 2 oe ind + 72 sinOcosd =0 ey 7.22 Consider the pulley-cable-mass system shown in Figure E7.22. Use Lagrange’s equations to determine the equation of motion of this system and place these equations in matrix form, Solution 7.22 In terms of the given coordinates x and @, the kinetic energy, the potential energy, the Rayleigh dissipation function, and the generalized forces are, respectively, (©2009 Cengage Learning. ALL RIGHTS. Solutions to Exercises — Chapter 7 Note that gravitational loading has not been taken into account because we are assuming, oscillations about the static-equilibrium position. The Lagrange equations for this unforced system are ‘Thus, after evaluation, we find that the first equation of motion is Jb +kyp?O+kr(r8-x)=M(D. JO+(k, +h) PO-kyrx=0 and the second equation of motion is mii k, (10-2) +08 mit+ 08+ kyx— kyr = Placing these equations in matrix form yields J 07fA) FO 0) /6) fk +k)r? kyr ](0] _ [MO 0 mix} "lo eflé -kr ella Jeal: © 7.23 For the system shown in Figure E7.23, determine the equations of motion, Assume that the length of the pendulum is L. 195 ©2009 Cengage Learning, ALL RIGHTS RESERVED. Solutions to Exercises ~ Chapter 7 Solution 7.23 Consider the figure shown to the right, where the different coordinates and the unit vectors to be used are provided. In terms of the given variables, the position vectors to mass m and the center of mass G of the disk are, respectively, r= x,i-Leosgj =(x, + Lsing)i-Leospj =x i= ri The corresponding velocities are V, =(%, + Locos) i+ Losingj Vi = b= bi ‘The system kinetic energy is Tad (YM) eam +5 So [@ +Lpcosg) +(Zsing)' | + 3m where, from Table 2.2, Jg= Ln? aD ‘Upon substituting and evaluating, we obtain T=; (m + aa) 24 pmbe +m,Li,pcosp ‘The system potential energy is V =-mgLoosp For this undamped system, the Lagrange equations take the form a(ar at\ a, (ar al ag which evaluate to, respectively, 196 ©2009 Cengage Learning, ALL RIGHTS RESERVED. Solutions te Exercises ~ Chapter 7 al(m ots smtocose] =0 Simos mx, cose |+m,Li,psing + m,gLsing =0 This leads to the governing equations 3m, ) A P 2 m, +~22 |¥ + m,L(@coso - ¢*sing)=0 ( 3 (gcosp—9' sing) mLG+mLi,cosy +mgLsing =0 7.24 Consider the system that is shown in Figure E7.24, where a uniform rod of length L2 and mass m is suspended from a massless rod of length Ly. The rod of length Ly is connected to the rod of length L2 by a frictionless hinge. Determine the equations of motion. Solution 7.24 The position of the center of mass of the rod is given by n=(ésing sing) P4( nomen and its corresponding velocity is Y =(rareos0 +4 cose, i+( ni sing, ~4¢,sino,)/ The kinetic energy is Tai m(o,-%,) +562 = jn) Lavon aonn) (hasina ping 1 pg? =smLge + gmaai meg + i mL,L,0,0, 0059-9) where, by using Table 2.2, we find that 2 2 J, = al (2) eR 2 Choosing point O as the datum, the potential energy is refer + ‘The Lagrange equations for this undamped system are 197 ©2009 Cengage Learning. ALL RIGHTS RESERVED. Solutions to Exercises ~ Chapter 7 (ar di\ 80, a(a ai\ a6, We note that the generalized forees Q, =@,, =0. The various terms in the Lagrange equations are d(or)_d(n. 1 S( 2) miss Smits cole -o)) =mlip, +hmbLty cos(9, 9) - Lbs Lops ~@,)sin(9,- 92) 7 Amt L-spssin(o-9,) Fe ometsing 2) dmg Lmiteosl- ) = mg + qlalad cos(9 -9:)— > + mlylod,(,~63)sin( 9, ~0:) ine ce -o) fa meh sin, Using these results in the Lagrange equations, we obtain 1... is . bh +5 Lab: 05(9, ~ 92) +> Lab sin(y, — 9.) + gsing, = L Fi 3 1,9, 008(2,— 92) — ; Lpy sin(p, 92) + iesing, = 7.25 Derive the governing equations of the frictionless system shown in Figure E7.25. Assume motions about the static-equilibrium position. Compare your results to those obtained for “small” oscillations in Example 7.3 after setting the damping coefficients 1 and ¢> to zero and discuss them. Solution 7.25 The kinetic energy is JP + hm @ qi8 +5 ie @) 198 (©2009 Cengage Learning. ALL RIGHTS RESERVED. Solutions to Exercises ~ Chapter 7 and the potential energy is 1 1 lh Vash (y-10) +5 ho (y+ ROY + hy’ © ‘The Lagrange equations for this undamped system are (2) ar HH) wy © (5 )-3 a&i\06) 00 Upon using Eqs. (a) and (b) in (c) and noting that Q, = Qp= 0, we obtain mi+(ky +k +h)y+(-kyr+kR)O=0 SO +( kyr? +hR*)0 +(-kyr +k) y= 0 Section 7.3.1 7.26 Determine the natural frequencies and mode shapes for the system shown in Figure £7.26, when 6; = 30° and 0 = 45° at the equilibrium position, Let L be the length of each spring at the equilibrium position and assume that the deflections in the springs at an angle are small Solution 7.26 The deflections 5, and é; are determined in the following manner. Referring to the Figure (a), we see that 5, fx + ¥* cos (4,4) where y= ry’ sind, x=yx'+y" cosd, Then, 6 =\e+y [cos@, cos, +sin#, sind, ] cos, + ysinO, a Referring to the Figure (b), we see that 6,=1,-L= ye +y* 0056, +0,) and, therefore, 199 (©2009 Cengage Learning. ALL RIGHTS RESERVED. Solutions to Exercises ~ Chapter 7 6, =x" + y* [c0s6, cosd, -sin 4, sind,] = xc0s8, ysind, ‘The potential energy is =hife (cos? 8, +cos? 4,)+y?(sin® 4, +5 +2xy(sinO, cos 4, ~sin8, cos) ] and the kinetic energy is The Lagrange equations are a (ar) ar ov dt ae ae After performing the indicated operations, we obtain, in matrix form [r oe (cos? 8, +cos*#,) K(sin8, cos, ~sin®, cos6,) } A + = 0 mil} | k(sind, cosd,—sind,cosA,) —k(1+sin?@+sin?@.) |LyJ (0 Assuming that x=Xeosot y=Yeosat we end up with the eigenvalue problem cos* 8, +cos*@,—Q? _sin@, cos0, —sin@, cos0, {xy _fo sin@, cos, —sin@,cos@, 1+sin’ 0, + sin’ 0, — OQ? ir} ) where Q? = «?/(k/m) is the eigenvalue and {X Y}" is the eigenvector. On substituting for the given equilibrium angles, we obtain 1.25-Q? 0.067 |{[X| _ fo 0.067 1.75-2? ||} lo, .326. The natural frequencies of the system whose eigenvalues are Q) = 1.114 and Q2 = are 200 (©2009 Cengage Learning, ALL RIGHTS RESERVED. Solutions to Exercises ~ Chapter 7 ‘The corresponding modal matrix is, 0.991 0.131 0.131 0.991 MATLAB program used to generate the numerical resulis for Exercise 726 th1=30*pi/180; th245*pi/180; Ke[eos(thl)'2-eos(th2)"2 sin(th2)*cos(th2)-sin(th1) cosh). sin(th2)*eos(th2)-sin(th1)*eos(th!) 1+sintht)'2+sin(h2)"2], M=diag({1 1); [modes Om2}-eig(M) Om=sqrt(Om2) 7.27 Determine the natural frequencies and mode shapes associated ith the system shown in Figure E7.27 for m = 10 kg, m= 20 kg, ki = 100 N/m, k2 = 100 Nim, and ks = 50 N/m, From Eqs. (7.41), we get 16 ee 100 Using Eqs. (7.46), we find that 4, =1403(L+m, +k) =1+0.107 x(14240,5)=2.75, 4, = 03 [1+ ky,(1+.03m,)]= 0.707? x[1-+ 0.5 x (1+ 0.707? x2)]=1 Thus, from Eqs. (7.47) ane la, - Ja? —4a, =p e1s—Va.15 4x =2.07 rad/s a= a+ laa, = 2702.75. 2.75° ax = 4.81 rad/s The mode shape ratios are obtained from Eqs. (7.49). Thus, 0.5 201 (©2009 Cengage Learning. ALL RIGHTS RESERVED. Solutions to Exercises ~ Chapter 7 i moo 2 20.707? «0, 1+@2m,-(0,/@,,) 1+20.707 —(2.07/3.16)" 2x0.707° 1+20.707" ~(4.81/3.16) ‘Thus, the first and second modes are, respectively, =f" a txh.-[ "44 7.28 Determine the natural frequencies and mode shapes associated with the system shown in Figure E7.28 for m, = 10° kg, m2 = 0.01 kg, and ky = ky =2 kN/m, Include plots of the mode shapes. Solution 7.28 From Eqs. (7.41), we get 3.16 1414.2 ee kh Using Eqs. (7.46), we find that a, =1403 (14m, +k) =14 0.316 x(1+10)=2.1 4, = 03 [1 +k, (1+ om,)]= 0.316 x1] = 0.1 ‘Thus, from Eqs. (7.47) a, + a, Jad 4a, aa ar 0. =3162 rad/s one la, + Ja? —4a, EP aa 0. = 2024.8 rad/s ‘The mode shape ratios are obtained from Eqs. (7.49). Thus, key 0 Ne m,o2 10x 0.316 =0513 Yy 1+e2m,—-(@,/@,)° 1+10%0.316? —(316.2/1414.2)" . 2 2 Ky mo? 100.316 Soy Sj oy L+@m,—-(@,/0,) 1410x0316? —(2024.8/1414.2) Thus, the first and second modes are, respectively, 202 ©2009 Cengage Learning. ALL RIGHTS RESERVED. Solutions to Exercises ~ Chapter 7 w-{| and {Y= mer ‘The modes shapes are sketched below. First mode ‘Second mode Not to scale Figure £7.28 7.29 For the system considered in Exercise 7.10, remove the nonlinear spring k; by considering small displacements about x», set the damping coefficients c1 = cy = 0, and consider the free oscillations of this system. Choose the values of the parameters as follows: m = 10 kg, my = 2 kg, ki = ky = k= 10 Nim, and a= 2 m*, Assume that the nonlinear spring is initially compressed by x, = 0.05 m. Determine the natural frequencies and mode shapes associated with free oscillations about the equilibrium position. Solution 7.29 The governing equations of motion are a special case of those determined in the solution to Exercise 7.10. Considering ‘small’ oscillations about the initially compressed position of the nonlinear spring we replace the nonlinear spring by an equivalent linear spring with stiffness determined by Eq. (2.25); that is, oF K de where x, =0.05 m. For the parameters given, Kk, =10x(1+3x2x0.05*) =10.15 Nim K(1+3ax2) K(1+3az") ‘The governing equations of motion are given by im 07 [%)] [hth +k -k][m]_ [0 + = 0 mile -k IIs J 10. ‘Making use of Eq. (7.37), we obtain the eigenvalue problem 203 (©2009 Cengage Learning. ALL RIGHTS RESERVED. Solutions to Exercises ~ Chapter 7 k, +k, +k,-o'm, -k, {Xi} _fo -k, bom, xJ7 0 which leads to the characteristic equation mym,o* —[(ky + ky +k,)my + kum, Jo? + (ky +k, +k, kk? =0 For the given parameters, we obtain (10x2)e* —[(10+10+10.15)x2+10.15x10]o? + (10 +10+10.15) x10.15—10.15? =0 or 20a - 161.80" +203=0 The roots of this polynomial provide the first and second natural frequencies as @, =1.246 rad/s 2.557 rad/s o, Then, the mode shape ratio for the first mode is given by Xy_ ke -myo? _10.15-2«1.246 =0.694 Xn ok 10.15 and the mode shape ratio for the second mode is given by X2.557' __o.288 10.15 ‘Thus, the system modes shapes are x, pol a on,= {28} 7.30 Consider a system with the following inertia and stiffness matrices: wa-[; ‘| ke: (x)-[00" 4] wn 06 7 If the modes of the system are given by 0.5414 5.5575 teh =fPS1 ant (ay, =f P54) and the natural frequency associated with {X}; is 19.55 rad/s, then determine the unknown coefficients in the stiffness matrix and the other natural frequency of the system, Solution 7.30 Starting with Eq. (7.38b), we have 204 (©2009 Cengage Learning, ALL RIGHTS RESERVED. Solutions to Exercises ~ Chapter 7 10° -2@* -k X,|_ fo -k ky, 607 |X, 10 We see that at @ = 19.55 rad/s; therefore, the first mode shape ratio is Xu £ =," __- 05414 X, 10*-2x19.55? or 4k =0.5414x(10' -2x19.55*)=5000 Nim Also, since Xu _ ky — 60? oF k we have that Xn 4 60? ay Xn which evaluates to a, = 5000% (0.5414) +6%19.55? =5000 Nim At @= a, we have ky = 605 ‘Therefore, @, = 73.92 rad/s 731 Determine if the system shown in Figure E7.31 has any rigid-body modes. Solution 7.31 ‘Since the system shown in Figure E7.31 is a special case of that shown in Figure 7.7a, we use Eqs. (7.5b) to determine the mass and stiffness matrices as. 205 (©2009 Cengage Learning, ALLRIGHTS RESERVED. Solutions to Exercises ~ Chapter 7 m 0 07) []=| 0 m, 0 0 0 a kk 0 [K]=|-k +k -k 0 -h ok Then, from Eq. (7.36), the characteristic equation is given by k,-a'm, -k, 0 det] -k ky +k-o’m, —-k, |=0 0 k,-o'm, Expanding the determinant, we obtain 0 {mmmor' —[ km, + rm, + (k + k, mm, Jo? + Fike (mm, +m, + m,)}=0 Since one of the solutions is « = 0, we have one rigid body mode. Alternatively, one could use the rank function in MATLAB on the [K] matrix to determine that the rank of [K] is 2. Therefore, since the stiffness matrix is not of full rank, there is 3 — 2 = 1 rigid body mode. 7.32 Let the system shown in Figure E7.31 represent a system of three railroad cars with ‘masses my = mz = ms = 1200 kg and interconnections k, = k = 4800 N/m. Determine the natural frequencies and mode shapes of this system and plot the corresponding mode shapes. Solution 7.32 ‘The natural frequencies of the system can be obtained from the characteristic equation given in the solution to Exercise 7.31. However, the determination of the mode shapes requires considerable algebra. Therefore, we will obtain the solution using the eig function in MATLAB. Using the [4M] and [K] matrices shown in Exercise 7.31 and the given values of m, and kj, we obtain the natural frequencies @ = 0.0 rad/s, @ = 63.246 rad/s, and «@; = 109.545 rad/s. The corresponding normalized modal vectors are: 1 [e]=]1 0 -2 11 ‘The mode shapes for the three modes are shown below. 206 (©2009 Cengage Learning. ALL RIGHTS RESE! Solutions to Exercises ~ Chapter 7 A A nM 1 [ms 1 [ms 1 [om = 0.0 rads; first mode Ho mt | [as 2» 63.2 rads; second mode ! MH a ie io ee MATLAB program used to generate the numerical values for Exercise 7.32 ‘m1=1200; m2=1200; m3=1200; KI=4.8e6; 134.806; fiag({ml m2 m3); (k1 -Kl 0; -k1 kI+K3 -K3; 0-K3 k3]; [Mode R}-eig(K.M); for k=1:3, ‘Mode(:,k)}-Mode(:,ky/max(Mode(3,k)); 7.33 A flexible structural system is represented by the model shown in Figure E7.33. Determine the governing equations of motion of this system, and from these three ‘equations, determine the eigenvalues and eigenvectors associated with free oscillations of this system. Find the locations of the nodes for the different mode shapes. The stiffness of the structural members is k= 3EU/L*. Solution 7.33 Starting with the formulation given in the solution to Exercise 7.31, we have that the mass and stiffness matrices are oo uw 2m 0 0m -k 0 [K]=|-k 2k -& 0 -k k and the governing equations of motion are of the form 207 (©2009 Cengage Learning, ALL RIGHTS RESERVED. Solutions to Exercises — Chapter 7 % fs 0 [M]y % 7 +[K]y 2 6 = 40 &, x} (0 From Eq. (7.36), the eigenvalue problem associated with free oscillations is determined tobe k-o'm -k o |(xX%) [0 -k — 2k-2@’m ki} X, b= 40 0 -k —-k-a'm|(x,} lo Then, from Eq. (7.37), the characteristic equation is k-a'm -k 0 det] -k 2k-2ma* -k |=0 0 ak k-mo’* Expanding the determinant, we obtain 0 (2m?! —[ 2k? + 2h? + 2k]? +E (m+2m-+m)}=0 @ {2m'o' — 6km'o? + 4mk*} =0 or @* {o' -30307 +20!}=0 @ (@? -203)(o-@3)=0 where oat m The roots of this equation are @? =0 =o; @; =2@) The modes shape ratios can be obtained from k-oim =k 0 1x, | | 0 -k — 2k-2ojm kX, = 40, 0 -k k-ajm|[X,,} (0. or 208 ©2009 Cengage Learning, ALL RIGHTS RESERVED. Solutions to Exercises ~ Chapter 7 @,-0 =o; 0 X,) [0 -0} 203-20} -w} |) X,,p=40 0 @; a -07 |X, 0 Solving for the ratios in the jth mode, we obtain AD Ay Mo =% 1 X, O20} 2g? Xy _ a2 -0) xX and the modal matrix is tlot [o]=]1 0 - 1. From the modal matrix, we see that there is no node point for oscillations in the first mode. For oscillations on the second mode, the mass nm, does not move; therefore, it is a node point. In the third mode, there is a node point between masses m, and mp and another between masses mz and ms. 734 Consider the system shown in Figure E7.34 in which the three masses mm, ma, and 2m; are located on a uniform cantilever beam with flexural rigidity E7. The inverse of the stiffness matrix for this system [K], which is called the flexibility matric, is given by p[7 4 4 [kJ'==]14 8 25 3EI 4 23 1 Determine the following: (a) the stiffness matrix of the system, (b) the governing equations of motion, and (c) when m = ni = my = m, determine the natural frequencies and mode shapes of the system. For part (c), let @ = 3EU/mL? and express the natural frequencies in terms of a. Solution 7.34 (@ The stiffness matrix is determined from the inverse of the flexibility matrix. Thus, 05385 -1.2308 0.9231 [x]-2=4|-1.2308 3.3846 3.5385 0.9231 -3.5385 6.1538 (b) The governing equations of motion are obtained after determining the mass and stiffiness matrices from Eqs. (7.5b) and the above equation; that is, 209 (©2009 Cengage Learning, ALL RIGHTS RESERVED. Solutions to Exercises ~ Chapter 7 m 0 07% 0.5385 1.2308 0.9231 ]{x,] [0 om, o|{x,b+34!) 1.2308 3.3846 -3.5385]!,!=Jo 0 0 mje 0.9231 -3.5385 6.1538 |[x,} 0 (©) For m, = my = m3 = m and a = 3EI/(mL*), the above equations can be rewritten as 1 0 O7fs, 0.5385 1.2308 0.9231 ]{x] (0 0 1 0/}%}+e-1.2308 3.3846 -3.5385|! x, b=10 oo ily, 0.9231 3.5385 6.1938 |[x,J [0 Then, the natural frequencies are determined from the roots of the determinant 0.5385-Q? —-1.2308 0.9231 det} -1.2308 — 3.3846-Q?— -3.5385 |=0 0.9231 3.5385 6.1538--Q? where Q = ava, Rather than solve this directly, we use the eg function of MATLAB, which gives the natural frequencies 0, =0.169a 1.106 2.971a and the corresponding mode shapes determined from the eigenvectors are 6.393 -0.788 0.215 [®]=]3.399 1.188 0.699 1.000 1.000 1.000 [ MATLAB program used to generate the numerical resuls for Exersise 74 wm(27 14 4; 1482.5; 42.5 I, Modiag(l1 1 1D); [Modes E}-cig(K,M); 7.35 If the stiffness matrix for the system shown in Figure E7.14 is given by the following matrix 210 (©2009 Cengage Learning. ALL RIGHTS RESERVED. Solutions to Exercises ~ Chapter 7 24 -12 0 0 EI|-12 24 -12 0 KI B\ 9 12 24-12 0 Oo -12 12 If all the masses of the floors of the four-story building are equal; that is, m = my = my = ‘m, = m, then determine the system eigenvalues and eigenvectors and plot the eigenvectors of this system. Let @= El/mL? and express the natural frequencies in terms ofa. Solution 7.35 From Exercise E7.14 and the stiffness matrix given above, we have that the governing equations are m 0 0 OVfx, 24 -12 0 O)fx) [0 0 mo olfe| m}-12 24 ~12 0 |Ix,[_ Jo 00 mo BP) 0 -12 24 -12])x,{° ]o 00 0 mils O10. 12) 12 |(2)) 10 which can be rewritten as 10 0 O}fx, 24 -12 0 0 {x 0 010 offe,| |-12 24 -12 0 |x, Jo +a) = 00 1 offs, 0 -12 2% -12])x/~ Jo 000 tly 0 o -2 12)}lxJ lo where EL mE Then, the natural frequencies are determined from the roots of the determinant 24-Q? = -12 0 0 -12) 24-Q? -12 0 det es 0 “12 24-Q? -12 0 0 -12. 12-9 where = w/a. Rather than solve this directly, we use the eig function of MATLAB to obtain the natural frequencies 0, =1.203@ 0, =3.464ar 0, =5.307a 6.510@ 2u1 ©2009 Cengage Learning. ALL RIGHTS RESERVED. ~ Solutions to Exercises — Chapter 7 and the normalized mode shape ratios 0.347 1.000 1.532 -1.879 lo] 0.653 -1.000 -0.532 2.879 0.879 0.000 1.347 2.532 1.000 1.000 1.000 1.000 These mode shapes are plotted below. Ft Second TN Fath mice sole oma aha ele oaste 1 Ff iA v Vv “i \or \ Figure £7.35 ‘MATLAB program used to generate the numerical resulls and figure for Exercise 7.35 Mediag({1 111); Ke[24-12 00; -12 24-12 0; 0 -12 24-12; 00-12 1 modes KM); wesqrtE) smod=[First'; ‘Second! Third Fourth’) for kel:4 modes(:,k)=modes(,kYmodes(4,k); end modes hold on for n= 1:4 ploi((2¢n-1 2*n-1] {1 4).k~) plot(2*n-1+modes(:,n)imax(abs(modes(:,n)),1:4/sk-) text2*n-1, 4.5,flomega_{° num2str(n)")=" num2str(w(n,0),3) "alpha fontsize’,14,"HorizontalAlignment center) text(2*n-l, 5.1;mod(n, | HorizontalAlignment’‘center’) text(2*n-1, 4.8;'mode' fontsize’ 14, Horizontal Alignment, center’) end axis((0 80 5)) axis off 7.36 Repeat Example 7.16 for the following values of the nondimensional ratios: ka = 0.5, L21 =2.0, and @, = 1.5. 212 ©2009 Cengage Learning. ALL RIGHTS RESERVED. Solutions to Exercises — Chapter 7 Solution 7.36 For the given parameter values fai and a1, we find from Eqs. (b) of Example 7.16 that the system is uncoupled. Furthermore, from Example 7.14, we find, upon substituting these given values into Eqs. (d) and (¢), that © = 1.2247 and 2 = 1.837. As expected, the mode shape ratios are both zero, which means that the system is uncoupled; that is, the translation of the center of mass occurs independently of the rotation about this point. 7.37 Repeat Example 7.15 when @,=2.0 and J, = 1.0. Solution 7.37 ‘The natural frequencies are obtained from Eq. (m) of Example 7.15. Thus, 0, = fie? 0-29)? +4 V2 =0.835 Q,=y1+2?+ [a2 4x1 V3 =2.07 ‘The mode shape ratios are obtained form Eq, (0), thus, es 10, 1-1-0835" x, 1 1 od LO,, 1-Q} 1-207? = 0.303 ‘These mode shapes are described qualitatively in Figure 7.16. 7.38 An elastically supported machine tool with a total mass of 4000 kg has a resonant frequency of 80 Hz. An 800 kg absorber system with a natural frequency of 80 Hz is attached to the machine tool. Determine the natural frequencies and mode shapes of this system, Solution 7.38 ‘The eigenvalue equation for this system is given by Eq. (7.38b); that is, [ree se From the given parameter values, we have that k, = myo, = 4000 x (2 x80)" =1.0106%10? Nim k, =m,@2, = 800x (24x80) =2.0213 x10" N/m ‘Then, the eigenvalue problem takes the form 1.212710 —400007 —-2.021x10 fx] _ fo 2.02110 2.021 10° 8000" || x, J ~ lo 213 ©2009 Cengage Learning, ALL RIGHTS RESERVED. Solutions to Exercises ~ Chapter 7 whose eigenvalues provide the natural frequen: 0, = 402.66 rad/s ©; = 627.46 rad/s es The corresponding modal matrix is 7 0.0099 -0.0123 “| -0.0276 0.0221 MATLAB program used to generate the numerical results and figure for Exercise 7.38 M=[4000,0;0,800);, =[1.2127e9, -2.021e8;-2.02108,2.021¢8]; [mode,Om2}=eig(K.M); Om=sqrt(Om2) mode 7.39 A six-cylinder, four-cycle engine driving a generator is modeled! by using an eight- degree-of-freedom system shown in Figure E7.39. Free oscillations of this system are described by the following system LG} +119} ={0} where 210000000 0200000 0 00200000 000720000 y]= 7 Mfg 9 0 0 2 0 0 of &™ 0 0 0 0 0 210 0 000000980 0000000 49 and "cena at 214 ©2009 Cengage Learning, ALL RIGHTS RESERVED. Solutions to Exercises — Chapter 7 51 S51 0 0 0 0 -51 102 -S1 0 0 0 0 -Si 1022 -51 0 0 0 -51 102 -S1 0 0 -S1 102 -51 0 0 0 -51 117 -66 0 0 0 0 -66 81 -15 00 0-0 15 15 10° Nm/rad ecco ccooce [x]= 0 0 0 0 coocle Determine the natural frequencies and mode shapes associated with the system and plot the mode shapes. Does the system have any rigid-body modes? Solution 7.39 ) Using the MATLAB funetion eig, we find the natural frequencies in rad/s and mode shapes, which are given below. ) The rank of the stiffness matrix is 7; therefore, there is one rigid-body mode, which in the figure is the mode shape corresponding to « = 0. Figure £7.39 MATLAB pro od se 7.38 Mediag({21 21 21 21 21 21 98 49); K={51-51000000 -51102-51 00000 215 ©2009 Cengage Learning. ALL RIGHTS RESERVED. Solutions to Exercises — Chapter 7 0-51 102-510000 00-1 102-51000 000-51 102-5100 0000-51 117-660 00000-6681 -15 000000-15 15}*10°6; smk=rank(K) {modes E]=cig(.M); w=sqrt) hold on w(I1)05 forn-1:8 plot(1 8},2%n-1 2%-1}-k~) plot(:8,2*n-L+modes(,n)max(abs(modes(.n)))'sk text(-4,2¢n-1, [omega {° num2str(n) snum2str(w(0,n),4)],fontsize' 4 HorizontalAlignment lef) end yeaxis; (1)-0.5; axis(v) axis off 7.40 Consider the system shown in Figure £7.40. (a) Determine the equations of motion for the system and put them in matrix form. (b) From the results obtained in part (a), obtain the determinant form of the characteristic equation. Solution 7.40 a) The kinetic energy and the potential energy are, respectively, 1 ere JG + gh 2 1 1 1 2 Vash (nd) +54 (n8 +n) +h (18) Since the system is undamped and free of external forces, the Lagrange equations are tafe dt\ a6, 4(ar) at\ 06, J Upon using the expressions for T'and V and evaluating, we obtain SG + (hy +h )7G, + 78, =0 6, + (ke +k) 20, + kerir8, In matrix form, these equations take the form [ene telat} b) If we let § = @coser, then the determinant form of the characteristic equation is 216 (©2009 Cengage Learning. ALL RIGHTS RESERVED. Solutions to Exercises — Chapter 7 de (4 th) a eo" th kann, (kth) -J,07 7.41 Two identical discs of rotary inertia Jo and radius r are mounted on identical shafts and undergo torsional oscillations. Each shaft has a torsional spring constant k). At a radial distance @ from their respective centers, the rotors are connected via a translation, spring of constant k, as shown in Figure E7.41. If Jo = 10 kg-m’, r = 0.25 m, k, = 600 Nm/rad, k= 1000 N/m, and a = 0.15 m, then determine the natural frequencies of the system. ‘Solution 7.41 Let 6; and @; be the angular rotations of the left and right disks, respectively. ‘The total kinetic energy and the potential energy are 1 1 SIG + SIO: ih + Ih Lata tpgryt 2 V =LkO? +2 k63 +k (a0, -a0,) Opa) (2 1a (ge Lit 2 ~ha (aise?) 44a'(@.-8,) ‘The Lagrange equations for this undamped and force-free system are (ar )_ at, av _ at\06,) 06, * 60, Cc ne ai\ 06,) 00, * a0, 0 which on evaluation leads to 41,3 ~(J,0,)+k,0, + ka’ (0,-0,)=0 5 (a6) +h, + ha? (0, —0,) J,6,+(k, +ka?)0, —ka’O, =0 and a Gls) +k, —ka’ (6,-0,)=0 SB, +(k, + ka’ )0, ka’, =0 At this point, it is convenient to introduce the non-dimensional quantities hat Then, the governing equations become 217 ©2009 Cengage Learning, ALL RIGHTS RESERVED. Solutions to Exercises ~ Chapter 7 £6. 1+), KO,=0 dr’ £8 6 14K)0,-KO,=0 de To determine the natural frequencies, we assume a solution of the form 0,=Oe" j=1,2 Upon substitution of this equation in to the governing equations, we obtain 070, +(1+ K)O,- KO, =0 -'O, +(1+ K)®, — KO, =0 Setting the determinant of the coefficients to zero, we obtain [-2+a+K)] -« or O82. KO? 11 2K =0 the roots of which are For the values given, we find that ka? _ 1000.1. xa 0375 600 o,= 1600 _ 7.746 rad/s 10 Then, 0, =0,=7.746 rad/s »,N1+2K =7.746Vi +2x0.0375 = 8.031 rad/s 7.42 A cable fixed at one end and carrying a mass m at the other end stretched over two pulleys, as shown in Figure E7.42. The pulleys have rotary inertia J; and J about the respective rotation centers, and the corresponding radii are r; and rp, respectively. The stiffness of their various sections of the cables is provided in the figure. Assume that there is sufficient friction so that the cable does not slip on the pulleys. a) Determine the equations governing this vibratory system and place them in matrix form. 218 (©2009 Cengage Learning. ALL RIGHTS RESERVED. Solutions to Exercises ~ Chapter 7 b) If m = 12 kg, J; = 0.2 kg m’, Jy = 0.3 kg m?, 7 = 120 mm, r; =160 mm, ki = 25x 10° N/m, and ky = 40x10° N/m, determine the natural frequencies and mode shapes. Solution 7.42 If the angular rotation associated with inertia J; is 6, the angular rotation associated with inertia Jz is 6, and the translation of the mass m from the equilibrium position is x, then the kinetic energy and the potential energy are, respectively, rosie +p smi? 1 21 ae 2 Vk (iA) + 5h (60. 0) +>hs (2-10) Note that the gravitational loading has already been taken into account, since we are assuming motions about the static-equilibrium positions, ‘The Lagrange equations for this undamped system take the form d(ar)_ar av sailor eae at\ 06,) 00, * 60, af a )\_ oT , ov dt\a6,) 00, ° 20, Eien at de) x” ox Then, on carrying out the evaluations, the first equation of motion is SG, + yO, — hah, — 59.) =0 IG. + (kh) 90, kar, =0 The second equations of motion is a8 + kets (20, —00,) — kas (x02) =0 SB; + (ky +k) 120, — kyr, — kyryx =0 ‘The third equation of motion is mi+k,(x-18,)=0 mit+ kx — kr, =0 Placing these equations in matrix form yields J, 0 07f8) [Ck +k)? karin 0 |f4) fo OF, OFA F+) hi (eth) ky 1} p=10 0 0 mile 0 -kr —& fix] lo Using the numerical values given, we have that 219 ©2009 Cengage Learning. ALL RIGHTS RESERVED. Solutions to Exercises ~ Chapter 7 02 0 0f6 0 03 0/4 oo 12i[z (25000 +40000) 0.12? -40000x0.12x0.16 0 a) {0 +| 40000x0.120.16 (25000 +4000) x0.16? -25000x0.16 |}, }=40 0 -25000 0.16 25000 |x} lo or 02 0 07(6) [936 -768 0 J(a) {0 0 03 0|{6,++/-768 1664 -4000|/a,}=40 0 0 12]{¥} | 0 4000 2s000]{x} |o The eigenvalues and modes shapes are obtained from the MATLAB function e.g, which gives @, = 21.586 rad/s @, = 56.260 rad/s @, =93.160 rad/s and 0.923 1.536 -1.338 -1.012 0.606 1,393 0.209 -1.187 -0.070 MATLAB program used to generate the numerical results for Exercise 742 31°02; 1203; m=12; k1=25000; k2=40000; k3=25000; 110.12; 20.16; iag({J1 2 m)); (k1+12)*11"2 42*11 2. 0 “24112 (2443)"22 342 0-432 k3]; 7.43 An electrical motor and pump system operates at 1800 rpm, and they are elastically mounted to a support structure, ‘The mass of the system is m1 = 25 kg and the effective viscous-damping factor of the mount is 0.15. Unfortunately, it is found that 1800 rpm coincides with the natural frequency 1 of the system and the horizontal amplitude of the system is excessive. To decrease the magnitude of the horizontal amplitude, it is decided that rather than change the stiffness of the support a second mass m) = 0.25 kg. will be added to the system by attaching m) to the end of a cantilever beam, as shown in 220 (©2009 Cengage Learning. ALL RIGHTS RESERVED. Solutions to Exercises ~ Chapter 7 Figure E7.43, The cantilever beam is a solid circular rod 6 mm in diameter. Use E = 1.96 x 10!' N/m? for the Young’s modulus of elasticity of the beam material. What should be the length of the rod so that the natural frequencies of the modified system are not in the range wy + 4%? Solution 7.43 We shall first determine the governing equations i of motion of this system, and then from the eigenvalue problem associated with the free oscillations determine what the appropriate length of the rod should be. The free-body diagrams of the Kora), masses m and m, are shown to the right along with the respective inertia forces. Carrying out a force balance along the horizontal direction leads to the ae equations of motion mi, +(k, +k) x — kay +e% =0 mi, + kx, kx, =0 ‘The associated eigenvalue problem for the undamped system can be obtained from Eqs. (7.37) and (7.38) as Mor EY It follows from Eq. (7.37) that the characteristic equation is mmo" —[(k, +k, )m, + km, Jo? +k, =0 For the given parameter values, we obtain 1800x 2 yy 60 k, = 02m, = (602)' x25 =888.26x10° N/m 602 rad/s Nm 4, -3EL 3x1,96%10" x7/2x(3x10")' 74.81 lar ——5 > where L is the length of the rod. On substituting the parameter values into the characteristic equation, we obtain 25x0.25xo" -{[sss25ci0 + 781) 0.5 82 ot 888.26%10°x74.81 ——oeee or 221 (©2009 Cengage Learning. ALL RIGHTS RESERVED. Solutions to Exercises — Chapter 7 6.250 - [222,065 7 lat + S451E as e It is required that the natural frequencies @ and @; determined from this characteristic equation be outside the range (0.96x60, 1.04x607) rad/s or (180.07, 197.92) rad/s. Using MATLAB, we find that for L =0.203 m, a = 179.5 rad/s and @ = 198.5 rad/s. MATLAB program used to generate the numerical results for Exereise 143 L=feero(@val,0.5) res=route(L) function r=val(L) es-route(L); ‘abs(rtes(I)-180.07)-abs(rtes(2)-197.92); function rtes=route(L) 3.25, -222065-1888.95/L."3, 66.45e6/L/3]; F(sqrt(roots(e))); re 7.44 Consider the coupled pendulum system shown in Figure E7.44, a) Use Lagrange’s equations to derive the equations of motion, Include the effects of gravity. Put the results in matrix form. b) For m = 10 kg, mz = 15 kg, ms = 5 kg, ki = kx = 100 N/m, Li determine the natural frequencies and mode shapes of the system. Solution 7.44 (@) We let the rod with mass m; have a rotation 4} from the vertical, the rod with mass ‘m; have a rotation 6, from the vertical, and the mass m have a horizontal displacement x. Then, the kinetic energy is, 5m, and Lp = 0.4 m, fae + fae + jm where ae =mG and J, ym For “small” rotations 6, and 6), we use Eq, (4) of Example 7.15 to obtain the following expression for the potential energy v 1 ee 21 2 gmsho? + cmghO,’ +k (x-LA) +k (x-L0,) For this undamped and unforced system, the Lagrange equations are 222 (©2009 Cengage Learning, ALL RIGHTS RESERVED. Solutions to Exercises — Chapter 7 Thus, the first equation of motion is Ji + 8 0, - yb (x-10,)=0 J +( B+ kit ahs 0 ‘The second equation of motion is Jb, + BE 0, — byl (x 18,)=0 Jb, (08 + hoa ~kLx=0 ‘The third equation of motion is mi+k, (x-1,0,) +k, (x-1,8,)=0 m+ (k, +h) x- kL, -k,L,0, =0 Placing these equations in matrix form results in mish «4 0 kL, cn) 2 4) [0 04 0 + 0 Pala ie kl, |10,b=40 h aly |) p= oo mY] ma, wb h+k|UF (b) Upon substituting the parameter values into the governing equations of motion determined in Part (a), we obtain 0.833 0 O}{6 0 08 0144, 0 0 sls 10%9,8x0.5/2+100%0.5" 0 -1000.5](8,) {0 + 0 15x9.8x0.4/2+100x0.4? -100x0.4|}0,+=10 -100%0.5 1000.4 100+100 |x} [0 223 (©2009 Cengage Learning. ALL RIGHTS RESERVED. Solutions to Exercises — Chapter 7 Assuming that x= Xeoswt 8,=O,cosat j we obtain the eigenvalue problem 49.50.8830" 0 -50 8, 0} 0 454-0807 40 |}, }=40 -30 —40 200-Sa7|[ XJ [0. ‘The eigenvalues and modes shapes are obtained from the MATLAB function e.g, which gives .029 rad/s ©, =7.601 rad/s ©, =9.064 rad/s and 0.497 -0.673 -0.708 ©=/0.441 0.881 0.528 0.357 -0.018 0.268 MATLAB program used to generate the numerical results for Exercise 744 10,8333; 20.8; m=5; L1=0.5; 12-0.4; Mediag({J1 J2 m)); 0 m2*g*L2/2+2*L2°2 -K2*L2 “KILI 42412 k1+42] [modes E}-eig(K,M) E 745 One model that has been used to study the vibratory motion of motor vehicles is shown in Figure £7.45. ‘The body of the vehicle has a mass m, and a rotary inertia Je about an axis through the center. The elasticity of the tires is represented by springs ko, and the elasticity of the suspension by springs k. The mass of the tire assemblies is m2. a) Determine the matrix form for the governing equations of the system. b) Obtain the natural frequencies and mode shapes for the case where m, = 800 kg, m= 25 ke, ki = 60 kN/m, fo = 20 KN/m, L = 1.4 m, and Jg= 180 kg-m?, 224 (©2009 Cengage Learning. ALL RIGHTS RESERVED. Solutions to Exercises — Chapter 7 Solution 7.45 (@) Assuming “small” angular oscillations, the kinetic energy and the potential energy are, respectively, T= Fuel + hms pm hms apht + phos th (x,+10-x,) +k Gs -10-x,) For the undamped system, the Lagrange equations are aT) oT wv + ‘The first equation of motion is Job +k, (x, +10—x,)-k,(x,-LO-x,)=0 Job + 2k 2O—kLx, +k Lx, =0 The second equation of motion is mii, +k, (x, + L8—x,) +k (x, -L0-x)=0 mi + 2k,x, ~ yx — yx = 0 The third equation of motion is m,X, +k,x, —k,(x,+L0-x,)=0 Imi, + (k, +k) mx, LO =0 The fourth equation of motion is mi, + kx, — k(x, LO-x, mi, + (ky +h). x,— Kx, + k,L0 = 0 Placing these equations in matrix form, we obtain 225 (©2009 Cengage Learning. ALL RIGHTS RESERVED. Solutions to Exercises ~ Chapter 7 ; 9 0 O71f6) [22 0 -kL kL |{O 1 0 O18) 0 2 =k [I 0 0 m Offs, ["}-KL -& k+h 0 |] 0 0 0 mile) LAL -h 0 k+k {by SSeS (b) Assuming solutions of the form 4= @cosar and x; = Xj coset, j= 1, 2, 3 and using the values given in the problem statement, we obtain 180 0 0 0) [2x60x14? 0 60x14 60x14 e) [0 _o'| 9 80 0 0} | oo 2x60 -60 60 | gs |] Xf _Jo 0 0 25 0}'| 60x14 -60 60+20 0 x,{~ Jo 0 0 0 25} | 60x14 60 0 60+20 x) lo which evaluates to i) 0 O Wir OG 29 ©} [0 ,| 9 800 0 0 0 120 60 -60} Jfx,| Jo “a + x10 7 O 0 oo 29 mH 6 x,[ Jo 0 0 0 25} | 84 -60 0 80 x} lo Upon using the e:ig function in MATLAB, we obiain the natural frequencies 0,=6.018 rad/s a, =15.662 rad/s @, =57.566 rad/s 0, =65.279 rad/s and corresponding modal matrix 0.0 0.630 0.0 0.0383 0.0347 0.0 0.0066 0.0 0.0264 0.0727 0.1389 -0.1213 0.0264 -0.0727 0.1389 0.1213, MATLAB program used to generate the numerical for Exercise 7.45 [180 800 25 25) 50000; k2=20000;, 02°k1 -kI -kl KIL -k1 k142.0 KI*L -k1 0K1+K2] [modes E]-eig(K,.M) Lwesqr(e) 226 (©2009 Cengage Learning. ALL RIGHTS RESERVED. Solutions to Exercises ~ Chapter 7 7.46 A tractor-trailer is hauling a large cylindrical drum that is elastically supported by spring k, as shown in Figure E7.46. The drum rolls on the floor of the trailer without slipping. ‘The trailer is attached to the tractor by a system that has an equivalent spring ky ‘The mass of the tractor is m, that of the trailer is m, and that of the drum is ms. 4) Obtain the equations of motion for this system. b) If = k =k, m = my =m, and m= 2m/3, then obtain the natural frequencies in terms of kim and the corresponding mode shapes. Solution 7.46 (@ In terms of the given coordinates and assuming motions about the static equilibrium position, the kinetic energy and the potential energy are, respectively, dns? + hn, le + hm? 2 2 2 2 Fala a) 4th —m) where x3 = x2 -r, and from Table 2.2 pales) pe (% 8)! ‘The Lagrange equations for this undamped and unforced system are a(ar)_ar av ax, Ox, mi, +k kx, =0 The second equation of motion is 227 ©2009 Cengage Learning. ALL RIGHTS RESERVED. Solutions to Exercises ~ Chapter 7 +h (5 —m)=0 kx kx =0 Placing these equations in matrix form, we obtain [m, 0 oO lx, kk (OO Tfx, 0 mtm,/2 -m,/2]}%,)4/-k kth kh |}x, 0 a |) p= 40 0 -m,/2 3m/2}(%) Lo -& & Jly} lo (b) Assuming solutions of the form x; = Xj cose, j = 1, 2, 3 and using the values given in the problem statement, we obtain m 0 0 k -k O7)(X,) [0 -0°| 0 Am/3 -m/3|+|-k 2k -k]|}x,b=I0 0 -m/3 m o -& ki\LxJ lo which can be rewritten as 10 0 1 = o7][x,) fo -2/0 4/3 -1/3]+/-1 2 -1/]).x,+=10 0-1/3 1 oa 1 where , = 0.000 rad/s », radi rad/s and corresponding mode shapes 0.612 -0.555 -0.563 =/0.612 0.000 0.666 0.612 0.832 -0.154 228 (©2009 Cengage Learning. ALL RIGHTS RESERVED. Solutions to Exercises - Chapter 7 Since one of the natural frequencies is zero, this system possesses a rigid-body mode. [modes EJ-eig(K,M) wesqrt(E) - 7.47 Determine the characteristic equation for the system shown in Figure E7.47 and solve this equation for the special case when ky = ka = ks = k and m, = mz = m3 = m. Determine if the system has any rigid-body modes. Solution 7.47 We will first use Lagrange’s equation to obtain the governing equation of motion and then use this equation to examine the free oscillations. ‘The kinetic energy is =pmal @ and the potential energy is -x) © ‘The Lagrange equations are ©) Upon using Eqs. (a) and (b) in (c) and noting that Q, = Q,, = Q,, = 0, we obtain my +h, (x 34) +h (x, -,)=0 mi, — hy (x, —%,) +h (4, 4) =0 mgs — hy (x =) hy (x, — 5) = 0 which can be written in matrix form as m 0 0)(%) [+k -k -& ][m Om O84] - bth Wk ihe t=0 @ 0 0 mils) | -& -m kh +k I Ls ‘The system has rigid-body modes when the rank of the stiffness matrix is less than the order of the matrix. ‘The number of rigid-body modes is given by the difference between 229 (©2009 Cengage Learning. ALL RIGHTS RESERVED. Solutions to Exercises ~ Chapter 7 the order of the matrix and rank of the matrix. ‘The number of rigid-body modes is at least one if the determinant of the stiffness matrix is zero. Using the MATLAB function rank from the Symbolic toolbox, we find that the rank of the stiffness matrix is 2 and, therefore, there is one rigid-body mode. To determine the characteristic equation, we assume a solution of the form x,=X,cos(wr) =1,2,3 We substitute Eqs. (c) into Eq. (d) and set the determinant of the coefficients to zero to obtain lk, +k, —@7m, =k, k,+k-o'm, -k, 0 -k cand m, = m= m= m, Eq, (f) simplifies to For the special case when ky = ka = 2k-a?m = -k -k = -2k-o'm -k Ke Introducing the notation @ we can rewrite Eq. (g) as jz-a? = -1 -1 -1 2-9 -1 -1 -1 2-97 Expanding the determinant, we obtain 2 (a? 3) Solving for the roots of Eq. Gj), we determine that the real-valued natural frequencies are Q=0, V3, and V3. As expected, there is one rigid-body mode, which corresponds to Q = 0. 7.48 Two disks that roll without slipping on a flat surface are connected to each other by a spring with constant k, as shown in Figure E7.48. Determine the natural frequencies and mode shapes of the system. Solution 7.48 ‘The equations of motion for fiee oscillations will be obtained using Lagrange’s equations. ‘The kinetic energy is 230 (©2009 Cengage Learning. ALL RIGHTS RESERVED. © (3) (@) (n) @ oO where ‘Therefore, ‘The potential energy is Solutions to Exercises ~ Chapter 7 Liealygealy ge > SSO +218 ami +d +5 v $k —n) For this undamped system, the Lagrange equations are a(ar dt\ a, afar dt\ &, Upon using Eqs. (a) and (b) in (c) and noting that the generalized forces are zero, we obtain where 3, +0n(4-%,)=0 H+ en (ax ‘To determine the natural frequencies and mode shapes, we assume a solution of the form x)=X,0s(ot) j=1,2 Upon substituting Eqs. (¢) into Eqs. (4), we obtain where (1-9") x,-X,=0 (03-9) X, -@2X, =0 231 (©2009 Cengage Learning. ALL RIGHTS RESERVED. @ () © @ © © Solutions to Exercises ~ Chapter 7 Setting the determinant of the coefficients in Eqs. (f) to zero, we obtain the characteristic equation The positive roots of Eq. (h) are 2,=0 and the mode shapes are determined from the first of Eqs. (f) as x, eae 42 -(1-03)=-0} = 2 Section 7.3.2 7.49 Show that the linear independence of eigenvectors given by Eq, (7.72) is true by making use of the orthogonality property of eigenvectors. Solution 7.49 Let us suppose that Eq. (7.72) is true. Then, if the eigenvectors are from a linearly independent set, this is the only possible if all of the coefficients cm are zero. To prove this, we pre-multiply Eq. (7.72) by etd and obtain Senior From Eqs. (7.64) and (7.67), we see that BLM =0 tem My i=m ‘Then, 232 (©2009 Cengage Learning. ALL RIGHTS RESERVED. (8) (n) @ ®o Solutions to Exercises - Chapter 7 CAXILMIE XG! = CaM gm Since My, #0, €m=0 for m= 1, 2,...,m. Thus, we have that Eq. (7.72) is satisfied only when the ¢» are zero, and hence, the eigenvectors form a linearly independent set. 7.50 Is it possible for a three degree-of-freedom system to have the following eigenvectors? 0 (hj lp (hay 1p (44,=32 1/2 1/2 1 Solution 7.50 For these eigenvectors to be a valid set, they must form a linearly independent set From Eq. (7.72) and the values given above, this means that He 1 ete +26, = 1 1 patzata=0 For linear independence, all of the cm must equal zero. From the above equations, we see that this is not true, since the second and third equations are identical. Hence, the given vectors do not constitute a valid eigenvector set. Section 7.3.3 7.51 Determine the modal mass, modal stiffness, and modal damping factors associated with the system whose mass matrix, stiffness matrix, and damping matrix are given by the following: 3-2 a} [a 3] Solution 7.51 ‘The natural frequencies are determined from Eq. (7.39); that is mmo —[(k, + ky) m, + (ky + hy), Jo? + (+) (ka) “KE Upon comparing Eq. (7.38a) with the matrices [M] and [K] that are given, we see that ky + ky= 1, ko = 1, ky + by = 2, m = 1, and m, = 2. Then the natural frequencies can be determined from the solution to 1x2x 4 -[1x2+2x1]o*+1x2-P =0 20 -40*+1=0 233 (©2009 Cengage Learning, ALL RIGHTS RESERVED. Solutions to Exercises ~ Chapter 7 or and, therefore, @, =0.541 rad/s 1.307 rad/s The mode shape ratios are found from Eq. (7.40); thus, eee X,, ktk-mo; 1-0} —(1# 2/2) v2 where the positive sign corresponds to j = I (first mode), and the negative sign to j = 2 (second mode). We construct the modal matrix as o-[7P a) Modal ms modal stiffness Then, from Eq. (7.68a,b), we find that the modal mass matrix and the modal stiffness matrix are given by twot-tor wator-[*0 N87) ope aN)1* &) TE PE IPE (&s1-107 tte} -[ Modal damping ‘To determine if the system has proportional damping, we use Eq. (7.99) and obtain 2/v2 aT 3 ay" Wa | 63430 | 1 1 2 6if1 1 0 17.657, ter tetto1-[ Since this matrix is diagonal, the damping matrix has proportional damping form. Thus, the modal damping factors are 6343 $1 Fy4x0541 6-38-1689 2 2<4%1.307 MATLAB program _used to generate numerical reslls for Exercise 7.31 234 (©2009 Cengage Learning, ALL RIGHTS RESERVED. Solutions to Exercises ~ Chapter 7 Phi-(2/sqrt(2) -2isqrt(2); 1 1]; 7.52 Show that the system treated in Example 7.21 is proportionally damped by making use of Eqs. (7.99). Solution 7.52 We determine the modal matrix for the system treated in Example 7.17 and then use Eqs. (7.99). First, for the system given in Example 7.17, we have that the natural frequencies are determined from Eq. (7.39¢); that is mmo! —[(k, + k,)m, + (ky +), Jo? +(k, +k, )(k, +k) Ke =0 Upon comparing Eq. (7.38a) with [M| and [K] in Example 7.17, we see that ky + ky = ky = 1, ky + ky = 1, my = 2, and m = 1. Then the natural frequencies can be determined from the solution to 2x1xe* -[2x1+1x2]o* +2x1-1 =0 2@*-407 +1=0 or v2 gat oi, -19% ‘The mode shape ratios are found from Eq. (7.40); thus, ik 1 %y htk-me; 2-20; 2[i-(1Fv2)2)) 2 where the positive sign corresponds to j = 1 (first mode), and the negative sign to j = 2 (second mode). We construct the modal matrix as fo] eas faa 1) 1 1 1 1 ‘Then, from Eq. (7.99), we have that ele TE ale 9 ite 0 “LO 1.083 235 (©2009 Cengage Learning. ALL RIGHTS RESERVED. Solutions to Exercises ~ Chapter 7 Since [Cp] is diagonal, the system is proportionally damped. MATLAB program used to generate numerical resulls for Exercise 7.52 Phi=[1/sqe(2) -L/sqrt(2); 1 1]; 7.53 To describe the vertical motions of an automobile, the two degree-of-freedom system shown in Figure £7.53 is used. This model is known as a quarter-car model. If the parameters of the system are m; = 80 kg, m2 = 1100 kg, k= 30 kNim, ki = 300 KN/m, and ¢, = 5000 N/(m/s), determine if the system is proportionally damped. Solution 7.53 ‘The natural frequencies are determined from Eq. (7.39) with k mym,o* —[(k, +k,)m, + km, Jo? +(k, +k,)k, —K3 =0 0; that is, or myn! —[(k, +) m, + km, Jo? +k, =0 Then the natural frequencies can be determined from the solution to 80x 1100 a* —[(300000 + 30000) x 1100 + 30000 80]a* +300000 x 30000-30000? = 0 or 88x 10° @* — 365.4 x10°@ + 9x10? =0 Therefore, @; =24.778 rad’/s*, @; =4127.49 rad?/s* 0, =4.98 rad/s, @, = 64.25 rad/s The mode shape ratios are found from Eq. (7.40); thus, xX k 30x10" 1 Xk +k—mo} — 300x10°+30x10°—8007 66710 a? From this equation, we construct the following modal matrix. 236 (©2009 Cengage Learning. ALL RIGHTS RESERVED. Solutions to Exercises ~ Chapter 7 [a] ee falta) (u- 2aer0°ai) ' (11-2.667x10"02) ] 1 f (11-2.667%10° x24.78)' (11-2.667%10° x4127.49) ] 1 [0.0915 -150, os To determine if the system is proportionally damped, we use Eq. (7.99) and (7.1b); thus, 0.0915 -150.3]"[ 5000 -S000]f0.0915 -150.3 1 }|-so00 sooo || 1 1 [es 687500.0 ] [c.]= ~ | 687500.0 114521185.26 Since [Co] is not diagonal, the system is not proportionally damped. format long g ‘cooff=[ml #m2 (kl-+k2)*m2+42*m!) (kI*2)} 12-roots(coeft) rsqrt=sqri(2) Phi=(K2/(k1-+K2-m *r2(2)) k2/(KI-+42-m1*r2(1)); 11] ‘C=[5000 -5000; -5000 5000}; (Ca=Phi"*C*Phi format short 7.54 If the modal matrix and the damping matrix for a two degree-o respectively, wef a] Gg Is this system proportionally damped? Solution 7.54 To determine if the system is proportionally damped, we use Eq. (7.99); thus, eedom system are, 237 (©2009 Cengage Learning. ALL RIGHTS RESERVED. Solutions to Exercises ~ Chapter 7 1 -17'[s oa -1]_[5 elf Hi cE ol HE 3] Since [Co] is not diagonal, the damping is not proportional damping. One could also have used Eq. (7.106) directly to obtain the result. MATLAB program used to generate numerical resulis for Exercise 7.54 Pk 1 C=[5 0; 0}; CD-PhiI*C*Phi 7.55 For the system shown in Figure E7.12, assume that the drive torque M,(‘) = 0. Let Jor = 1 kg-m*, Jig = 4 kgm’, J,3 = 1 kg-m? and the torsional stiffness of each shaft be as follows: ky =k ,=10 Novrad and ,=5 Nmvrad, In addition, let the damping coefficients be such that c,, =0.5 Nmi/rad/s, ¢,=2 Nmv/rad/s, and ¢,=0.5 Nm/rad/s. Determine the following, a) Is the system proportionally damped? b) If the damping associated with the second flywheel changes from 2 Nnv/rad/s to 1.8 ‘Nmv/rad/s, can the system be approximated as a system with modal damping? Solution 7.55 Rewriting the solution for Exercise E7.12 in matrix form, we obtain Jun 9 O)fG) [er 0 O]fH O Sn ON Apt] 0 ep Od 0 0 sald} Lo 0 ella atk, ky 0 Ja) (0) +| ha hatha “kayo t=10 Gos Slt ib To determine the natural frequencies, we set the damping to zero and rewrite the equations as follows: Jy (0 07) Tkytky ky 0 TI (O, -07} 0 J. 0 |+] -ky kathy ka ||}, b= 0 0 Jy 0 =ks ks {Ils Ses which, for the given parameters values is 10 0] [20 -10 oj]f{a, 0/0 4 0|+/-10 15 -S|@,}= O10) TQ. 55 Ilo. oss 238 ©2009 Cengage Learning. ALL RIGHTS RESERVED. Solutions to Exercises ~ Chapter 7 To determine if the damping is proportional damping, we first use the eig function from MATLAB to determine the natural frequencies and the modal matrix. The natural frequencies are , = 0.955 2.529 631 and the associated modal matrix is 0.218 -0.176 -0.960 [o]=|-0.416 -0.240 0.139 0.509 0,860 -0,042 ‘The damping matrix is QO je o 0 [c]l=]0 ¢, o|=]0 2 0 O 0 al oo os To determine if the damping is proportional damping, we use Eq. (7.99) to compute 0.218 -0.176 -0.960]'[2 0 0 ][-0.218 -0.176 -0.960 [Cp]=|-0.416 -0.240 0.139 | |0 2 0 |}-0.416 -0.240 0.139 ~0.509 0,860 -0.042| [0 0 0.5||-0.509 0.860 0.042 0.571 0.058 0.314 =|0.058 0.547 0.254 0314 0.254 1.882] Since the matrix [Cp] is not a diagonal matrix, the system is not proportionally damped. Changing the value of ¢o to 1.8 Nm/rad/s changes the damping matrix [C] to 20 0 [c]=]0 18 0 0 0 05. Then, the matrix product 0.218 -0.176 -0.960]'[2 0 0 [0.218 -0.176 -0.960 0.416 0.240 0.139 0 18 0 |/-0416 -0.240 0.139 0.509 0.860 -0.042} [0 0 0.5]|-0.509 0.860 -0.042 0.537 0.038 0.326 =| 0.038 0.535 0.260 0.326 0.260 1.878 239. (©2009 Cengage Learning. ALL RIGHTS RESERVED. Solutions to Exercises ~ Chapter 7 This damping matrix is a not a good approximation for a system with modal damping, since the off-diagonal terms are not small with respect to the diagonal terms. MATLAB program used to generate the numerical results for Exercise 7.55 M=diag({1 4 1); K=[20-10 0; -10 15 -5; 0-5 5]; C-diag((2 2.0.5); [modes E}-eig(K,M) Ext-sqrt(E) (CD=modes"*C*modes Cnew=diag(2 1.8 0.5); L cDnew=modes'*Cnew*modes 7.56 The eigenvalues associated with a damped three degree-of-freedom system are given by the following: Ay,, = OF 70.995 Ay, =OISF 1.299 Ay, = OAF 1.960 Determine the system natural frequencies, modal damping factors, and damped natural frequencies. Solution 7.56 We recall Eq. (7.97), which is and compare it to the form Then, from which we have This leads to oe ‘Then, the natural frequency can be determined as 240 (©2009 Cengage Learning. ALL RIGHTS RESERVED. Solutions to Exercises — Chapter 7 o, = Ss, Making the appropriate substitutions into the last three equations, we find that k G Tay (rads) a (rads) 1 OL | 1.0 0.995 2 os 15 1.299 3 02 20 1.960 Section 7.3.4 7.57 Consider free oscillations of the gyro-sensor treated in Example 7.4 and determine if energy is conserved in the system. Solution 7.57 For free oscillations, fz = 0 in Eq. (d) of Example 7.4, and the equations governing the gyto-sensor reduce to afte} rf} ta} fo Equation (7.109) can be used in the present case to obtain 4(htay nts} +5 69) Lelts)) 08" elt} Duc to the presence of damping in the system 4 (4a) lta + Lb ExIts}) #0 a2 that is, the system energy is not conserved. 7.58 Consider the three degree-of-freedom system shown in Figure £7.31 and determine if the linear momentum and the total energy of this system are conserved. Solution 7.58 For the system of Figure E7.31, since there are no extemal forces acting on the system along the horizontal direction, the linear momentum along this direction is conserved; that is jai + jmat + jmat = constant In addition, since there is no damping in the system, Eqs. (7.111) lead to jmat hms + hms +h (x, -3,)' +4 (4 —,)' =constant 241 (©2009 Cengage Learning, ALL RIGHTS RESERVED. Solutions to Exercises ~ Chapter 7 that is, the total energy of the system is conserved, Section 7.5 7.59 The eigenvalues determined for two different three degree-of-freedom systems are as follows: @ 4,>0,4>0 a,>0, 6, >0 a,>0, b,>0 (b) a,>0, &>0 @,>0, ,>0 =b,; a,>0, ,>0 Determine which of these systems is stable, Solution 7.59 In case (a), since the real part of each eigenvalue is negative, the system is stable. However, since the real part of one of the eigenvalues in the third pair in case (b) positive, the system is unstable. 242 (©2009 Cengage Leaming. ALL RIGHTS RESERVED. Solutions to Exercises ~ Chapter 8 Solutions to Exercises Chapter 8 Section 8.2.1 8.1 Consider the two degree-of-freedom system shown in Figure E8.1, where a forcing fs is imposed on the mass mz. For m= my = 1 kg, kj = 2 Nim, ko = 1 Nim, and fg = F:sinoot N use the normal-mode approach to determine the solution of the system. Assume that all of the initial conditions are zero. Solution 8.1 We follow the procedure outlined in Section 8.2.1 to determine the response of this system, From Eq. (7.1b) and the information given in this exercise, we find that the inertia and stiffness matrices are, respectively, wa fh ef 7] and the governing system of equations is fe} eC} =[a ae Using the eig function from MATLAB, we find the system natural frequencies 0, = 0.7654 rad/s @, =1.8478 rad/s and the modal matrix o-[Xe Xn] _[-03827 09239 ~0.9239 0.3827 Xy Xa] Using Eq. (7.68a) and Eq. (8.7), we obtain the modal mass matrix iP 0.3827 -0.9239]"[1 Of 0.3827 -0.9239 [ato]=[0F MI I=| 56939 9.3827 0.9239 0.3827 i and the modal force vector 243, ‘©2009 Cengage Learning. ALL RIGHTS RESERVED. Solutions to Exercises — Chapter 8 resis .[ 1 0] [0.3827 -0.9239]' fo) {o}=[¥o] To} tly H| [eons oe {ipso faz) = Fysinot 0.3827 Then, from Eq. (8.12) and noting that the system is undamped, the equations governing the modal coordinates are Oran (O=O() f=1,2 which, on substituting the numerical values take the form Hi(0) + 0.58587, (1) = -0.9239F, sin ot fi (t) +3.4142n,(1) = 0.3827F, sino The solutions to these equations are obtained by assuming a solution of the form n(Q)=N,sinow Upon substituting this equation into the modal equations, we obtain the following algebraic system -o° N, + 0.5858N, = -0.9239F, oN, +3.4142N, = 0.3827F, These equations are solved to determine that After using Eqs. (8.17), we obtain the system displacements x; as O=XynO+ XaMmO = (0.3827) —0-9239F sin ot + (-0,9239) 038275 gin wp 05858-0 3412-0 03536 0.3536 ) : =( 03536 _ 0.3596 —) sino 34M2—-0")? and 244 (©2009 Cengage Learning. ALL RIGHTS RESERVED. Solutions to Exercises — Chapter 8 (= Xn (O+ Xn = (0.9239) 092392 sin nt + (0.3827) 382 sin ot 05858-0 3412-0) ( 0.8535 0.1465 = {28839 ___0.1409 |r, sinor 0.5858—@? 3.4142-07 MATLAB program used to generate some of the resuls for Exercise 8.1 M-{1 0; 01} Ke[3 -1; -1 1); [modes E]-eig(K.M) wesqrtE) MD=modes"*M*modes Q (MD)*modes'*[0 1] 8.2 Consider the two degree-of-freedom system shown in Figure E8.2, where my = m= 1 kg, k= 2. Nim, ky = 1 Nim, c = 0.4 Nims, c2 = 0.2 Nim/s, and fs = f(). Starting with Eq. (7.1b), uncouple these equations by using the modal matrix and present the uncoupled system of equations. Solution 8.2 We start with the governing equation given in the solution to Eq, (7.11) with ks = cs = eis SIGH SRM EH) Upon substituting the parameters given in the problem statement, we have that 1 o7f%] ,[ 06 ~02)(%] [3 Ifa] _f o o1 sf -02 02 {aS "1 1 JlxJ lA. Since the mass and stiffiness matrix are the same as those in Exercise 8.1, we have from this exercise that the natural frequencies are @, =0.7654 rad/s @, = 1.8478 rad/s and the modal matrix is _[-0.3827 -0.92397 -0.9239 0.3827 To uncouple the equations as in the form of Eq, (8.11), we have to determine the matrix quantity given by Eqs. (8.9) and (8.10), that is, [(2)]=[MoJ'[Co]=[Mo}' [oF [ello] Thus, 245 ‘©2009 Cengage Learning. ALL RIGHTS RESERVED. Solutions to Exercises ~ Chapter 8 [(2¢@)]=[Mo}'ICo] _fl op'[-0.3827 -0.9239]'[ 0.6 -0.27[-0.3827 -0.9239 “Lo 1} [-0.9239 0.3827 | |-0.2 02 || -0.9239 0.3827 _fo172 0 “| 0 0.6828 From Eq, (8.12), the uncoupled equations in terms of the modal coordinates are (026,01, +0nD=Q j=12 where Q,(0) is given by fol=[M.] Te] 1} +} 1 [ae ll { 0 } ol 0.9239 0.3827 f,0, 2 {oe} 0.3827 4,10 ‘Then, these equations become y(t) + 0.117219 (0) + 0.5858 7, (1) =-0.9239 f,() iis (t) + 0.68287), (4) +3.4142n, (1) = 0.3827 f,(0) MATLAB program used to generate some of the resulis for Exercise 8.2 Me={1 0; 0.1]; K-[3 -1; wesqrt(E) MD todes!*M*modes Qeinvab)*modes*(0 1) ZW=inv(MD)*modes'*C*modes 8.3 Use the normal-mode approach to determine a solution for the response of system discussed in Exercise 8.2 when an harmonic forcing f: = 10 sin(20/) N is imposed on mass m,. Assume that all of the initial conditions are zero and ignore the transient portion of the response. Solution 83 Since the system is underdamped, as we know from the solutions to Exercises 8.2, we start from Eq. (8.13), note that the ial conditions are zero, and obtain n= fe sintoglt EDO, (45 4 246 ©2009 Cengage Learning, ALL RIGHTS RESERVED. Solutions to Exercises ~ Chapter 8 From Exercise 8.2, we have that the modal force vector is, (0)-{2)-{% basinor={ Pooh n= ah i eK Sin(w yt -E) sinogdgs Oy 9 sinot ‘Thus, we have that ‘The evaluated integral can be determined from Eqs. (5.5) and (5.8). The steady-state portion of these results provide n,(0) =@,F,H, sin(wt-0,) where From the solution to Exercise 8.1, we have that , =0.7654 rad/s @, =1.8478 rad/s and from Exercise 8.2 that 2, =0.1172 26,00, = 0.6828 ‘Then, for the given excitation frequency « = 20 rad/s, the respective functions evaluate as H, =-——_____—_- 0.005 (0.7654? 20°)’ +(0.117220)" H,-——__ 9.0025 (1.8478? — 20°)’ + (0.6828 20)" 1 0.1172%20 51357 sag 0.76547 -20? = tant 0-6828%20 a, 19478720" 1.8478? 20° =3.1072 rad Noting that the excitation amplitude F~ 10'N, we obtain the modal responses 247 ‘©2009 Cengage Learning. ALL RIGHTS RESERVED. Solutions to Exercises ~ Chapter 8 , Sin (wt -8,) = -0.9239 x10 x 0.0025sin (201 ~3.1357) 023sin (201 -3.1357) 1,(0) = &, FH, sin (wt - 0,) = 0.3827 x 10 x 0.0025sin (20r 3.1072) .01sin(201—3.1072) Making use of the modal matrix used in the solution to Exercise 8.1 and Eq. (8.17), the response of mass m; is determined as HO = XunO+ Xv = 0.3827 x 0,023sin(20r -3.1357) — 0.9239 x 0.01sin (20 —3.1072) =-8.8% 10" sin (20/—3.1357) -9.2x10° sin(20¢—3.1072)_ m mO= ‘The responses of mass m, is (= Xp + Xn = 0.9239 x 0.023 sin (201 —3.1357) + 0.3827 x 0.01sin (20 ~3.1072) = 21.2 x10” sin(20/—3.1357) +3.83 x10” sin(20r—3.1072)_ m 8.4 Use the normal-mode approach to determine a solution for the response of the three degree-of-freedom system shown in Figure E8.4 when an impulse f= F36(0) is imposed on mass mz. Assume that all of the initial conditions are zero and that my = mp = ms =m and that k = ke = ky =k. Solution 8.4 From Eq. (7.5b) for N= 3 and c = are 3 = 0, we have that the governing equations m 0 O7Vf%) [hth he 0 V[x, 0 om 0 +| -b& bth -beb={ E60 0 0 mils 0 -&h & ils o To use the normal-mode approach, we first determine the natural frequencies and mode shapes from the free-oscillation problem. To this end, we set = 0 and assume a solution to the governing equations of the form (0) [X,cosor (1) }=}.X, cost x0) [Xycosar Then, we obtain i 0) 0] 2-1 on|(x,) {6 -O7/0 1 Ol+}-1 2 -1//)¥,}=40 oo 1} lo - 14x} lo 248 (©2009 Cengage Learning. ALL RIGHTS RESERVED. Solutions to Exercises ~ Chapter 8 where Affier using the eig function from MATLAB, we obtain the natural frequencies 0.4450, rad/s 1.2470, rad/s 0, =1.8020, rad/s and the modal matrix Xy Xy Xy] [-0.328 0.737 -0.591 =| XX, X,|=|-0.591 0.328 0.737 X Xy_ X,) [0.737 0.591 0.328 Since the system doesn’t have any damping, we use Eq. (8.20) to obtain w x)= >> to, where, from Eq. (8.7), the modal force vector is {o}=[M.} [oy tr} ‘The modal mass matrix is given by []=[0] [lle] 0.328 0.737 -0.591/ [1 0 Of-0328 0.737 -0,591 =|-0.591 0328 0.737] |0 1 of] -0591 0328 0.737 0.737 -0.591 0328] |0 0 1||-0.737 -0.591 -0328 100 =|0 1 0 oo1 sin(om,¢-EDO,(Eae 8 |b ‘Thus, we determine that Q) [1 0 o7'f—0328 0.737 -0.s91] (0 {Q}={0,}=|0 1 0} |-0.591 0.328 0.737 | 1b 6(D 2} [0 0 1) [-0.737 -o591 -0328] [0 B -0.591 =} BS F6(1) =4 0.328 }Fd(0) b, 0.737 249, (©2009 Cengage Learning. ALL RIGHTS RESERVED. Solutions to Exercises ~ Chapter 8 Therefore, the displacements of the masses are (=H finco ft - V0, @)ds fo =D fointo tren 8, nous =A YA sino ft ®, where the f, are as previously determined. MATLAB program used to generate some of the resulls for Exereise 84 Mediag({(1 11); Ke[2-10;-12-1;0-1 1; [modes eiJ=eig(K.M) | B-sart(ei) MD=modes'*M*modes Qinv(MD)*modes'*[0 LO} 8.5 Repeat Exercise 8.4 with an impulse fi = F,4() imposed on mass m, instead of mass m. Solution 8.5 ‘The form of the last results of Exercise 8.4 remains the same. The only change is in the value of , which can be found from Q) [1 0 of'[-0328 0.737 -os91]' [1 {Q}=10,t=|0 1 0} |-0591 0328 0.737 | Jo}RS() Q,) [0 0 1} |-0.737 -0.591 -0.328] [0 B, 0.328 = BRO) =} 0.737 LS) B ~0.591 [ MATLAB program used to generate some of the resulls or Exercise 83] Mediag({1 11); K=[2 -1 0; Qeinv(MD)*modes'*[1 0 0} 250 (©2009 Cengage Learning. ALL RIGHTS RESERVED. Solutions to Exercises ~ Chapter 8 Section 8.2.2 8.6 Consider the three degree-of-freedom system shown in Figure E8.6. The different system parameter values are as follows = kgm’, Jig = 4 kgm’, Jos = 1 kg-m?, ka = k= 10 Nem/rad, and ky =5 N-m/rad. Assume that the damping matrix is proportional to the inertia matrix; that is, mH OO te oO 0 0c O}=a) 0 J, 0 0 0 c, 0 0 Igy where a When the external moment M,(0) is absent, determine the response of this system by using the normal-mode approach for each of the following sets of initial conditions and discuss the participation of different modes in each case. 8) $1(0) = 92(0) = ga(0) = 1 rad 4,(0) = 4,(0) = 4,(0) = 0 rad/s ) #i(0) = #x(0) = 1 rad, #a(0) = -0.5 rad ©) (0) = 9,(0) = 4,(0) = 0 rad/s. Solution 8.6 ‘The governing system of equations in matrix form is given by Vite} +[c]{e} +4. to} = (0) but, since [C] = al], [}te} +e 7]te} +14 Ho} = {0} where Jo 0 0) [100 [4]=] 0 Jo: 0 |=|0 4 0} kg-m? 0 0 4} [0 01 and, from Eq. (7.5b), with appropriate changes in notation, we arrive at the stiffness matrix kath, ke 0] [20 -10 0 [K]=] -k kath; -ks|=]-10 159-5] N-m/tad 0 ky hy 0 35 5 After using the procedure to arrive at Eq, (8.6), we transform the governing equations of motion into the following system in terms of the modal coordinates fi} +e{a} +[2%5 fn} =(0} where 251 ‘©2009 Cengage Learning, ALL RIGHTS RESERVED. Solutions to Exercises ~ Chapter 8 {9} =[0]{7} (23 ]=[4T "Kol (]=[eF Vo] [Kol=[#T [&,][2] The modal matrix [©] is obtained by solving the eigenvalue problem associated with the undamped system. By using the eig function from MATLAB, we obtain the natural frequencies ©, =0.9548 rad/s @, =2,5287 rad/s @, = 4.6308 rad/s and the modal matrix Xz Xy] [-0.2181 -0.1762 -0.9599 Xy X,|=|-0.4163 -0.2398 0.1386 X_ Xy] [0.5091 0.8597 -0.0421 After using Eq. (8.19) with appropriate changes in the notation, we have that the responses of the masses are 90 Yo x46" sin(ogt +95) where A, and gy are determined from the initial conditions. From Eq. (8.15), the modal damping factors are obtained as a@__ 0s 6= =? _ 0.262 8120, 20.9548 G= = _<0.097 2a, 2*2.5287 eZ oe O54 2, 24.6308 and the damped natural frequencies are given by ©, = 0,1 ~G} =0.9548V1—0.262? = 0.921 rad/s 0, =0,\ 1G} =2.5287Vi- 0.097 =2.52. rad/s ofl —G} = 4.63081 — 0.0547 =4.62 rad/s (he initial conditions in terms of the modal coordinates are determined from O% 252 (©2009 Cengage Learning. ALL RIGHTS RESERVED. Solutions to Exercises — Chapter 8 £70}=[40] [2] te} {0} =[4,) [2] Lo} (a) Since the initial velocities @,(0) = 0, the modal velocities {0} = {0} ‘The initial modal displacements are {10} =[4.] [TF [7]{e@} (0) [1 0 O]'[-0.2181 -0.1762 0.95997! 1 100 n(0)>=|0 4 0] |-0.4163 -0.2398 0.1386 | |o 4 0/41 m0) [0 0 1] |-05091 0.8597 -0.0421] |0 0 1] [1 2.3924 = 40.2756 0.4476 For these initial conditions, the amplitudes are a w+(Sa0) ey xian OLE 1,0 Therefore, and rad 253 (©2009 Cengage Learning, ALL RIGHTS RESERVED. Solutions to Exercises ~ Chapter 8 ‘The system response is given by a) [Xu Xn Xp ](nO POP=|Xun Xn Xn lymO PO} [X Xn Xn} HO. Hy Xp Ky] { Ae sin(oyt-9y) =|Xn Xa Xx |} he sin(o,t-9,,) Xs Ky Xp} de sin(@pt- 2.) which leads to g(t)) [0.2268 -0.1771 -0.9608 fe 250% sin(0.921¢ -1.306) (0) > =| -0.4330 -0.2410 0.1387 |{ e°*™ sin(2.52r-1.474) g(t)} [0.5295 0.8640 -0.0422 }| °°" sin(4.62t - 1.517) All three modes participate in the torsional response of each of the flywheels. However, the participation of the third mode is strong in the torsional response of the first flywheel and weak in the torsional response of the third flywheel. The participations of the first and second modes are the strongest in the torsional response of the third flywheel. (b) For this set of initial conditions, the amplitudes become 04 rad and the phase angles become 112 0:262" 1306 rad = tan oa 0.262 fzo.0se® ay = tan? Y1=0.054" 1517 rad 0.054 ‘The system response is given by 254 (©2009 Cengage Learning, ALL RIGHTS RESERVED. Solutions to Exercises ~ Chapter 8 MO) [Xu Xn Xs] [nO PO 0=|X Xn Xn |ymO PO} [Xn Xa Xs JUnO. Xn My Xs][ Ae" sin(ont—9,,) 2 Xa |) Ae sin(,.t ~ 942) Xy Xn Ay] | Ae" sin(oyst—9,3) which leads to @(D) [-0.2268 0.0885 -0.9608) fe“? sin(0.921 -1.306) (0) > =| 0.4330 -0.1204 0.1387 }) e°** sin(2.52r-1.474) gs} [0.5295 0.4315 -0.0422] | e°*" sin(4.62r-1.517) Again, in this case, all three modes participate in the torsional response of each of the flywheels. However, the participation of the second mode in the torsional response of each flywheel differs from what was noticed in the previous case. In the torsional response of the first flywheel, the second mode has the weakest participation. In the torsional response of the third flywheel, the third mode has the weakest participation. Mx dio. me of i iag(l1 4 1 Ds Kt-(20-10 0; -10 15 -5;0-5 5]; {modes eil-eig(Kt) E=sqri(ei) ‘ID=modes'*J*modes KiD-modes'*Kt*modes ceta0=inv(JD)*modes"*J*[1 1 1] ‘A=[1.04, 0.502, 1.001] for Phi(k)-modes(k)*A(K); end Phi 8.7 Consider the damped system of Exercise 8.2 and set the forcing amplitude to zero. Examine the free oscillations of this system by using the normal-mode for the following sets of initial conditions and discuss the participation of different modes in each case: (a) x1(0) = x2(0) = 0.2 m, %(0)=%,(0)=0 m/s, and (b) x(0) = -0.2 m, x(0) = 0.2 m, ¥(0) = %,(0) =0 mis. Solution 8.7 ‘Since we know from the solution to Exercise 8.2 that the system is underdamped, we will start from the damped free response given by Eq, (8.19); that is, 2 4() =X, Ae" sin(oyt+ oy) 1=1,2 ni From Eq. (8.14), we have that 255 (©2009 Cengage Learning, ALL RIGHTS RESERVED. Solutions to Exercises ~ Chapter 8 0) -( 4,0) on %% tant 24) 4 FOE on, O and, from Eqs. (8.25) and (7.67), (0) ={X}; [M]{x(0)}/a7, 4) =(X} [M]2)}/47, My = {x} [MI{X}, J=1,2 From the solution to Exercise 8.2, the natural frequencies are @, = 0.7654 rad/s @, = 1.8478 rad/s and the modal matrix and the damping matrix are, respectively, o-[%u Xn] _[-03827, -0.9239 Xn Xn} [-0.9239 0.3827 0.1172 0 | [(2s0)] -| 0 0.6828 ‘The modal mass matrix is [at] 0.3827 -0.9239]'[1 Of -0.3827 0.9239] 21 |-0.9239 0.3827 | [0 1||-0.9239 0.3827 | al (a) The initial conditions in terms of the modal coordinates are ah ajoafig leo j=12 41,)= ‘Therefore, 256 (©2009 Cengage Learning. ALL RIGHTS RESERVED. Solutions to Exercises — Chapter 8 bs cia Ot {0.3827 -0.9239} 1 0)/02) | 4.2613 0 14{02. (0) x, (0), 0) {0.: {0.9239 0.3827} eee =~0.1082 0 1f\02 {Xe situa ‘The modal damping factors are 0.1172 $'=3 50.7654 0.6828 = 2.0828 _ 9.185 $51 8478 and the damped natural frequencies are 4 = oy —G? =0.7654¥1-0.077 =0.763 rad/s nl G? =1.8478V1- 0.185? =1.816 rad/s ‘The amplitudes and phase angles of the responses are w (Sen ®) _ | V3 =0.077 Oy = Be 1 MOG 0) which give 0.2613 = =0.262 m Vi-007F [Notice that the signs of the initial displacements in the numerator and denominator of the aretangent function must be explicitly stated so that the proper quadrant can be determined] 257. (©2009 Cengage Learning. ALL RIGHTS RESERVED. Solutions to Exercises ~ Chapter 8 ‘The response of mass m, is 3) =X Ae sin(oyt +9.) + Xp Ae sin(yst + Oya) = (-0.3827) (0.262) 2°" sin(0.763t — 1.648) +(-0.9239) (0.110) e-@1"*""" sin(1.8161 ~1.757) = =0.10e"°** sin(0.7631 — 1.648) -0.102e°°*” sin(1.8161~1.757) m Tapas componatineBintwale——Regunie anpesat ne cosmos and the response of mp is 2D =X Ae sin +9) + XyAeO™ sin(@pyt + Py) = (-0.9239) (0.262) °°" sin(0.7637 — 1.648) +(0.3827)(0.110) e-@"%"*"™ sin(1.816¢ ~ 1.757) = ~0.242e°"™" sin(0.7631 — 1.648) +0.042 le?" sin(1.816f—1.757)__m ean epee de Tea cape nt lade Notice that when = 0 the resulting values for x(0) equal the given initial conditions. Both modes participate in the response of mass m and m respectively. However, amplitude-wise, the participation of the second mode in the response of mass. m is weaker. (b) The initial conditions in terms of the modal coordinates are ts, jnafSOfe0 sta Therefore jot eet *,(0) ate |-0.3827 von te | =-0.108 nO=T(Xy Xa wale aio: 9239 0. a(t he: on ‘|. 0.2613 ‘The amplitudes and phase angles of the responses are 258 ©2009 Cengage Learning. ALL RIGHTS RESERVED. Solutions to Exercises ~ Chapter 8 Pay = tant ITO." 81385 rad ‘The response of mass m is X(N =X Ae sin(@nt + 9) + X.A,eO sin(@yst + Py) (-0.3827) (0.109) ©" sin(0.7631 ~1.648) + (0.9239) (0.266) e°18%"#"®" sin(1.8161 +1.385) 0.0417¢°"™" sin(0.763t —1.648) -0.246e"°*™ sin(1.8167+1.385) m Resp axipanct teint mode Response compen ini ed nae and the response of mis X1(1)= Xp, Ae sin(Ogt +9) + Xp Ae™ sin. + Py) = (-0.9239) (0.109) e°°"%°7" sin(0.7631— 1.648) + (0.3827) (0.266) e°*'*"*" sin(1.8167 +1.385) = 0.1016 sin(0.7631—1.648) +0.1026°°> Repo smpont be Rina Response cpm inti caine m Both modes participate in the response of mass m; and mass m., respectively. However, amplitude-wise, the participation of the first mode in the response of mass m is weaker. MATLAB program used to gunerate some of he results for Exercise 87 Me{1 0,0 1} K=3 -1;-11); C-(0.6-02;-02 02}; {modes E)-eig(K,M) wesqrt(E) MD-modes'*M*modes Q=inv(MD)*modes"*(0 1} ZW=inv(MD)*modes'*C*modes part (a) modes(:,1*M*[0.2 0.2}/MD(I,1) 1n20=modes(-2)'*M*[0.2 0.2]/MD(2,2) 21=0.077; 22°0.185; pdl=atan2(-sqrt(1-212),21) pa2-atan2(-sqrt(1-22/2),22) 259, (©2009 Cengage Learning, ALL RIGHTS RESERVED. Solutions to Exercises ~ Chapter 8 ‘al 0=modes(1)*M*[-0.2 0.2]7MD,1) 1n20-modes(:2)'*M*[-0.2 0.2]/MDQ,2) Section 8.2.3 8.8 Consider the two degree-of-freedom system shown in Figure E8,2, where m= m3 = | kg, ky = 2 N/m, kp = 1 N/m, ci = 0.4 Ns/m, and c= 0.2 Ns/m. To determine the response of this damped system to a harmonic excitation, one can assume the excitation to be of the form Sl = Foe which is referred to as a complex-valued excitation because of the e term. To determine the steady-state solution for the response of this system, assume a solution of ‘the form 0] _ (X00) | x0)” X,G0) and then solve for the response amplitudes X\(jw) and X2(jw) from the governing equations of motion. Solution 8.8 Starting from Eq. (7.1) with ¢3 = ky (0 leh? SECS Tee} 0 miles | -q a Slaf"L-m & fle lhe We assume a solution of the form (0|_[X,Ge)| a {rol Se substitute into the governing system and obtain [rete +c,)-mo? — -(joc, +k) nat A = 0, we obtain -(Joe, +k) ky + jac, -mo* ||X,(jo)J [F Solving for Xi(jo) and Xa(j«), we obtain [see also Eqs. (8.73), (8.74), and (8.102)] jQ+a, srygey = Filed) km,D,(jQ) x.) = F,(-Q? +2G, +Sm,@,)jQ+1+m, ym, Dy( JQ) where (©2009 Cengage Learning. ALL RIGHTS RESERVED. Solutions io Exercises ~ Chapter 8 D,(jQ) =O! — j126, +26,0,m, + 2£,0,}2° — [1+ m,07 +07 +46,6,0,]0° + j2¢,0, + 26,07]0 +07 Upon using the given values, we find that oun v2 rad/s, enn ae 1 rad/s o, Ee m, 1 4G 04 i= = 0.1414 Amo, 2x1xV2 & 028 Ta 2mo,, 2x1x1 and, therefore, (20.1« jQ/V2 +0. Xo) = 2x1 x D3(JQ) * +20. 1414+0.1x1/V2) jQ+1+1x05)F, X,(jQ) =) EE 2x1x D,JQ) where D,(4O) =O ~ j[2%0.1414 + 20.1x1/V2 +2x0.1/ V2] ~ [14 1<05+0.5-44%0.1414«0.1/¥2 J? + j[2%0.1/V2+2%0.1414x0,5]0 +0.5 =! 0.5657 jQ’ — 2.04? + 0.2828 j2 + 0.5000 __(0.5+0.1414jQ)F Z 0.5657 jQ" ~ 2.040? + 0.2828 70 + 0.5000) (15-0? +0.4243 70) F, —0.5657 jQ° — 2.0407 +0.2828 jO +0.5000) XO) => 20M) = 261 (©2009 Cengage Learning. ALL RIGHTS RESERVED. Solutions to Exercises ~ Chapter 8 8.9 Consider the system of flywheels shown in Figure E8.9. For an applied moment Mgt) = Mycosot, determine a solution for the response of the system by using the direct approach presented in Section 8.2.3. Solution 8.9 ‘The system shown in Figure E8.9 is a special case of the system shown in Figure 7.3 Noting that the present system is undamped, we determine from Eq. (7.2b) that the governing system is i Ig} Selo We assume a solution of the form ®, {A} fees #10} [®, substitute it into the governing equations and obtain [1+ 02m, -07]®, -m,02@, = M,/k,y 070, +[@} -2°]o, where Solving for ®,, we obtain where Thus, the system response is given by 262 ©2009 Cengage Learning, ALL RIGHTS RESERVED. Solutions to Exercises ~ Chapter 8 coset {Ae} A(0, 8.10 Consider the system shown in Figure E8.10 in which the three masses mi, mz, and ‘ms are located on a uniform cantilever beam with flexural rigidity £/. The inverse of the stiffiness matrix for this system [K], which is called the flexibility matrix, is given by 27:14 «4 ET = 148 25 425 1 If the masses of the system are all identical; that is, m =m = ms =m, then determine the response of this system when it is forced sinusoidally at the location of mass m2 with a forcing amplitude F> and an excitation frequency . Solution 8.10 ‘The general form of the equations governing the response amplitude of an undamped undergoing harmonic excitation with frequency « is ([x1-0 [My] yy =} where [&] is given in the problem statement and m 0 0 100 [M]=|0 m O]=mjo 1 0}=m{[s] oom) lo 1 0 ¥ lrl-teh, (rh={x, 0 Xs Multiplying the above matrix equation by [K]', we obtain (Hl KT do) =a} or al 01.0 27:14 «47f1 0 O}][y 27 14 4 )7(0 0 1 Oj-9/14 8 25/0 1 0 y}=4 4 8 25/1 001 425 1]f0 0 afily} "La 25 1 Jo where 263 (©2009 Cengage Learning. ALL RIGHTS RESERVED. Solutions to Exercises ~ Chapter 8 Upon carrying out the matrix multiplications we arrive at 100) [27 14 4]/[x% 4 01 o|-c14 8 2s\tyt-41s 00 1 ase tres ‘Thus, the response amplitudes at any excitation frequency Q for harmonic forcing on the second mass are given by 4 | fi-27a? -140? 40? T' (14 y,}=2) 14g? 1-80? -250°| | 8 may x, °| 4g? 2.50? 1-0? | [2.5 8.11 Consider the three degree-of-freedom system shown in Figure E84. Assume that a harmonic forcing fs = F,coswot is imposed on the mass mp. Determine if the mass m, can have a zero response at any of the excitation frequencies. Solution 8.11 From Eq, (7.56) for N= 3 and c = ¢ = c; = 0, we have that the governing equations are m 0 OJf%) [+k -k 0 ][x 0 0 m O/}% b+) ky tk, -k]}x, b=) Reosar 0 0 mlz, 0 - & ily 0 ‘We assume a solution of the form x0] {X,coson x) }=4X,cosor x()} [X,cosot and substitute into the governing equations to obtain k, +k, —mo” -k, 0 xX, 0 -k ytk-mo? kX b=4 0 k,-ma* | |X, 0 which can be solved to determine the response amplitudes Xj. Here, we are interested in finding the values of @ for which Xi will be zero. The numerator of the solution to this system of equations for X; is obtained from 264 (©2009 Cengage Learning. ALL RIGHTS RESERVED. Solutions to Exercises ~ Chapter 8 0 0 Numerator of X,=det|F, ky +k,—myo? =k, |= Fi (ky —mo*) 0 -k, k,- mo’ ‘Thus, the response of mass m; has a zero when Physically, at this frequency, mass m; behaves as though it is a fixed end; that is, mass my also has a zero response apart from mass m. 8.12 For the system given in Exercise 8.1, carry out the following: (a) determine a solution for the response of this system by using the direct approach discussed in Section 8.2.3 and (b) construct the frequency-response functions Hia(Q) ard Hox() and plot these functions as a function of the nondimensional excitation frequency ©. Solution 8.12 ‘The system shown in Figure E8.1 is a special case of that shown in Figure 7.1. Upon using Eq. (8.33) with ks ~ fi ~ 0 and / = Fsinart, we obtain the governing equations [> ates SICH a) We assume a solution of the form (oh ino where the amplitudes .X; are to be determined. After substituting the assumed solution into the governing equations, we obtain [he all ta} After using Eqs. (8.35) and (7.41), we arrive at [1+a7m,-0" ]x,-m,o}X, =0 02%, +[0} 07] X, = 03 F,/k, where, from Eq. (7.41), o, = JA p-# radis, = ee flat rad/s m VI m, Vt o,-2n- 1m a2 ®, oe ™ 265, ©2009 Cengage Leaming. ALL RIGHTS RESERVED, Solutions to Exercises ~ Chapter 8 Solving for X,, we obtain Emo x, =a ED, x ehh +oim,-2") kD, where D, = 08 -aQ +a, le o}(+m,)=1450+1)=2 Then, the magnitudes of the responses are 025K, QF 207 +05 0.5(1.5-9°)F, 08-207 +05 ‘Thus, the system response is given by 0.25F, x(t) Of - 207 +0.5 {20h ~“Jos(15-2%) A [nt QS -207 +05 (b) The non-dimensional frequency-response functions are given by H4(0) =A = 98 Bik, OF=20? +05 X, _ 05(1.5-9") Rijk, 28-207 +05 H,,(Q)= which are plotted below. Solid lines are used for Hi2(Q) and broken lines are used for HQ). 266 ©2009 Cengage Learning. ALL RIGHTS RESERVED. Solutions to Exercises — Chapter 8 H (DH, (0) Figure £8.12 ‘MATLAB program used to generate figure for Exereise 8.12 Om=linspace(0,2.201); F-sort(sqr(roots({l -2 0.5))); regl=find(Omsr(1); reg2=find(Om>r(1}&Om<12)); reg3-find(Om>r2 H12-0.25./(Om.4-2*Om.'2+0.5);, H22=0.5%(1.5-Om."2).(Om.4-2*Om_20.5); plot(Om(reg!),H112(ceg]),-, Om(reg2),H12(reg2)/k-Om(reg3),H112(reg3),k-) hold on plot(Om(regl),H22(regl),k~",Om(reg2),H22(reg2),k-~Onn(reg3),H22(ceg3):k~) 3hyR~[r2) r2)MI-2 31K) plot(0 2},{0 O1-k-'feC1) (0H plot({1.5,1.65}.(2.5.2.5}, text(I.7,2.5/H_{12}(\Omega)' fontsize,14) plot(1.5,1.65}{2.2.)/k--) text(1.7,2,,H_{22}(\Omega),,fontsize’,14) waxis; v3)=-2; v(a)=3; axis(v) text(e(1)+0.22,[\Omega_1~' num2str((1))}fontsize’14) text(r(1)+0.2,1-5,[\Omega_2=" num2str(r(2))}fontsize’,14) xlabel(\Omega'fontsize’ 14) ylabel(H_{12}(\Omega) H_{22}(\Omega)’ Set(gea,fontsize’\4) ntsize' 14) 8.13 For the system given in Exercise 8.2, carry out the following: (a) determine a solution for the response of this system by using the direct approach discussed in Section 8.2.3 and (b) construct the frequency-response functions Hi,(@) and H2\(@) and plot these functions as a function of the nondimensional excitation frequency Q. 267 (©2009 Cengage Learning. ALL RIGHTS RESERVED. Solutions to Exercises ~ Chapter 8 Solution 8.13 ‘The system shown in Figure E8.1 is a special case of that shown in Figure 7.1. Upon using Bq. (8.33) with ky = fs = 0 and fi = Fisinot, we obtain m 071%] [hth ke] fn] _[Asinor 0 miles | eb Ilse 0 a) We assume a solution of the form xO] _ JX =47! bsin ot (1), xX, where the amplitudes X, are to be determined. After substituting the assumed solution into the governing equations, we obtain k, +k, -@'m, -k, X|_fhi ky k,-o*m, |X to After using Eqs. (8.35) and (7.41), we arrive at [1+ fm, -07].x, -m,02X, = K/k, -o}X, +[@} -0 |x, =0 where, from Eq. (7.41), tere Pee @,=, |b = JE =1 rad/s, = fe ft ad/s aE eis Ont m2 o o, Q V2" Solving for X;, we obtain where D,=08 -a,Q +a, +o}(1+m,) 145041) =2 1 2 of =— 2 Then the magnitudes of the responses are 268 ©2009 Cengage Learning. ALL RIGHTS RESERVED. Solutions to Exercises - Chapter 8 OF -207 +05 0.258, 28-207 +05 Thus, the system response is given by 0.5(0.5-2)F, fee} = 207 +05 | sinor x (0) SF, 20° +0.5 (b) The non-dimensional frequeney-response functions are given by Xx, (0.5-9°) HQ)= = G0 0s () Fe 2 05 0.5 H,(Q) = = a) Ri/k, OF-20? +05 which are plotted below. Solid lines are used to show Hj1(®) and broken lines are used to show H1(Q). HH, 0) AS Figure £8.13 269, (©2009 Cengage Learning. ALL RIGHTS RESERVED. Solutions to Exercises ~ Chapter & MATLAB program used to generate figure for Exercise 8.13 Om=linspace(0,2,201); rsort(sqrt(roots({1 -2 0.5); rep}-findOm>r2); H21-0.5.(Om.4-2%0m/2+05); HI1=(05-Om_2).Om.4-2°Om,/2+0.5); plot(Om(reg!),HI 1(regl),k-',Om(reg2),HI I(reg2),k-Om(reg3),HI 1(reg3);k-') hold on plot(Omiregt) 21 (reg) k~,Om(reg2) H21(1ep2)K-~ Omi eg) 2 (F3).K-~) plow(o 200} 41) (D2 3.4 ) tevte()+02.2NOmeg_ nama) si) wum2str(r(2))},fontsize’14) yylabel(H_{11}(Omega),H_{21}QOmega),fontsize’ 14) ‘set(gea,fontsize’.14) 8.14 Based on the response amplitudes determined in Exercise 8.8, construct the frequency-response functions G,,(GQ) = X,GQ)/F, G(JQ) = X,GQ)/F, where Q is the nondimensional excitation frequency. Plot graphs of the amplitudes and phases of the each of these functions as a function of Q, compare them to the plots of frequency-response functions determined in Exercise 8.1, and discuss their differences and similarities. Solution 8.14 From the solution to Exercise 8.8, we have that 05+0.1414j = 0.5657 j —2.040? + 0.2828 jO +0.5000) Q? +0.4243 jQ. 2(2" - 0.5657 jQ? - 2.040? +0.2828 j2+0.5000) G(jQ)= in(JQ) 30" G(JQ)= ‘The amplitude and phase of each of these functions are plotted below. Solid lines are used to show the magnitudes (G,(/Q)| and the broken lines are used to show the respective phases. Comparing the results presented in the solution to Exercise 8.1, we see that here the responses are finite-valued at the resonances Q, = 0.5412 and Q2 = 1.3056. This is due to the presence of damping in this case. Phase change characteristics of the system at resonance are seen in the results shown here. On examining the phase information, it is clear that for © <1 the sign of the ratio of the frequency-response functions is the same in the undamped and damped cases. 270 (©2009 Cengage Learning. ALL RIGHTS RESERVED. 16,00 16,609) Solutions to Exercises ~ Chapter 8 --— 180 (0) oseug 05 1 asaee Figure E8.14 271 (©2009 Cengage Learning. ALL RIGHTS RESERVED. Solutions to Exercises ~ Chapter 8 ‘MATLAB program used to generate figures for Exercise 8.14 wr I/sqr(2); mr=1; 21~.1*sqr(2); 220.1; wend=1.5; D2=[1 221+2*22* wet me+2%22*wr) | 4mrt wr Dwr 24442142241) 242222 wD wr]; ‘Om=linspace(0,wend,201); D=Om.4+D2(2)*Om."3*}+D2(3)*Om."2+D2(4)*Om*}+D2(5); HI2=(wr°242%22" wr*Om)/D; all12=abs(H112); pHI2-angle(Hi12)*180/pi; H22=(L+me*wr°2-Om/22°(21+22*mewr)*Om)/D; aH22=abs(H22); | pH22-angle(H22)*180/pi; figure(1) lim=ceil(max(aH12)); plot(Om,aH12,'k-',Or axis((0 wend 0 lim), xlabel(\OmegeYontsize14) ylabel(|G_{12}(\Omega)},fontsize’ 14) text(1.03*wend,lim/2,num2str(0),'fontsize’,14) text(1.03*wend,0,num2str(-180),'t , lim*(pH12+180)360,k—') ntsize',14) text(1.03*wend, lim num2str(180),fontsize' 14) text(1.08*wend, 5*lim, Phase (\cie rotation’ -90, HorizontalA lignment’ center, fontsize’ 14) set(gea,'fontsize' 14) figure(2) lim=ceil(max(aH122)); plot(Om,al122,'k-,Om, lim*(pH22+180)/360,k—) axis({0 wend 0 lim)) xlabel("\Omega,fontsize'14) ‘ylabel(|G_{22}(\Omega)).'fontsize’14) text(1.03*wend,lin/2,num2str(0),fontsize’ 14) text(1.03*wend,0,num2str(-180),Tontsize’,14) text(1.03+wend,lim,num2str(180),fontsize',14) text(1.08*wend,.5*lim, Phase (\citc), rotation’ -90,'HorizontalAlignment’‘center'fontsize',14) set(gea, fontsize’,14) Section 8.3 8.15 Put the linearized system governing “small” amplitude motions of the pendulum- absorber system of Exercise 7.1 in state-space form. Solution 8.15 From the solution to Exercise 7.1 in matrix form, we obtain (m,+m,) mLI[¥] Tk 0 Tfx]_ [sO mL — mE ||6S* 10 mel. alt 0 } Then, from Eq. (8.58), with [#1] = [G] =[C] =[0] we obtain 272 ©2009 Cengage Learning, ALL RIGHTS RESERVED. Solutions to Exercises ~ Chapter 8 wf BLOB (a2? 22) [owe] Loe | (“ay Noting that the state vector and the force vector are, respectively, givea by x o x 6 (l- and Y= (a 3+ [8] ‘we arrive at the state-space form 8.16 Derive the governing equations of motion for the system shown in Figure E8.16 and present them in state-space form, Solution 8.16 From the solution to Exercise 7.6, we have the system matrices w(t 2] tel-[*5* j] [x]-[Eththth “h-& -h-k jth, Then, from Eq. (8.58), with [H] = [G] = [0], we obtain 273, (©2009 Cengage Learning. ALL RIGHTS RESERVED. Solutions to Exercises ~ Chapter 8 00 10 00 o1 m, 0] [ki +k, +h +k -k-k m, 0 he | teen | eee Om, pol [2] 00) “|fm, 07! om, Noting that the state vector and the force vector are, respectively, given by lal te, 0 & the governing equations of motion in state-space form are given by {F}=[4ly} +L) =[Alty} 8.17 Obtain a state-space form of the equations of motion governing “small” amplitude motions of the pendulum-absorber system shown in Figure £7.10. Assume that oscillations about the static-equilibrium position corresponding to the bottom position of the pendulum are of interest. Solution 8.17 When the system of Figure E7.1 undergoes “small” oscillations about the static equilibrium position g, = 0, the governing equations of motion take the form eee ee ‘Then, from Eq. (8.58), with [H] = [G] = 0, we obtain 274 (©2009 Cengage Learning. ALL RIGHTS RESERVED. Solutions to Exercises ~ Chapter & 00 10 00 ol _[M+m 0]'Tk 0] [M+m 0]'fe 9 0 ml’} [0 mg. 0 mi} (06, 00 00 M+m 07' 0 me Noting that the state vector and the force vector are, respectively, given by lal [s]- E,cosot 0 é we arrive at the state-space form O}-14104 +1214} 8.18 Present the governing equations of motion for “small” oscillations of the airfoil system shown in Figure E8.18 in state-space form. In this system, the stiffness of the translation spring is &, the stiffness of the torsion spring is k, G is the center of the mass of the airfoil located a distance / from the attachment point, m is the mass of the airfoil, and Jg is the mass moment of inertia of the airfoil about the center of mass. Use the generalized coordinates y and 0 to obtain the equations of motion, where y is the vertical translation of the point O’ and @ is the angular oscillation about an axis normal to the airfoil plane. Solution 8.18 After using the solution to Exercise 7.13 and noting that we are considering oscillations about the equilibrium position 0, = mglcos0, we obtain the following governing equations of motion for “small” oscillations about the equilibrium position m — -mleos®,|[#|_ fk 0 $]_ fo 0 k,+mglsind, ||6{ |0 215 -mlcos0, J+ml? ‘©2009 Cengage Learning. ALL RIGHTS RESERVED. Solutions to Exercises ~ Chapter 8 Then, from Eq. (8.58) with [#1] = [G] = [C] = 0, we obtain 00 00 m — -mlcos6,)'[k 0 00 micosO, J, +ml? | |0 k,+mglsind, 00 [l- [ m — —mlcos oe {y}= x 6 the state-space form of the system is given by P= +e} =l4lt%} 8.19 The damping matrix given in Exercise 8.6 is altered so that the damping matrix now has the following structure cy 0 07) [02 0 0 0c Of=| 0 06%, 0 O00 0 0 0.Woy Determine the response of the flywheels for the inertia and stiffness parameters given in Exercise 8.6 and the initial conditions in case (a) of Exercise 8.6. Solution 8.19 ‘The governing system of equations in matrix form is [Lt +1C]o} +1& Ko} = {0} where Jo 9 O07 [1 00 [Y]=] 0 J. 0 |=|0 4 0} kg-m® 0 0 Ja} [0 01 276 (©2009 Cengage Learning. ALL RIGHTS RESERVED. Solutions to Exercises ~ Chapter 8 and, from Eq. (7.5b) with an obvious change in notation, kath, ky 07] [20 -10 0 [K]=| -ka ka ths ka }=]-10 15 -5| N-mv/rad Otek \O) as 5 and 02Jy, 0 0] fo2 0 0 [c]l=] 0 067. 0 |=|0 24 0] Nims) 0 0 02%) [0 0 02 Since the system is not proportionally damped as in Exercise 8.6, we first use the state-space form of the system to determine the free response. To this end, we first form [4], which becomes 000 100 000 o10 oo00 ool [4]= 7 a 1 0 Oy'[20 -10 0 10 0y'fo2 0 0 -|0 40 -10 15 -5} -|0 4 0 0 24 0 ool o -5 5 ool 0 02 which evaluates to 0 0 0 1 0 0 0 0 0 0 1 0 [a= 0 0 0 0 0 1 “|-20 10 0 02 0 0 25 -3.75 125 0 “06 0 0 5 5 0 0 02 Since the external forcing is zero, the state-space form of the system is P-L) ‘The initial conditions are given by the column matrix (0) 1 (| |1 (0) 1 YO}=s 0 p= PO) 501710 (0) 0 60) [0 217 (©2009 Cengage Learning. ALL RIGHTS RESERVED. Solutions to Exercises ~ Chapter 8 We use the MATLAB function ode45 to obtain the rotational responses of the three inertial elements. These are given below. 40) rad 10 1) Figure £8.19 ‘MATLAB program used to generate figure for Exereise 8.19 fuention Exercise8_19 Mediag({1 4 ID K=[20-10 0; -10 15 -5;0-5 5}; iag(l2 24 2); zer0s(3,3) eye(3);-inv(M)*K -inv(M)*C] [ty}-ode45(@cop,(0 20}{1 1 100 0}(),A); plos(ty(1, ke) hold on plot(y(,2)/k~) plot(y(3):-) plot((0 20},(0 0)k-) [ab]-legend('phi_1(),%phi_2(0;"\phi_3¢)); set(6(1),fontsize’ I xlabel¢t (3) fontsize’, 14) ylabel(iphij() rad fontsize’14) set(gea fontsize'14) function co=cop(t,Y.A) *Y: 8.20 Consider the two degree-of-freedom system with two different damping models treated in Example 8.9 and obtain the state matrix for this system. Study the eigenvalues and eigenvectors of this state matrix for the systems with constant modal damping and 278 (©2009 Cengage Learning. ALL RIGHTS RESERVED. Solutions to Exercises ~ Chapter 8 arbitrary damping models. Also, compare these eigenvalues and eigenvectors with those for the undamped system, draw comparisons, and discuss them, Solution 8.20 Based on the information presented in Exercise 8.9, the damping matrix for constant modal damping has the form 0377 0.246 c]= NK (ch [‘ 246 oom | is) and in the arbitrary damping ease has the form 0821-0246 (cl, [is 1144 Noting from Example 7.16 that the mass and stiffness matrices for the system have the form | Nims) ba-['y pal 3 w-[3 Fal a wwe construct the state matrices for the case with constant modal damping and for the case with arbitrary modal damping. Thus, from Eq. (c) of Example 8.8, we arrive at the following state matrices State matrix for constant modal damping case 0 0 1 0 i4-| 0 0 1 N}-30/1.2 20/1.2 -0.547/1.2 0.246/1.2 20/2.7 -35/2.7 0.246/2.7 -0,900/2.7 State matrix for arbit modal dampin 0 0 1 0 0 0 oO 1 [4b=) sor.2 20/12 -ogaii2 0246/2 20/27 -35/2.7 0.246/2.7 -1.144/2.7 ‘The eigenvalues and eigenvectors of the state matrices [4], and [4]p are determined by using the eig function in MATLAB. The results are Constant modal damping case Eigenvalues 0.2720 5.616 1226+ 72.516 Ang” 279. (©2009 Cengage Learning, ALL RIGHTS RESERVED. Solutions to Exercises ~ Chapter 8 Eigenvectors 0.0107 70.1624 0.0209 0.2450) 4 0.0058 + 70.0643 | (24, = {00mm v0.23 tha 0.9151 2 10.6194 F j0,0069 0.3632 F 0.0091 0.6930 Arbitrary modal damping case Eigenvalues 3681+ j5.611 0.1859 + 2.512 Eigenvectors ~0.0107¥ 0.1624) 0.0209 70.2450 ,.- 0.0058 + 0.0643 m 0.0203 F 0.2743 Be 0.9151 : P| 0.6194 ¥ 70.0069 0.3632 + 0.0091 0.6930 As expected, the eigenvalues are different in the two different cases of the damping matrix. This is apparent when one examines the real parts of the eigenvalues. Comparing the eigenvalues and eigenvectors of the damped systems to that of the undamped systems, we see that the structure of the eigenvectors observed in the damped case is not preserved in both cases. MATLAB program used to generate some of the resulls for Exercise 8.20 Al=[0010 0001 -30/1.2 20/1.2 -0.547/1.20.246/1.2 20/2.7 -35/2.7 0.246/2.7 -0.9/2.T}; A2=[0010 0001 -30/1.2 20/1.2 -0.821/1.2 0.246/1.2 202.7 -35/2.7 0.246/2.7 -1.1442.7); [Modi E1}-eig(A1) [Mod2 E2}eig(A2) Section 8.5 8.21 Study the micromechanical filter of Example 8.13 and graph the amplitude response of the micromechanical filter similar to that shown in Figure 8.16 for ka: = 0.1 and i = 0.0015. Upon using these parameters in a MATLAB program of Eq. (g) of Example 8.13, we obtain the amplitude response shown below. We see that the choice of parameters is such 280 (©2009 Cengage Learning. ALL RIGHTS RESERVED. Solutions to Exercises — Chapter 8 that the system no longer acts as a filter, but as a two degree-of-freedom system with two well-separated natural frequencies. p s 10 | H,,(@) (48) ar) 095 1 105 14 148 2 Figure £8.21 MATLAB program used to generate figure for Exercise 821 function Exercise8, 21 70.0015; k21-0.1; x5-0.90; xe=1.153; Om=linspace(xs,re,400); DS~Dsfilter(Om,z,k21); mxl=max(D5); D5=20*logI0(D5/mx1);, plot(Om,D5,~) ays Mayas BY oO: xlabel(\Omega’fontsize'14) ylabelCH_{21}(Omega) (4B);fontsize’,14) set(gea, fontsize’14) function Ds = Dsfilter(Om,z,k) DS=abs(k/(Om.\4-4*Om,/3*24i-(44z2+2+2%k)*Om.2+(4t2 424k) *Omeit2 kt): 8.22 For the system of Exercise 8.12, verify that the frequency-response functions determined by using the unit impulse excitation fs (r) = 6(¢) agree with those determined in that problem. 281 (©2009 Cengage Learning. ALL RIGHTS RESERVED. Solutions to Exercises — Chapter 8 Solution 8.22 We use the material from Section 8.5 that pertains to the two degree-of-freedom system shown in Figure 8.8. The system of Exercise 8.12 is a special case of the system shown in Figure 8.8, with the absence of spring ks and the dampers c and ¢), From Eqs. (8.101), we obtain __ BUQ) KG,(JQ) = m,D,GQ) GD) KG_( j0) = A m,D,(jQ) '. (8.102) reduce to A(jQ) =-Q? +1+m0} BQ) D,(jQ) = D, =O! ~[1+m,0} +0 |]? +0} where, for Gi = = 0, ‘Thus, o KG, JQ) = 2 (IQ) =F 1+m,@? -Q? m,D, KiGzy(JQ) In the solution to Exercise 8.12, we found that since, from Eq, (7.41), ‘Thus, the frequency-response functions obtained through the different methods are the same, as expected. Section 8.6.1 8.23 An industrial-fan system, a model of which is shown in Figure E8.23, is found to experience undesirable vibrations when operated at 600 rpm. It is assessed that these undesirable vibrations are due to a mass unbalance in the fan, and it is estimated the mass unbalance m, = 2 kg is located at a distance e = 0.5 m from the point 0 shown in Figure 282 (©2009 Cengage Learning. ALL RIGHTS RESERVED. Solutions to Exercises ~ Chapter 8 E8.23. Design a vibration absorber for this industrial-fan system with the restriction that the mass of the absorber cannot exceed 75 kg. Solution 8.23, The excitation frequency is oe 600 rev/min x 2 rad/rev 60 s/min To satisfy the mass restriction on the absorber, let mz = 70 kg. Then, since @2 = @1 = @, the absorber stiffness is ky =m,o73, = m,o* = 1062.83 = 276.3 KNim = 62.83 rad/s 8.24 For the industrial-fan system of Exercise 8.23, if the original restriction on the absorber mass is removed, design the absorber with the following new restrictions: (a) the natural frequencies of the system with the absorber should lie outside the range of 575 1pm to 625 rpm and (b) the response amplitude of the absorber mass should not exceed 25 mm. Solution 8.24 From Eq, (K) of Example 8.3, we have that the non-dimensional natural frequencies of the system with the absorber are given by Qf -(2+m,)Q? +1=0 where m, is the ratio of the absorber mass to the mass of the primary system. Upon solving for m,, we obtain Since @, = 600 rpm from Exercise 8.23, we have that at @ = 575 rpm. . (575/600) +1 _ o,o0725 (575/600) and at @, = 625 rpm ‘600\' = (25/600) +1 _» _ 9.0067 (625/600)° Hence, any value of m, > 0.01 will provide natural frequencies that are outside the required limits. From Eq, (j) of Example 8.3, we have that the maximum displacement requirement is eam, h Therefore, the absorber spring constant needs to be such that =0.025 m Inl-Z- k 283 ‘©2009 Cengage Learning. ALL RIGHTS RESERVED. Solutions to Exercises ~ Chapter 8 or =157.9 kNim > Sextebr) x2 Os 0.025 where we have from Exercise 8.23 that ¢= 0.5 m, m, = 2 kg, and @ = 2600/60 = 20 rad/s. Therefore, we'll select k= 160 kN/m. The absorber mass m) needs to be such that ‘m,/my > 0.01, where mis the mass of the primary system. 8.25 A machine has a mass of 150 kg and a natural frequency of 150 rad/s. An absorber mass of 30 kg and a spring-damper combination is to be attached to this machine, so that the machine can be operated in as wide a frequency range as possible around the machine’s natural frequency. Determine the optimal parameters for the absorber. Solution 8.25 ince the damping of the primary system is zero, we can use Eq, (8.118). For the mass ratio m, = 30/150 = 0.2, @, 0.8333, ro Lem, 140.2 3m, 3x02 = [3m = |_3%02 _ _ 99083 See 8(l+m,)° ¥8(1+0.0.2)° Since we have that the absorber stiffness is ky =m,03, = m,0,,0, = 300.8333" «150? = 562.5. kN/m Also, since pees oman we have that the absorber damping coefficient is 6, = 2p My =2 0.2083 x 30 x 0,8333%150 = 1561.6 Ni(nw/s) 8.26 In order to attenuate oscillations of telecommunication towers, tuned vibration absorbers are typically used. The second natural frequency of a representative telecommunication tower is 0.6 Hz and the third natural frequency of this tower is 1.5 Hz. Design two optimal absorbers; one to be effective in a frequency range that includes 284 (©2009 Cengage Learning. ALL RIGHTS RESERVED. Solutions to Exercises ~ Chapter 8 0.6 Hz and another to be effective in a frequency range that includes 1.5 Hz. The mass ratio for each absorber can be picked to lie in the range from 0.02 to 0.05. Solution 8.26 From Eq, (g) of Example 8.3, we have that the two natural frequencies of the system at the optimum frequeney ratio (1+o2d+m,)) 403 7=2,3 Siu = for cach absorber modified system. Here, fi,y are the lower and upper natural frequencies of the system, respectively, and fi = 0.6 Hz and fj = 1.5 Hz. Since there are two resonance frequencies of the original system, we impose a limit that for fs = 0.6 Hz, fu < 1 Hz and for f= 1.5 Hz, fi. > 1 Hz. ‘The easiest way to determine the vale of m, in each case is to graph fi,y as a function of m, as shown below. From this graph, we select m, = 0.05 for both tuned dampers. = 1+@2(1+m,) Figure £8.26 ‘MATLAB program used tot generate fi ixereise 826 ‘mr-Linspace(0,2,70); wr=L (14m). 2; a=L+wr.*(1+m 285 ©2009 Cengage Leaming. ALL RIGHTS RESERVED. Solutions to Exercises — Chapter 8 iA(0.5¥(arsqri(a 2-4.) fumsqrt(0.5*(atsqri(a."2-4*wr."2))); plot(mr,f1*0.6,k-,mr,fu¥0.6,k-~"') hold on plot(mr,f*1.5 xlabel('m_r' fo anf 1.5, ize’ 14) fontsize’,14) 8.27 An optical platform is found to be experience undesirable vibrations at a frequency of 100 Hz. The associated magnitude of the disturbance acting on the platform is estimated to be 100 N. Design a spring-mass system as an absorber for this system with the constraint that the absorber response amplitude cannot exceed 5 mm. Solution 8.27 From Eqs. @) of Example 8.3, the condition on the absorber response amplitude can be expressed as la]= 4 <0.005 m ky which results in Paes 100 2" |x,] 5x107 20 KN/m In addition, we choose the absorber so that «2 = qi = «, from which we arrive at ky ky 20000 _ oy @ (2nx100)" =0.051 kg Section 8.6.2 8.28 A rotary engine experiences disturbances at the sixth harmonic of the rotating speed ofthe system. Determine the pendulum length of a centrifugal absorber for this system. Solution 8.28 From Eq. (8.136), the non-dimensional pendulum length is 2 (60, Ro o@ Y 36 ‘Therefore, r = 36R, which means that the pendulum length must be 36 times the distance from the pendulum’s pivot attachment point to the center of rotation. Section 8.6.3 8.29 Determine the free responses of the bar-slider system described by Eqs. (8.144) for the following parameter values: m = 0.5, £= 0.20, and @. = 0.5, 1.5, 2, and 2.5, Assume 286 ©2009 Cengage Learning. ALL RIGHTS RESERVED. Solutions to Exercises ~ Chapter 8 that in each case, the motions are initiated from (0) = 0.5 rad with all of the other initial conditions being zero. Graph the time histories for the radial displacements of the slider and the angular motions of the bar. Discuss the effectiveness of the nonlinear vibration absorber in suppressing the angular motions of the bar. Solution 8.29 ‘The results are shown below. From these results, it appears that the angular motions are attenuated the most for the frequency ratio @ = 2.0. os 05 A - = of go 05) o205 \ om 4 6 a “9 2 «0 6) 0 os eu eof AMMA ANAn 4 ests VV 0 20 40 60 80 055 20 400 «60 10 7 “TAA ONAN pnen | = PANN a ame ae 0 20 «40 a0 0 20 40 «60 80 05 0.06) . zo zd 0.05 e258 oO 2 840 «660 80 055 20 40 «60 (80 Figure £8.29 ‘MATLAB program used to generate figure for Exercise 8.29 function Exereise8_ 29 2-02; m-05; wer[0.5 1.52 2.5]; Ne-lengih(we): Tim=[0.8 0.16 0.14 0.09]; Jed=80; ti-0.5; forkeLN [ty}-ode45(@barslider,[0 led], [0 0 ti 0),{),we(k),m,z); subplot(N.2.2%(k-1}#1) plot(ty(:,.1),'k-[0 led], [0 0],'k-') ylabel('r(\tau)’,'fontsize’,14) Ser(gea,fontsize'14) text(50,-2"lim(k)3,[lomega_e~' num2str(we(k))}fonsize’,14) ifk—N ‘xlabel(‘\tau’,'fontsize’,14) end axis({0 led -lim(k) lim(k)]) subplot(N.2.2*¢k-1)2) 287 ‘©2009 Cengage Learning. ALL RIGHTS RESERVED. Solutions to Exercises — Chapter 8 plot@y(3)47(0 Ted} [0 0}-4-5 ‘ylabel(‘theta( tau), fontsize’, 14) Set(gea,fontsize 14) ifk=N xlabel(\taufontsize14) end axis([0 led tt) end function vee=barslider(ty,we.m.2) vvee=[y2)(1+9(1))*y(AY 2-224 wery(2)-we"2*y(1):yA)s24m*(L4¥()*¥2)*VA).- -¥GV+2m*Csy))* VOI Section 8.6.4 8.30 Determine the free responses of the pendulum-absorber system described by Eqs. (8.153) for the following parameter values: Q = 10, G = 0.05, mt, =0.10 and f, = 0.4. Assume that in each case, the motions are initiated from g(0) = 0.2 rad with all of the other initial conditions being zero. Graph the time histories for the vertical motions of the system and the angular motions of the pendulum. Discuss the effectiveness of the nonlinear vibration absorber in suppressing the vertical translations, Solution 8.30 The results are shown below. From these results, it is clear that for this choice of absorber parameters, the system is not effective in suppressing the vertical translations. 7 20) os) 0. AVN 288 (©2009 Cengage Learning. ALL RIGHTS RESERVED. Solutions to Exercises ~ Chapter 8 MATLAB program used to generate figure for Exercis function Exercise8_30 smr0.1;xi-0.1; xi0.05; Om=1; wr=0.5; fo=0.4; {tw}-ode4s(@PendulumAbs|,{0 300}(0 0 02 0},(Jamrwr-xisxitOm,); figure(1), plot(t,w(:,1),'k-') labei('tau,Yonsize14) ylabel¢tau)fonsiz’14) set(gca, ontsize’ 4) figure(2) plot((,3)) ‘xlabel(\tau’,"fontsize’, 14) label phi(au)''ontsize 14) ‘set(gca,'fontsize’,14) function Q=PendulumAbs (want, wrxixit,0m,fo) ‘Asfo*cos(Om*t)-w(I)-2*xi*w2)mr*w(4Y2*eos{w(3)}; B=2*xit*w(4)-wr2¢sin(w3)), Pe(B-Atsin(ov(3))/(L-mr*sin(w(3))2); me“Bsin( (3) -mesingw(3)72) LQ=fw(2)-Ziw(a)iP}; Section 8.7 8.31 A 200 kg machine is to be operated at 500 rpm on a flexible platform that can be modeled as a single degree-of-freedom system with the following mass, stiffness, and damper values: m = 3000 kg, k= 2 x 10° N/m, and c; = 7500 N/(mv/s). Determine the 289, (©2009 Cengage Learning, ALL RIGHTS RESERVED. Solutions to Exercises ~ Chapter 8 ry mounting so that the transmissibility ratio does not exceed properties of the machi 0.1. Solution 8.31 From the parameters given, we have that Using the results given in Figure 8.32 as a guide, we see that for a TR < 0.1 will be attained for Q = 2 if @ <0.9. Therefore, we shall assume that w, — 0.8 and <3 — 0.05. Then, using Eqs. (8.156) and (8.102), we obtain (2612+1) 2¢,0,j0+07) D,(jQ) where D,(jQ) = QF = j126, +2G,0,m, + 2G,0, }2 —[1+m,07 +07 +4¢6,6,0,]0° + j[26,0, + 26,@7]2 +0? Upon substituting in the above parameters and using MATLAB to perform the numerical calculation, we obtain that 7R = 0.0627 < 0.1. Thus, the properties of the machinery mounting are determined as follows. k, = @)m, = oF 0},m, = 0.8? x 25.81 x 200 = 85.27 KN/m 6, = 2Sym0,, = 2m 0,0,, =2*0.05%200x0.825.81=413 Niaw/s) MATLAB program used to generate some of the results for Exercise 8.31 21=0.048; 22=0.05; mr=0,0667; (Om=2.03; wr-0.8; D2=abs((Om.4-(i-+mrtwe.“2+wr 4244421 *22.¢wr).*0m,/2+Ww1."2).. 4572421422 wrt mrz2*wr),*Om./3+29(22*wrzl *wr."2),*Om)); 9((142*21*Om*))*(2*22* wr*Om*j+wr"2)); TR=N/D2 8.32 Construct the transmissibility ratio plot of Figure 8.32 for the following parameters ofa two degree-of-freedom system: m, =0.2 and ¢1 = ¢2 = 0.1. 290 (©2009 Cengage Learning. ALL RIGHTS RESERVED. Solutions to Exercises — Chapter 8 Solution 8.32 ‘The solution is shown below. TR=0.1 Figure £8.32 MATLAB program used to generate figure for Exercise 8.32 function Exercise8_32 z1-0.1; 220.1; mr=0.2; TR=[0.1 3}; Oend=4; ws=[3.3 4}; wLen=10; rzeros(wLen,1); ‘Omm=[linspace(0.1,1.2,15)linspace(1.2,Oend,25)}; ‘wrr=linspace(0.1,2.5,48); [Om, wr}-meshgrid(Omm,wre); h-HTR2dof{Om,wr,m,21,22); rmesh(wr,Om,h) opt-optimset(Display''oft); for n=1:2 inspace(ws(n),2.5,wLen), for K=I:wLen k)-feer0( @TEifg Oendl,optwrt(K),mr.2,22, F(a); err(h; end hold on {(swLen)=TF(n); plot3(wrtyfr.1k''Linewidth’2) end colormap((0 0 0}) xlabel(iomega_r,fontsize’14) yilabel(' Omega, Yontsize’ 14) label(TR: fontsize’14) text(2.5,4.2,5,[TR~ num2str(TF(1))] fontsize' 14) text(2.5,3.2.5,(TR~' num2st(TF(2))},fontsize’ 14) sete, fontsize14) 291 (©2009 Cengage Learning. ALL RIGHTS RESERVED. Solutions to Exercises ~ Chapter 8 VOP=25; MORIS: axis(v) grid off vview([-80 70)) function h-HTR2dof{Om,wrmr,21,72) realpart-Om.4-(I+mr*wr.”24wr."2+4*z1*22.*wr).*Om."2+wr2; imagpart=-2*(z1+22*wrtmr+22* wr).*Om."3+24(@2*wr+Z1*Wr."2). nrealpart-mr* wr."2; nnimagpart~2*72.*wr.*mr.*Om; Ni-sqrt(nrealpart.°2-nimagpart."2); N2=sqrt(1+(2*21*Om)."2)mr; h=N1.*N2./sqrt(realpart."2+imagpart.“2); 8.33 A model of an electronic system m; contained in a package m is shown Figure 8.33. Determine the transfer function Xs(s)/X3(s) and show that it has the following form where the polynomials C(s) and D;(s) are given by Eqs. (8.64) and (8.74), respectively. Solution 8.33 ‘The system shown in Figure E8.33 can be treated as a special case of the system shown in Figure 8.33. From Eqs. (8.168), we have that K(s)C(s) D,(s) X,(s8) = where, from Eq. (8.169) Kis) = 26s +DX\(3)- 26.40) Since we are interested in the transfer function only, we set the initial condition x3(0) = 0 and find that X,(s) = LES#DKOIC) : Dis) which leads to the ratio X4(s) _ (26,8 +I)C(s) Xs) DCs) 8.34 Obtain the transfer function between the absorber response and the base excitation in the vehicle model shown in Figure E8.34. 292 ©2009 Cengage Learning. ALL RIGHTS RESERVED. Solutions te Exercises ~ Chapter 8 Sok n 8.3: ‘The governing equations for this system have been obtained in Exercise 7.7. They are my, + hy hy. t oH, —e2=0 min + (ke + hy +B) Ya BY ~BVs + (Co +0, + O3)P2 — 0) CIs = KaVe HOI Mj + Kays kV2 + O35 — C559 = 0 We take the Laplace transform of these equations with all initial conditions set to zero to obtain m,s°Y,(8) + kY,(8) —K¥,(s) + 68%; (s) —€,8¥,(8) =0 m,S°Y,(3)+ (ky +k + )E(8) hi (9) bY) +(e +6, +6,)8%x(8)— G84, (8) —e8Y(5) =BY.(8) +057, (8) m,s°Y,(8) + k,¥,(s) — bY, (s) + ¢,s¥,(s)—¢,s¥,(s) =0 Collecting terms and rearranging, we have (ms? +e5 +k )¥(s)-(k, Fes) (5) =0 (ky +68) ¥ (5) + (ms? +(c, +6, +0,)5+k +h +h) = (ky +e) =(k tes) F8) =(ky +e) ¥, (9) +(ms? +658 +) ¥(s)=0 Putting these equations in matrix form we get (s]}={%) where -k- 6s 0 mS +(+6,+e)s+kthtk — -b-es ms’ + 0,8 +k, ‘The transfer function ¥3(s)/¥.(s) is 293 ‘©2009 Cengage Learning. ALL RIGHTS RESERVED. Solutions to Exercises ~ Chapter 8 |m,s? + cs + ky -k-¢s 0 2 Fag] “Ao88 MP HG tq ta) ththth ky tes (8) dels} -ky—o,s 0 1 aes" +e +k) (k, +¢,5)(k +08) 8.35 The vertical motions of an automobile are described by using the quarter-car model shown in Figure E8.35. The different system parameters in this two degree-of-freedom model are as follows: m = 80 kg, m2 = 1100 kg, k = 30 KN/m, ki = 300 kN/m, and c> = 5000 Ni(m/s). When this vehicle is traveling with a constant speed on a flat road, it hits a bump, which produces a base displacement of the form x3(0) = 0.2[u() — u(t - 0.5)] m. Determine the ensuing response of this system. Solution 8.35 The governing equations for this system are a special case of those given by Eqs. (8.166) and (8.167). Sei in these equations, we obtain 2 Pi 2¢mo,+(+m0})x, -2¢,m0, #2 —mo2x, de fr 2 © 4260, 82+ ox, -2¢,0, 41 o'x, =0 dr where x3 is the displacement of the base and the non-dimensional quantities are as defined in Eqs. (7.41). Then it follows from Eqs. (8.167) and (8.168) that X,(s)(s° +24,0,8+02) X(9)= Be _X(9)(26,0,5+0? ae where Djs)=s* +[26,0,m, + 2¢,0,]° +[1+m,o} + @} |s*+ 26,05 +03 In terms of the non-dimensional variables, the base displacement is x(t) =0.2[u(r)=u(r -7,)] where 1, = 0.5aq1. The Laplace transform of this quantity is obtained fiom entry 8 Table ‘A of Appendix A, which results in Solutions to Exercises ~ Chapter 8 m, unis 0 te [A _ [30x10" 5.292 rad/s mV 1100 py _ 61.237 _ 9 9953 @, 5222 & 5000 = 0.4352 2m,0,, 2*1100%5.222 Hence -ossmas) _ 0.2 e = X,(s)= 221 and the responses of the masses mm, and m in the Laplace domain are 0.2(s? +2¢,0,s+0?)(1-e™ Hor sDx() _ _0.2(26,0,8+03)(1-e2"" 2(s) = sale) ‘The numerically obtained inverse Laplace transforms using the MATLAB function ilaplace is given below. Figure £8.35 295 ‘©2009 Cengage Learning. ALL RIGHTS RESERVED. Solutions to Exercises ~ Chapter 8 ‘AB proy re for Exercise 835 syms s mr=13.75; wr-0.0853; co 320.4352; tend10; figure(1) xlsn=s°2+24x2¢wrtstwr2; x2sn=2492* wrtstwr2; xsd=s*poly2symn({1 2%(x2*mr* wrtx2*wr) [+mrtwr°2+wr2 2*x2*wr wr"2].s); xIbase=vpa(ilaplace(xlsn/xsd),5); x2basevpa{ilaplace(x2sn/xsd),5): Xit-inline(veetorize(x1base),t); X2tminline(veetorize(x2base)"); tlinspace(0,tend, 250); pfun1=co *real(X t())-co*real(X1t(¢-2.611)).*(-2.611); pfun2co real(X241))-co*real(X24(t-2.611)).*(O=2.611); plot(tpfiunt,-'t.pfun2,k~[0 tend] [0 0}.\-') set(gca,fontsize’,14) yilabel(x_j(tau) (m);"fontsize’,14) xlabel( ‘tau’ fontsize'14) is az(2)-tend; az{4)-0.26; [abJmlegend(x_1(\tauy,'_2¢tau)); set(b(1),fontsiza 14) (©2009 Cengage Learning. ALL RIGHTS RESERVED.

You might also like